Sie sind auf Seite 1von 214

Threefriendsdividedsomebulletsequally.

Afterallofthemshot4bullets
thetotalnumberofbulletsremainingisequaltothebulletseachhadafter
division.Findtheoriginalnumberdivided.
Answer

18

Assumethatinitialtherewere3*Xbullets.

SotheygotXbulletseachafterdivision.

Allofthemshot4bullets.Sonowtheyhave(X4)bulletseach.

Butitisgiventhat,aftertheyshot4bulletseach,totalnumberofbullets
remainingisequaltothebulletseachhadafterdivisioni.e.X

Therefore,theequationis
3*(X4)=X
3*X12=X
2*X=12
X=6

Thereforethetotalbulletsbeforedivisionis=3*X=18

FindsumofdigitsofD.

Let
1999
A=1999

B=sumofdigitsofA
C=sumofdigitsofB
D=sumofdigitsofC

(HINT:A=B=C=D(mod9))
Answer

ThesumofthedigitsodDis1.

LetE=sumofdigitsofD.

ItfollowsfromthehintthatA=E(mod9)
Consider,

1999
A=1999

2000
<2000

2000
2000
=2
*1000

200
6000
=1024
*10

800
6000
<10
*10

6800
=10

6800
i.e.A<10

i.e.B<6800*9=61200

i.e.C<5*9=45

i.e.D<2*9=18

i.e.E<=9

i.e.Eisasingledigitnumber.

Also,

1999=1(mod9)

1999
so1999
=1(mod9)


ThereforeweconcludethatE=1.
Thereisa50mlongarmyplatoonmarchingahead.Thelastpersoninthe
platoonwantstogivealettertothefirstpersonleadingtheplatoon.Sowhile
theplatoonismarchingherunsahead,reachesthefirstpersonandhands
overthelettertohimandwithoutstoppingherunsandcomesbacktohis
originalposition.

Inthemeantimethewholeplatoonhasmovedaheadby50m.

Thequestionishowmuchdistancedidthelastpersoncoverinthattime.
Assumingthatheranthewholedistancewithuniformspeed.
Submitted
Answer

Thelastpersoncovered120.71meters.

Itisgiventhattheplatoonandthelastpersonmovedwithuniformspeed.
Also,theybothmovedfortheidenticalamountoftime.Hence,theratioof
thedistancetheycoveredwhilepersonmovingforwardandbackwordare
equal.

Let'sassumethatwhenthelastpersonreachedthefirstperson,theplatoon
movedXmetersforward.

Thus,whilemovingforwardthelastpersonmoved(50+X)meterswhereas
theplatoonmovedXmeters.

Similarly,whilemovingbackthelastpersonmoved[50(50X)]Xmeters
whereastheplatoonmoved(50X)meters.

Now,astheratiosareequal,
(50+X)/X=X/(50X)
(50+X)*(50X)=X*X

Solving,X=35.355meters

Thus,totaldistancecoveredbythelastperson
=(50+X)+X

=2*X+50
=2*(35.355)+50
=120.71meters

Notethatatfirstglance,onemightthinkthatthetotaldistancecoveredby
thelastpersonis100meters,asheranthetotallenghtoftheplatoon(50
meters)twice.TRUE,butthat'stherelativedistancecoveredbythelast
personi.e.assumingthattheplatoonisstationary.

Ifyoutakeamarker&startfromacorneronacube,whatisthemaximum
numberofedgesyoucantraceacrossifyounevertraceacrossthesame
edgetwice,neverremovethemarkerfromthecube,&nevertraceanywhere
onthecube,exceptforthecorners&edges?
Answer

Toverifythis,youcanmakeadrawingofacube,&numbereachofits12
edges.Then,alwaysstartingfrom1corner&1edge,youcandetermineall
ofthepossiblecombinationsfortracingalongtheedgesofacube.

Thereisnoneedtostartfromothercornersoredgesofthecube,asyouwill
onlyberepeatingthesamecombinations.Theprocessisalittlemore
involvedthanthis,butisusefulforsolvingmanytypesofspatialpuzzles.
OneofMr.Bajaj,hiswife,theirsonandMr.Bajaj'smotherisanEngineer
andanotherisaDoctor.

IftheDoctorisamale,thentheEngineerisamale.
IftheEngineerisyoungerthantheDoctor,thentheEngineerandthe
Doctorarenotbloodrelatives.
IftheEngineerisafemale,thensheandtheDoctorareblood
relatives.

CanyoutellwhoistheDoctorandtheEngineer?
Answer

Mr.BajajistheEngineerandeitherhiswifeorhissonistheDoctor.

Mr.Bajaj'swifeandmotherarenotbloodrelatives.Sofrom3,ifthe
Engineerisafemale,theDoctorisamale.Butfrom1,iftheDoctorisa
male,thentheEngineerisamale.Thus,thereisacontradiction,ifthe
Engineerisafemale.Hence,eitherMr.BajajorhissonistheEngineer.

Mr.Bajaj'ssonistheyoungestofallfourandisbloodrelativeofeachof
them.Sofrom2,Mr.Bajaj'ssonisnottheEngineer.Hence,Mr.Bajajisthe
Engineer.

Nowfrom2,Mr.Bajaj'smothercannotbetheDoctor.SotheDoctoris
eitherhiswifeorhisson.Itisnotpossibletodetermineanythingfurther.

ThreemenSam,CamandLauriearemarriedtoCarrie,BillyandTina,
butnotnecessarilyinthesameorder.

Sam'swifeandBilly'sHusbandplayCarrieandTina'shusbandatbridge.No
wifepartnersherhusbandandCamdoesnotplaybridge.

WhoismarriedtoCam?

Answer

CarrieismarriedtoCam.

"Sam'swifeandBilly'sHusbandplayCarrieandTina'shusbandatbridge."

ItmeansthatSamisnotmarriedtoeitherBillyorCarrie.Thus,Samis
marriedtoTina.

AsCamdoesnotplaybridge,Billy'shusbandmustbeLaurie.

Hence,CarrieismarriedtoCam.

Thereare3personsX,YandZ.Onsomeday,XlenttractorstoYandZas
manyastheyhad.AfteramonthYgaveasmanytractorstoXandZas
manyastheyhave.AfteramonthZdidthesamething.Attheendofthis
transactioneachoneofthemhad24.

Findthetractorseachoriginallyhad?
Answer

Onewaytosolveitisbymaking3equationsandsolvethem
simultaneously.ButthereisrathereasierwaytosolveitusingBacktracing.

It'sgiventhatattheend,eachhad24tractors(24,24,24)i.e.afterZgave
tractorstoX&Yasmanyastheyhad.Itmeansthataftergettingtractors
fromZtheirtractorsgotdoubled.SobeforeZgavethemtractors,theyhad
12tractorseachandZhad48tractors.(12,12,48)

Similarly,beforeYgavetractorstoX&Z,theyhad6&24tractors
respectivelyandYhad42tractorsi.e.(6,42,24)

Again,beforeXgavetractorstoY&Z,theyhad21&12tractors
respectivelyandXhad39tractorsi.e.(39,21,12)

Hence,initially
Xhad39tractors,Yhad21tractors
and
Zhad12
tractors.
Acertainstreethas1000buildings.Asignmakeriscontractedtonumber
thehousesfrom1to1000.Howmanyzeroeswillheneed?
Answer

Thesignmakerwillneed192zeroes.

Divide1000buildingnumbersintogroupsof100eachasfollow:
(1..100),(101..200),(201..300),.......(901..1000)

Forthefirstgroup,signmakerwillneed11zeroes.
Forgroupnumbers2to9,hewillrequire20zeroeseach.
Andforgroupnumber10,hewillrequire21zeroes.

Thetotalnumbersofzeroesrequiredare
=11+8*20+21
=11+160+21
=192

Thereare9coins.Outofwhichoneisoddonei.eweightislessormore.

Howmanyiterationsofweighingarerequiredtofindoddcoin?
Answer

Itisalwayspossibletofindoddcoinin3weighingsandtotellwhetherthe
oddcoinisheavierorlighter.
1. Take8coinsandweigh4against4.
Ifbotharenotequal,gotostep2
Ifbothareequal,gotostep3

2.

Oneofthese8coinsistheoddone.Namethecoinsonheaviersideof
thescaleasH1,H2,H3andH4.Similarly,namethecoinsonthe
lightersideofthescaleasL1,L2,L3andL4.EitheroneofH'sis
heavieroroneofL'sislighter.Weigh(H1,H2,L1)against(H3,H4,
X)whereXisonecoinremaininginintialweighing.
Ifbothareequal,oneofL2,L3,L4islighter.WeighL2against
L3.
Ifbothareequal,L4istheoddcoinandislighter.
IfL2islight,L2istheoddcoinandislighter.
IfL3islight,L3istheoddcoinandislighter.

If(H1,H2,L1)isheaviersideonthescale,eitherH1orH2is
heavier.WeightH1againstH2
Ifbothareequal,thereissomeerror.
IfH1isheavy,H1istheoddcoinandisheavier.
IfH2isheavy,H2istheoddcoinandisheavier.

If(H3,H4,X)isheaviersideonthescale,eitherH3orH4is
heavierorL1islighter.WeightH3againstH4
Ifbothareequal,L1istheoddcoinandislighter.
IfH3isheavy,H3istheoddcoinandisheavier.
IfH4isheavy,H4istheoddcoinandisheavier.

3.

TheremainingcoinXistheoddone.WeighXagainsttheanyone
coinusedininitialweighing.

Ifbothareequal,thereissomeerror.
IfXisheavy,Xistheoddcoinandisheavier.
IfXislight,Xistheoddcoinandislighter.

Inasportscontestthereweremmedalsawardedonnsuccessivedays(n>
1).
1.
2.
3.

Onthefirstday1medaland1/7oftheremainingm1medalswere
awarded.
Onthesecondday2medalsand1/7ofthenowremainingmedalswas
awardedandsoon.
th
Onthen
andlastday,theremainingnmedalswereawarded.

Howmanydaysdidthecontestlast,andhowmanymedalswereawarded
altogether?
Answer

Total36medalswereawardedandthecontestwasfor6days.

Onday1:Medalsawarded=(1+35/7)=6:Remaining30medals
Onday2:Medalsawarded=(2+28/7)=6:Remaining24medals
Onday3:Medalsawarded=(3+21/7)=6:Remaining18medals
Onday4:Medalsawarded=(4+14/7)=6:Remaining12medals
Onday5:Medalsawarded=(5+7/7)=6:Remaining6medals
Onday6:Medalsawarded6

Igotthisanswerbywritingsmallprogram.Ifanyoneknowanyother
simplermethod,dosubmitit.
Anumberof9digitshasthefollowingproperties:

Thenumbercomprisingtheleftmosttwodigitsisdivisibleby2,that
comprisingtheleftmostthreedigitsisdivisibleby3,theleftmostfour
by4,theleftmostfiveby5,andsoonfortheninedigitsofthe
numberi.e.thenumberformedfromthefirstndigitsisdivisiblebyn,
2<=n<=9.
Eachdigitinthenumberisdifferenti.e.nodigitsarerepeated.
Thedigit0doesnotoccurinthenumberi.e.itiscomprisedonlyof
thedigits19insomeorder.

Findthenumber.

Answer


Theansweris
381654729

OnewaytosolveitisTrial&Error.Youcanmakeitbiteasierasodd
positionswillalwaysoccupyODDnumbersandevenpositionswillalways
occupyEVENnumbers.Further5thpositionwillcontain5as0doesnot
occur.

Theotherwaytosolvethisproblemisbywritingacomputerprogramthat
systematicallytriesallpossibilities

1/3rdofthecontentsofacontainerevaporatedonthe1stday.3/4thofthe
remainingcontentsofthecontainerevaporatedonthesecondday.

Whatpartofthecontentsofthecontainerisleftattheendofthesecondday?

Answer

AssumethatcontentsofthecontainerisX

Onthefirstday1/3rdisevaporated.
(11/3)ofXisremainingi.e.(2/3)X

OntheSecondday3/4thisevaporated.Hence,
(13/4)of(2/3)Xisremaining
i.e.(1/4)(2/3)X=(1/6)X

Hence1/6thofthecontentsofthecontainerisremaining

Vipulwasstudyingforhisexaminationsandthelightswentoff.Itwas
around1:00AM.Helightedtwouniformcandlesofequallengthbutone
thickerthantheother.Thethickcandleissupposedtolastsixhoursandthe
thinonetwohoursless.Whenhefinallywenttosleep,thethickcandlewas
twiceaslongasthethinone.

ForhowlongdidVipulstudyincandlelight?
Answer


Vipulstudiedfor3hoursincandlelight.

AssumethattheinitiallenghtofboththecandlewasLandVipulstudiedfor
Xhours.

InXhours,totalthickcandleburnt=XL/6
InXhours,totalthincandleburnt=XL/4

AfterXhours,totalthickcandleremaining=LXL/6
AfterXhours,totalthincandleremaining=LXL/4

Also,itisgiventhatthethickcandlewastwiceaslongasthethinonewhen
hefinallywenttosleep.
(LXL/6)=2(LXL/4)
(6X)/6=(4X)/2
(6X)=3*(4X)
6X=123X
2X=6
X=3

Hence,Vipulstudiedfor3hoursi.e.180minutesincandlelight.

IfyoustartedabusinessinwhichyouearnedRs.1onthefirstday,Rs.3on
thesecondday,Rs.5onthethirdday,Rs.7onthefourthday,&soon.

Howmuchwouldyouhaveearnedwiththisbusinessafter50years
(assumingthereareexactly365daysineveryyear)?
Answer

Rs.333,062,500

Tobeginwith,youwanttoknowthetotalnumberofdays:365x50=
18250.

Byexperimentation,thefollowingformulacanbediscovered,&usedto
determinetheamountearnedforanyparticularday:1+2(x1),withxbeing
thenumberoftheday.Takehalfofthe18250days,&pairthemupwiththe

otherhalfinthefollowingway:day1withday18250,day2withday
18249,&soon,&youwillseethatifyouaddthesepairstogether,they
alwaysequalRs.36500.

Multiplythisnumberbythetotalnumberofpairs(9125),&youhavethe
amountyouwouldhaveearnedin50years.

Mathgurusmayuseseriesformulatosolveit.(series:1,3,5,7,9,11.....upto
18250terms)
Aworkerearnsa5%raise.Ayearlater,theworkerreceivesa2.5%cutin
pay,&nowhissalaryisRs.22702.68

Whatwashissalarytobeginwith?
Answer

Rs.22176

AssumehissalarywasRs.X

Heearns5%raise.Sohissalaryis(105*X)/100

Ayearlaterhereceives2.5%cut.Sohissalaryis((105*X)/100)*(97.5/100)
whichisRs.22702.68

Hence,solvingequation((105*X)/100)*(97.5/100)=22702.68
X=22176
At6'oaclockticks6times.Thetimebetweenfirstandlastticksis30seconds.
Howlongdoesittickat12'o.

Answer

66seconds

Itisgiventhatthetimebetweenfirstandlastticksat6'ois30seconds.
Totaltimegapsbetweenfirstandlastticksat6'o=5
(i.e.between1&2,2&3,3&4,4&5and5&6)

Sotimegapbetweentwoticks=30/5=6seconds.

Now,totaltimegapsbetweenfirstandlastticksat12'o=11
Thereforetimetakenfor12ticks=11*6=66seconds(andnot60seconds)

500menarearrangedinanarrayof10rowsand50columnsaccordingto
theirheights.

Tallestamongeachrowofallareaskedtocomeout.Andtheshortest
amongthemisA.

Similarlyafterresumingthemtotheiroriginalpositions,theshortestamong
eachcolumnareaskedtocomeout.AndthetallestamongthemisB.

NowwhoistallerAorB?
Answer

Nooneistaller,botharesameasAandBarethesameperson.

Asitismentionedthat500menarearrangedinanarrayof10rowsand50
columns
accordingtotheirheights
.Let'sassumethatpositionnumbers
representtheirheights.Hence,theshortestamongthe50,100,150,...450,
500ispersonwithheight50i.e.A.Similarlythetallestamong1,2,3,4,5,
.....48,48,50ispersonwithheight50i.e.B

Now,bothAandBarethepersonwithheight50.Hencebotharesame.

InMr.Mehta'sfamily,thereareonegrandfather,onegrandmother,two
fathers,twomothers,onefatherinlaw,onemotherinlaw,fourchildren,
threegrandchildren,onebrother,twosisters,twosons,twodaughtersand
onedaughterinlaw.

HowmanymembersarethereinMr.Mehta'sfamily?Giveminimalpossible
answer.
Answer

Thereare7membersinMr.Mehta'sfamily.Mother&FatherofMr.Mehta,
Mr.&Mrs.Mehta,hissonandtwodaughters.

Mother&FatherofMr.Mehta
|
|
Mr.&Mrs.Mehta
|
|
OneSon&TwoDaughters
WhenAlexandertheGreatattackedtheforcesofPorus,anIndiansoldier
wascapturedbytheGreeks.Hehaddisplayedsuchbraveryinbattle,
however,thattheenemyofferedtolethimchoosehowhewantedtobe
killed.Theytoldhim,"Ifyoutellalie,youwillputtothesword,andifyou
tellthetruthyouwillbehanged."

Thesoldiercouldmakeonlyonestatement.Hemadethatstatementand
wentfree.Whatdidhesay?

Answer

Thesoldiersaid,"Youwillputmetothesword."

ThesoldierhastosayaParadoxtosavehimself.Ifhisstatementistrue,he
willbehanged,whichisnottheswordandhencefalse.Ifhisstatementis
false,hewillbeputtothesword,whichwillmakeittrue.AParadox!!!

ApersonwantedtowithdrawXrupeesandYpaisefromthebank.But
cashiermadeamistakeandgavehimYrupeesandXpaise.Neitherthe
personnorthecashiernoticedthat.


Afterspending20paise,thepersoncountsthemoney.Andtohissurprise,
hehasdoubletheamounthewantedtowithdraw.

FindXandY.(1Rupee=100Paise)

Answer

Asgiven,thepersonwantedtowithdraw100X+Ypaise.

Buthegot100Y+Xpaise.

Afterspending20paise,hehasdoubletheamounthewantedto
withdraw.Hence,theequationis

2*(100X+Y)=100Y+X20

200X+2Y=100Y+X20

199X98Y=20

98Y199X=20

Now,wegotoneequationbutthereare2variables.Wehavetoapply
littlebitoflogicoverhere.WeknowthatifweinterchangeX&Y,
amountgetsdouble.SoYshouldbetwiceofXoronemorethantwiceof
Xi.e.Y=2XorY=2X+1

CaseI:
Y=2X
Solvingtwoequationssimultaneously
98Y199X=20
Y2X=0
WegetX=20/3&Y=40/2

CaseII:
Y=2X+1
Solvingtwoequationssimultaneously
98Y199X=20

Y2X=1
WegetX=26&Y=53

Now,itsobviousthathewantedtowithdrawRs.26.53

Submit

Answer

Users
Answer(2)

BrainVista
Answer

ThegameofTicTacToeisbeingplayedbetweentwoplayers.Onlythelast
marktobeplacedinthegameasshown.

Whowillwinthegame,OorX?Canyoutellwhichwasthesixthmarkand
atwhichposition?Doexplainyouranswer.
AttheParty:
1. Therewere9menandchildren.
2. Therewere2morewomenthanchildren.
3. Thenumberofdifferentmanwomancouplespossiblewas24.Note
thatiftherewere7menand5women,thentherewouldhavebeen35
manwomancouplespossible.

Also,ofthethreegroupsmen,womenandchildrenattheparty:
4. Therewere4ofonegroup.
5. Therewere6ofonegroup.
6. Therewere8ofonegroup.
Exactlyoneoftheabove6statementsisfalse.

Canyoutellwhichoneisfalse?Also,howmanymen,womenandchildren
arethereattheparty

Assumethatboththeplayersareintelligentenough.

Answer

Owillwinthegame.ThesixthmarkwasXinsquare9.

The7thmarkmustbeplacedinsquare5whichisthewinsituationforboth
XandO.Hence,the6thmarkmustbeplacedinalinealreadycontaining
twooftheopponentsmarks.Therearetwosuchpossibilitiesthe6thmark
wouldhavebeeneitherOinsquare7orXinsquare9.

Asweknowboththeplayersareintelligentenough,the6thmarkcouldnot
beOinsquare7.Instead,hewouldhaveplacedOinsquare5andwould
havewon.

Hence,thesixthmarkmustbeXplacedinsquare9.Andtheseventhmark
willbeO.ThusOwillwinthegame.

Answer

Statement(4)isfalse.Thereare3men,8womenand6children.

AssumethatStatements(4),(5)and(6)arealltrue.Then,Statement(1)is
false.ButthenStatement(2)and(3)bothcannotbetrue.Thus,
contradictorytothefactthatexactlyonestatementisfalse.

SoStatement(4)orStatement(5)orStatement(6)isfalse.Also,Statements
(1),(2)and(3)allaretrue.

From(1)and(2),thereare11menandwomen.Thenfrom(3),thereare2
possiblecaseseitherthereare8menand3womenorthereare3menand8
women.

Ifthereare8menand3women,thenthereis1child.ThenStatements(4)
and(5)botharefalse,whichisnotpossible.

Hence,thereare3men,8womenand6children.Statement(4)isfalse.

Thereisashortageoftubelights,bulbsandfansinavillageKharghar.Itis
foundthat

Allhousesdonothaveeithertubelightorbulborfan.
exactly19%ofhousesdonothavejustoneofthese.
atleast67%ofhousesdonothavetubelights.
atleast83%ofhousesdonothavebulbs.
atleast73%ofhousesdonothavefans.

Whatpercentageofhousesdonothavetubelight,bulbandfan?
Answer

42%housesdonothavetubelight,bulbandfan.

Let'sassumethatthereare100houses.Hence,thereshouldbetotal300
itemsi.e.100tubelights,100bulbsand100fans.

Fromthegivendata,weknowthatthereisshortageofatleast(67+83+73)
223itemsinevery100houses.

Also,exactly19housesdonothavejustoneitem.Itmeansthatremaining
81housesshouldaccountfortheshortageofremaining(22319)204items.
Ifthoseremaining81housesdonothave2itemseach,therewouldbea
shortageof162items.Buttotalof204itemsareshort.Hence,atleast
(204162)42housesdonothaveall3itemstubelight,bulbandfan.

Thus,42%housesdonothavetubelight,bulbandfan.
Mr.SubramaniamrentsaprivatecarforAndheriColabaAndheritrip.It
costshimRs.300everyday.

OnedaythecardriverinformedMr.Subramaniamthatthereweretwo
studentsfromBandrawhowishedtogofromBandratoColabaandbackto
Bandra.BandraishalfwaybetweenAndheriandColaba.Mr.Subramaniam
askedthedrivertoletthestudentstravelwithhim.

Onthefirstdaywhentheycame,Mr.Subramaniamsaid,"Ifyoutellmethe
mathematicallycorrectpriceyoushouldpayindividuallyforyourportionof
thetrip,Iwillletyoutravelforfree."

Howmuchshouldtheindividualstudentpayfortheirjourney?
Answer

TheindividualstudentshouldpayRs.50fortheirjourney.

Notethat3personsaretravellingbetweenBandraandColaba.

TheentiretripcostsRs.300toMr.Subramanian.Hence,halfofthetrip
costsRs.150.

ForAndheriBandraAndheri,onlyonepersoni.e.Mr.Subramaniamis
travelling.Hence,hewouldpayRs.150.

ForBandraColabaBandra,threepersonsi.eMr.Subramaniamandtwo
students,aretravelling.Hence,eachstudentwouldpayRs.50.

SubstitutedigitsfortheletterstomakethefollowingDivisiontrue
OUT

STEM|DEMISE

|DMOC

TUIS

STEM

ZZZE

ZUMM

IST
Notethattheleftmostlettercan'tbezeroinanyword.Also,theremustbea
onetoonemappingbetweendigitsandletters.e.g.ifyousubstitute3forthe

letterM,nootherlettercanbe3andallotherMinthepuzzlemustbe
Answer

C=0,U=1,S=2,T=3,O=4,M=5,I=6,Z=7,E=8,D=9

ItisobviousthatU=1(asU*STEM=STEM)andC=0(asIC=I).

S*OisasingledigitandalsoS*Tisasingledigit.Hence,theirvalues(O,S,
T)mustbe2,3or4(astheycannotbe0or1orgreaterthan4).

Consider,STEM*O=DMOC,whereC=0.ItmeansthatMmustbe5.Now,
itssimple.O=4,S=2,T=3,E=8,Z=7,I=6andD=9.
OUT413

STEM|DEMISE2385|985628

|DMOC|9540

TUIS3162

STEM2385

ZZZE7778

ZUMM7155

IST623
Also,whenarrangedfrom0to9,itspells
CUSTOMIZED
.
Atwhattimeafter4.00p.m.istheminuteshandofaclockexactlyaligned
withthehourhand?

Answer

4:21:49.5

AssumethatXminutesafter4.00PMminutehandexactlyalignswithand
hourhand.

Foreveryminute,minutehandtravels6degrees.
Hence,forXminutesitwilltravel6*Xdegrees.

Foreveryminute,hourhandtravels1/2degrees.
Hence,forXminutesitwilltravelX/2degrees.

At4.00PM,theanglebetweenminutehandandhourhandis120degrees.
Also,afterXminutes,minutehandandhourhandareexactlyaligned.So
theanglewithrespectto12i.e.VerticalPlanewillbesame.Therefore,

6*X=120+X/2
12*X=240+X
11*X=240
X=21.8182
X=21minutes49.5seconds

Hence,at4:21:49.5minutehandisexactlyalignedwiththehourhand.

Asoldierlooseshiswayinathickjungle.Atrandomhewalksfromhis
campbutmathematicallyinaninterestingfashion.

FirsthewalksonemileEastthenhalfmiletoNorth.Then1/4miletoWest,
then1/8miletoSouthandsoonmakingaloop.

Finallyhowfarheisfromhiscampandinwhichdirection?

Answer

Thesoldieris0.8944milesawayfromhiscamptowardsEastNorth.

ItisobviousthatheisinEastNorthdirection.

DistancetravelledinNorthandSouthdirections
=1/21/8+1/321/128+1/5121/2048+andsoon...(ageometricseries
withr=(1/4))

n
(1/2)*(1(1/4)
)
=
(1(1/4))

=1/(2*(1(1/4)))
=2/5

SimilarlyinEastandWestdirections
=11/4+1/161/64+1/256andsoon...(ageometricserieswithr=
(1/4))

n
(1)*(1(1/4)
)
=
(1(1/4))

=1/((1(1/4))
=4/5

Sothesoldieris4/5milesawaytowardsEastand2/5milesawaytowards
North.Sousingrightangledtriangle,soldieris0.8944milesawayfromhis
camp.

RajhasajewelchestcontainingRings,PinsandEarrings.Thechest
contains26pieces.Rajhas21/2timesasmanyringsaspins,andthe
numberofpairsofearringsis4lessthanthenumberofrings.

HowmanyearringsdoesRajhave?

Answer

12earrings

AssumethatthereareRrings,PpinsandEpairofearrings.

Itisgiventhat,hehas21/2timesasmanyringsaspins.

R=(5/2)*PorP=(2*R)/5

And,thenumberofpairsofearringsis4lessthanthenumberofrings.
E=R4orR=E+4

Also,therearetotal26pieces.
R+P+2*E=26
R+(2*R)/5+2*E=26
5*R+2*R+10*E=130
7*R+10*E=130
7*(E+4)+10*E=130
7*E+28+10*E=130
17*E=102
E=6

Hence,thereare6pairsofEarringsi.e.total12Earrings
Howmanywaysarethereofarrangingthesixteenblackorwhitepiecesofa
standardinternationalchesssetonthefirsttworowsoftheboard?

Giventhateachpawnisidenticalandeachrook,knightandbishopis
identicaltoitspair.
Submitted

Answer

6,48,64,800ways

16
Therearetotal16pieceswhichcanbearrangedon16placesin
P
=16!
16
ways.
(16!=16*15*14*13*12*.....*3*2*1)

But,therearesomeduplicatecombinationsbecauseofidenticalpieces.
8
Thereare8identicalpawn,whichcanbearrangedin
P
=8!ways.
8
Similarlythereare2identicalrooks,2identicalknightsand2
2
identicalbishops.Eachcanbearrangedin
P
=2!ways.
2
Hence,therequireansweris
=(16!)/(8!*2!*2!*2!)
=6,48,64,800

Apersonwithsomemoneyspends1/3forcloths,1/5oftheremainingfor
foodand1/4oftheremainingfortravel.HeisleftwithRs100/

Howmuchdidhehavewithhiminthebegining?

Answer

Rs.250/

AssumethatinitiallyhehadRs.X
Hespent1/3forcloths=.(1/3)*X
Remainingmoney=(2/3)*X

Hespent1/5ofremainingmoneyforfood=(1/5)*(2/3)*X=(2/15)*X
Remainingmoney=(2/3)*X(2/15)*X=(8/15)*X

Again,hespent1/4ofremainingmaoneyfortravel=(1/4)*(8/15)*X=
(2/15)*X
Remainingmoney=(8/15)*X(2/15)*X=(6/15)*X

ButafterspendingfortravelheisleftwithRs.100/So
(6/15)*X=100
X=250

Grassinlawngrowsequallythickandinauniformrate.Ittakes24daysfor
70cowsand60daysfor30cowstoeatthewholeofthegrass.

Howmanycowsareneededtoeatthegrassin96days?
Answer

20cows

ggrassatthebeginning
rrateatwhichgrassgrows,perday
yrateatwhichonecoweatsgrass,perday
nnoofcowstoeatthegrassin96days

Fromgivendata,
g+24*r=70*24*yA
g+60*r=30*60*yB
g+96*r=n*96*yC

Solvingfor(BA),
(60*r)(24*r)=(30*60*y)(70*24*y)
36*r=120*yD

Solvingfor(CB),
(96*r)(60*r)=(n*96*y)(30*60*y)
36*r=(n*9630*60)*y
120*y=(n*9630*60)*y[FromD]
120=(n*961800)
n=20

Hence,20cowsareneededtoeatthegrassin96days.
Thereisasafewitha5digitnumberasthekey.The4thdigitis4greater
thantheseconddigit,whilethe3rddigitis3lessthanthe2nddigit.The1st
digitisthricethelastdigit.Thereare3pairswhosesumis11.

Findthenumber.
Answer

65292

Aspergivenconditions,therearethreepossiblecombinationsfor2nd,3rd
and4thdigits.Theyare(3,0,7)or(4,1,8)or(5,2,9)

Itisgiventhatthereare3pairswhosesumis11.Allpossiblepairsare(2,
9),(3,8),(4,7),(5,6).Nowrequirednumberis5digitnumberandit
contains3pairsof11.Soitmustnotbehaving0and1init.Hence,theonly
possiblecombinationfor2nd,3rdand4thdigitsis(5,2,9)

Also,1stdigitisthricethelastdigit.Thepossiblecombinationsare(3,1),
(6,2)and(9,3),outofwhichonly(6,2)with(5,2,9)gives3pairsof11.
Hence,theansweris65292.
FourfriendsArjan,Bhuvan,GuranandLakhawerecomparingthenumber
ofsheepthattheyowned.


ItwasfoundthatGuranhadtenmoresheepthanLakha.

IfArjangaveonethirdtoBhuvan,andBhuvangaveaquarterofwhathe
thenheldtoGuran,whothenpassedonafifthofhisholdingtoLakha,they
wouldallhaveanequalnumberofsheep.

Howmanysheepdideachofthempossess?Givetheminimalpossible
answer

Answer

Arjan,Bhuvan,GuranandLakhahad90,50,55and45sheep
respectively.

AssumethatArjan,Bhuvan,GuranandLakhahadA,B,GandLsheep
respectively.Asitisgiventhatattheendeachwouldhaveanequalnumber
ofsheep,comparingthefinalnumbersfromtheabovetable.

Arjan'ssheep=Bhuvan'ssheep
2A/3=A/4+3B/4
8A=3A+9B
5A=9B

Arjan'ssheep=Guran'ssheep
2A/3=A/15+B/5+4G/5
2A/3=A/15+A/9+4G/5(asB=5A/9)
30A=3A+5A+36G

22A=36G
11A=18G

Arjan'ssheep=Lakha'ssheep
2A/3=A/60+B/20+G/5+L
2A/3=A/60+A/36+11A/90+L(asB=5A/9andG=11A/18)
2A/3=A/6+L
A/2=L
A=2L

Also,itisgiventhatGuranhadtenmoresheepthanLakha.
G=L+10
11A/18=A/2+10
A/9=10
A=90sheep

Thus,Arjanhad90sheep,Bhuvanhad5A/9i.e.50sheep,Guranhad
11A/18i.e.55sheepandLakhahadA/2i.e.45sheep.
Consideranumber235,wherelastdigitisthesumoffirsttwodigitsi.e.2+
3=5.

Howmanysuch3digitnumbersarethere?

Answer

Thereare45different3digitnumbers.

Thelastdigitcannotbe0.

Ifthelastdigitis1,theonlypossiblenumberis101.(Notethat011isnota
3digitnumber)

Ifthelastdigitis2,thepossiblenumbersare202and112.

Ifthelastdigitis3,thepossiblenumbersare303,213and123.

Ifthelastdigitis4,thepossiblenumbersare404,314,224and134.

Ifthelastdigitis5,thepossiblenumbersare505,415,325,235and145.


NotethepatternhereIfthelastdigitis1,thereisonlyonenumber.Ifthe
lastdigitis2,therearetwonumbers.Ifthelastdigitis3,therearethree
numbers.Ifthelastdigitis4,therearefournumbers.Ifthelastdigitis5,
therearefivenumbers.Andsoon.....

Thus,totalnumbersare
1+2+3+4+5+6+7+8+9=45

Altogetherthen,thereare45different3digitnumbers,wherelastdigitisthe
sumoffirsttwodigits.

Findthesmallestnumbersuchthatifitsrightmostdigitisplacedatitsleft
end,thenewnumbersoformedisprecisely50%largerthantheoriginal
number.
Answer

Theansweris
285714
.

Ifitsrightmostdigitisplacedatitsleftend,thennewnumberis428571
whichis50%largerthantheoriginalnumber285714.

Thesimplestwayistowriteasmallprogram.Andtheotherwayistrialand
error!!!
TwoidenticalpackofcardsAandBareshuffledthroughly.Onecardis
pickedfromAandshuffledwithB.ThetopcardfrompackAisturnedup.
IfthisistheQueenofHearts,whatarethechancesthatthetopcardinBwill
betheKingofHearts?

Answer

52/2703

Therearetwocasestobeconsidered.

CASE1:
KingofHeartsisdrawnfromPackAandshuffledwithPackB

ProbabilityofdrawingKingofHeartsfromPackA=1/51(asQueenof
Heartsisnottobedrawn)

ProbabilityofhavingKingofHeartsonthetopofthePackB=2/53

Sototalprobabilityofcase1=(1/51)*(2/53)=2/(51*53)

CASE2:
KingofHeartsisnotdrawnfromPackA

ProbabilityofnotdrawingKingofHeartsfromPackA=50/51(asQueenof
Heartsisnottobedrawn)
ProbabilityofhavingKingofHeartsonthetopofthePackB=1/53

Sototalprobabilityofcase2=(50/51)*(1/53)=50/(51*53)

Nowaddingboththeprobability,therequiredprobabilityis
=2/(51*53)+50/(51*53)
=52/(51*53)
=52/2703
=0.0192378
Thereare3antsat3cornersofatriangle,theyrandomlystartmoving
towardsanothercorner.

Whatistheprobabilitythattheydon'tcollide?

Answer

Let'smarkthecornersofthetriangleasA,B,C.Therearetotal8waysin
whichantscanmove.
1. A>B,B>C,C>A
2. A>B,B>C,C>B
3. A>B,B>A,C>A
4. A>B,B>A,C>B
5. A>C,C>B,B>A
6. A>C,C>B,B>C
7. A>C,C>A,B>A
8. A>C,C>A,B>C

Outofwhich,thereareonlytwocasesunderwhichtheantswon'tcollide:
A>B,B>C,C>A
A>C,C>B,B>A


Findallsetsofconsecutiveintegersthataddupto1000.
Submittedby:JamesBarberousse

Answer

Therearetotal8suchseries:
1. Sumof2000numbersstartingfrom999i.e.summationof
numbersfrom999to1000.
(999)+(998)+(997)+.....+(1)+0+1+2+.....+997+998
+999+1000=1000
2. Sumof400numbersstartingfrom197i.e.summationofnumbers
from197to202.
(197)+(196)+(195)+.....+(1)+0+1+2+.....+199+200
+201+202=1000
3. Sumof125numbersstartingfrom54i.e.summationofnumbers
from54to70.
(54)+(53)+(52)+.....+(1)+0+1+2+.....+68+69+70=
1000
4. Sumof80numbersstartingfrom27i.e.summationofnumbers
from27to52.
(27)+(26)+(25)+.....+(1)+0+1+2+.....+50+51+52=
1000
5. Sumof25numbersstartingfrom28i.e.summationofnumbers
from28to52.
28+29+30+31+32+33+34+35+36+37+38+39+40+
41+42+43+44+45+46+47+48+49+50+51+52=1000
6. Sumof16numbersstartingfrom55i.e.summationofnumbers
from55to70.
55+56+57+58+59+60+61+62+63+64+65+66+67+
68+69+70=1000
7. Sumof5numbersstartingfrom198i.e.summationofnumbers
from198to202.
198+199+200+201+202=1000
8. Sumof1numberstartingfrom1000.
1000=1000

Thereisa4charactercode,with2ofthembeinglettersandtheother2
beingnumbers.

Howmanymaximumattemptswouldbenecessarytofindthecorrectcode?
Notethatthecodeiscasesensitive.
Answer

Themaximumnumberofattemptsrequiredare16,22,400

Thereare52possiblelettersatozandAtoZ,and10possiblenumbers0
to9.Now,4characters2lettersand2numbers,canbeselectedin
4
52*52*10*10ways.These4characterscanbearrangedin
C
i.e.6different
2
waysthenumberofuniquepatternsthatcanbeformedbyliningup4
objectsofwhich2aredistinguishedoneway(i.e.theymustbeletters)and
theother2aredistinguishedanotherway(i.e.theymustbenumbers).

Consideranexample:Let'sassumethat@representsletterand#represents
number.the6possiblewaysofarrangingthemare:@@##,@#@#,@##@,
#@@#,#@#@,##@@

Hence,therequiredansweris
=52*52*10*10*6
=16,22,400attempts
=1.6millionapprox.

ThankstoTimSandersforopeningBrainVista'sbrain!!!
Howmanypossiblecombinationsarethereina3x3x3rubicscube?

Inotherwords,ifyouwantedtosolvetherubicscubebytryingdifferent
combinations,howmanymightittakeyou(worstcasesenerio)?

Howmanyfora4x4x4cube?
Submitted

Answer

Thereare4.3252*10^19possiblecombinationsfor3x3x3Rubicsand

7.4012*10^45possiblecombinationsfor4x4x4Rubics.

Let'sconsider3x3x3Rubicsfirst.

Thereare8cornercubes,whichcanbearrangedin8!ways.
Eachofthese8cubescanbeturnedin3differentdirections,sothereare3^8
orientationsaltogether.Butifyougetallbutoneofthecornercubeinto
chosenpositionsandorientations,onlyoneof3orientationsofthefinal
cornercubeispossible.Thus,totalwayscornercubescanbeplaced=(8!)*
(3^8)/8=(8!)*(3^7)

Similarly,12edgecubescanbearrangedin12!ways.
Eachofthese12cubescanbeturnedin2differentdirections,sothereare
2^12orientationsaltogether.Butifyougetallbutoneoftheedgecubeinto
chosenpositionsandorientations,onlyoneof2orientationsofthefinaledge
cubeispossible.Thus,totalwaysedgecubescanbeplaced=(12!)*
(2^12)/2=(12!)*(2^11)

Here,wehaveessentiallypulledthecubesapartandstuckcubesbackin
placewhereverweplease.Inreality,wecanonlymovecubesaroundby
turningthefacesofthecubes.Itturnsoutthatyoucan'tturnthefacesin
suchawayastoswitchthepositionsoftwocubeswhilereturningallthe
otherstotheiroriginalpositions.Thusifyougetallbuttwocubesinplace,
thereisonlyoneattainablechoiceforthem(not2!).Hence,wemustdivide
by2.

Totaldifferentpossiblecombinationsare
=[(8!)*(3^7)]*[(12!)*(2^11)]/2
=(8!)*(3^7)*(12!)*(2^10)
=4.3252*10^19

Similarly,for4x4x4Rubicstotaldifferentpossiblecombinationsare
=[(8!)*(3^7)]*[(24!)]*[(24!)/(4!^6)]/24
=7.4011968*10^45

Notethatthereare24edgecubes,whichyoucannotturnin2orientations
(henceno2^24/2).Also,thereare4centercubesperfacei.e.(24!)/(4!^6).

Youcanswitch2cubeswithoutaffectingtherestofthecombinationas
4*4*4hasevendimensions(hencenodivisionby2).Butpatternononeside
isrotatedin4directionsover6faces,hencedivideby24.
Substitutedigitsfortheletterstomakethefollowingrelationtrue.
NEVER

LEAVE

+ME

ALONE
Notethattheleftmostlettercan'tbezeroinanyword.Also,theremustbea
onetoonemappingbetweendigitsandletters.e.g.ifyousubstitute3forthe
letterM,nootherlettercanbe3andallotherMinthepuzzlemustbe3.

Answer

Atoughone!!!

SinceR+E+E=10+E,itisclearthatR+E=10andneitherRnorEis
equalto0or5.Thisistheonlyentrypointto

solveit.Nowusetrialnerrormethod.

NEVER21419

LEAVE31541

+ME+61

ALONE53021

OneofthefourpeopleMr.Clinton,hiswifeMonika,theirsonMandyand
theirdaughterCindyisasingerandanotherisadancer.Mr.Clintonis

olderthanhiswifeandMadyisolderthanhissister.
1. Ifthesingerandthedancerarethesamesex,thenthedancerisolder
thanthesinger.
2. Ifneitherthesingernorthedanceristheparentoftheother,thenthe
singerisolderthanthedancer.
3. Ifthesingerisaman,thenthesingerandthedancerarethesameage.
4. Ifthesingerandthedancerareofoppositesexthenthemanisolder
thanthewoman.
5. Ifthedancerisawoman,thenthedancerisolderthanthesinger.
Whoseoccupationdoyouknow?Andwhatishis/heroccupation?
Answer

CindyistheSinger.Mr.ClintonorMonikaistheDancer.

From(1)and(3),thesingerandthedancer,bothcannotbeaman.From(3)
and(4),ifthesingerisaman,thenthedancermustbeaman.Hence,the
singermustbeawoman.

CASEI
:SingerisawomanandDancerisalsoawoman
Then,thedancerisMonikaandthesingerisCindy.

CASEII
:SingerisawomanandDancerisalsoaman
Then,thedancerisMr.ClintonandthesingerisCindy.

Inboththecases,weknowthatCindyistheSinger.AndeitherMr.Clinton
orMonikaistheDancer.
Thereare20peopleinyourapplicantpool,including5pairsofidentical
twins.

Ifyouhire5peoplerandomly,whatarethechancesyouwillhireatleast1
pairofidenticaltwins?(Needlesstosay,thiscouldcausetrouble))
Submitted

Answer

Theprobabilitytohire5peoplewithatleast1pairofidenticaltwinsis
25.28%

5peoplefromthe20peoplecanbehiredin20C5=15504ways.

Now,divide20peopleintotwogroupsof10peopleeach:
G1withalltwins
G2withallpeopleotherthantwins

Let'sfindoutallpossiblewaystohire5peoplewithoutasinglepairof
indenticaltwins.
People
People
NoofwaystohireG1
Noof
Total
fromG1 fromG2
withoutasinglepairof
waysto
ways
indenticaltwins
hireG2
0

10C0

10C5

252

10C1

10C4

2100

10C2*8/9

10C3

4800

10C3*8/9*6/8

10C2

3600

10C4*8/9*6/8*4/7

10C1

800

10C5*8/9*6/8*4/7*2/6 10C0

32

Total

11584

Thus,totalpossiblewaystohire5peoplewithoutasinglepairofindentical
twins=11584ways

So,totalpossiblewaystohire5peoplewithatleastasinglepairof
indenticaltwins=1550411584=3920ways

Hence,theprobabilitytohire5peoplewithatleastasinglepairof
indenticaltwins
=3920/15504
=245/969
=0.2528
=25.28%
Inahotel,roomsarenumberedfrom101to550.Aroomischosenat
random.Whatistheprobabilitythatroomnumberstartswith1,2or3and
endswith4,5or6?

Answer


Therearetotal450rooms.

Outofwhich299roomnumberstartswitheither1,2or3.(asroomnumber
100isnotthere)Nowoutofthose299roomsonly90roomnumbersend
with4,5or6

Sotheprobabilityis90/450i.e.1/5or0.20
Draw9dotsonapage,intheshapeofthreerowsofthreedotstoforma
square.Nowplaceyourpenonthepage,draw4straightlinesandtryand
coverallthedots.

You'renotallowedtoliftyourpen.

Note:Don'tbeconfinedbythedimensionsofthesquare.
Submitted

Thereare3personsX,YandZ.Onsomeday,XlenttractorstoYandZas
manyastheyhad.AfteramonthYgaveasmanytractorstoXandZas
manyastheyhave.AfteramonthZdidthesamething.Attheendofthis
transactioneachoneofthemhad24.

Findthetractorseachoriginallyhad?
Answer

Onewaytosolveitisbymaking3equationsandsolvethem
simultaneously.ButthereisrathereasierwaytosolveitusingBacktracing.

It'sgiventhatattheend,eachhad24tractors(24,24,24)i.e.afterZgave
tractorstoX&Yasmanyastheyhad.Itmeansthataftergettingtractors
fromZtheirtractorsgotdoubled.SobeforeZgavethemtractors,theyhad
12tractorseachandZhad48tractors.(12,12,48)

Similarly,beforeYgavetractorstoX&Z,theyhad6&24tractors
respectivelyandYhad42tractorsi.e.(6,42,24)


Again,beforeXgavetractorstoY&Z,theyhad21&12tractors
respectivelyandXhad39tractorsi.e.(39,21,12)

Hence,initially
Xhad39tractors,Yhad21tractors
and
Zhad12
tractors.

Thereisa50mlongarmyplatoonmarchingahead.Thelastpersoninthe
platoonwantstogivealettertothefirstpersonleadingtheplatoon.Sowhile
theplatoonismarchingherunsahead,reachesthefirstpersonandhands
overthelettertohimandwithoutstoppingherunsandcomesbacktohis
originalposition.

Inthemeantimethewholeplatoonhasmovedaheadby50m.

Thequestionishowmuchdistancedidthelastpersoncoverinthattime.
Assumingthatheranthewholedistancewithuniformspeed.
Submitted

Answer

Thelastpersoncovered120.71meters.

Itisgiventhattheplatoonandthelastpersonmovedwithuniformspeed.
Also,theybothmovedfortheidenticalamountoftime.Hence,theratioof
thedistancetheycoveredwhilepersonmovingforwardandbackwordare
equal.

Let'sassumethatwhenthelastpersonreachedthefirstperson,theplatoon
movedXmetersforward.

Thus,whilemovingforwardthelastpersonmoved(50+X)meterswhereas
theplatoonmovedXmeters.

Similarly,whilemovingbackthelastpersonmoved[50(50X)]Xmeters
whereastheplatoonmoved(50X)meters.

Now,astheratiosareequal,
(50+X)/X=X/(50X)

(50+X)*(50X)=X*X

Solving,X=35.355meters

Thus,totaldistancecoveredbythelastperson
=(50+X)+X
=2*X+50
=2*(35.355)+50
=120.71meters

Notethatatfirstglance,onemightthinkthatthetotaldistancecoveredby
thelastpersonis100meters,asheranthetotallenghtoftheplatoon(50
meters)twice.TRUE,butthat'stherelativedistancecoveredbythelast
personi.e.assumingthattheplatoonisstationary.
Assumethatyouhaveenoughcoinsof1,5,10,25and50cents.

Howmanywaysaretheretomakechangeforadollar?Doexplainyour
answer.

Thereare292waystomakechangeforadollarusingcoinsof1,5,10,
25and50cents.

Let'sgeneralisedtheteaserandmakeatableasshownabove.

Ifyouwishtomakechangefor75centsusingonly1,5,10and25cent
coins,gotothe.25rowandthe75columntoobtain121waystodothis.

Thetablecanbecreatedfromlefttorightandtoptobottom.Startwiththe
toplefti.e.1centrow.Thereisexactlyonewaytomakechangeforevery
amount.Thencalculatethe5centsrowbyaddingthenumberofwaysto
makechangefortheamountusing1centcoinsplusthenumberofwaysto
makechangefor5centslessusing1and5centcoins.

Let'stakeanexample:
Togetchangefor50centsusing1,5and10centcoins.
*50centschangeusing1and5centcoins=11ways
*(5010)40centschangeusing1,5and10centcoins=25ways
*50centschangeusing1,5and10centcoins=11+25=36ways

Let'stakeanotherexample:
Togetchangefor75centsusingallcoinsupto50centi.e.1,5,10,25and
50centscoins.
*75centschangeusingcoinsupto25cent=121ways
*(7550)25centschangeusingcoinsupto50cent=13ways
*75centschangeusingcoinsupto50cent=121+13=134ways

Forpeoplewhodon'twanttoteasetheirbrainandlovetodocomputer
programming,thereisasimpleway.Writeasmallmultiloopprogramto
solvetheequation:A+5B+10C+25D+50E=100
where,
A=0to100
B=0to20
C=0to10
D=0to4
E=0to2

TheprogramshouldoutputallthepossiblevaluesofA,B,C,DandEfor
whichtheequationissatisfied.

InaRoadRace,oneofthethreebikerswasdoing15kmlessthanthefirst
and3kmmorethanthethird.Healsofinishedtherace12minutesafterthe
firstand3minutesbeforethethird.

Canyoufindoutthespeedofeachbiker,thetimetakenbyeachbikerto
finishtheraceandthelengthofthecourse?

Assumethattherewerenostopsintheraceandalsotheyweredrivingwith
constantspeedsthroughoutthe

Answer

Letusassumethat
SpeedofFirstbiker=V1km/min
SpeedofSecondbiker=V2km/min
SpeedofThirdbiker=V3km/min
Totaltimetakebyfirstbiker=T1min
Totaldistance=Skm

Nowasperthedatagivenintheteaser,atatimeTmin
X1=V1*T>1

X115=V2*T>2

X118=V3*T>3

AtaDistanceSKm.
S=V1*T1>4

S=V2*(T1+12)>5

S=V3*(T1+15)>6

Thusthereare6equationsand7unknowndatathatmeansithasinfinite
numberofsolutions.

Bysolvingabove6equationsweget,

Timetakenbyfirstbiker,T1=60Min.
TimetakenbySecondbiker,T2=72Min.
Timetakenbyfirstbiker,T3=75Min.

Also,weget
Speedoffirstbiker,V1=90/Tkm/min
Speedofsecondbiker,V2=(5/6)V1=75/Tkm/min
Speedofthirdbiker,V3=(4/5)V1=72/Tkm/min

Also,thelengthofthecourse,S=5400/Tkm

Thus,forthedatagiven,onlythetimetakenbyeachbikercanbefoundi.e.
60,72and75minutes.Forotherquantities,onemoreindependentdatumis
requiredi.e.eitherTorV1orV2orV3

ThankstoTheerthamSrinivasfortheanswer!!!
Whatisthefourdigitnumberinwhichthefirstdigitis1/3ofthesecond,the
thirdisthesumofthefirstandsecond,andthelastisthreetimesthe
second?

Answer

The4digitnumberis1349.

Itisgiventhatthefirstdigitis1/3ofthesecond.Thereare3such
possibilities.
1. 1and3
2. 2and6
3. 3and9
Now,thethirddigitisthesumofthefirstandseconddigits.
1. 1+3=4
2. 2+6=8
3. 3+9=12
Itisclearthatoption3isnotpossible.Soweareleftwithonlytwooptions.
Also,thelastdigitisthreetimesthesecond,whichrulesoutthesecond
option.Hence,theansweris1349.

DifferencebetweenBholu'sandMolu'sageis2yearsandthedifference

betweenMolu'sandKolu'sageis5years.

Whatisthemaximumpossiblevalueofthesumofthedifferenceintheir
ages,takentwoatatime?
Answer

Themaximumpossiblevalueofthesumofthedifferenceintheirages
takentwoatatimeis14years.

Itisgiventhat
"DifferencebetweenBholu'sandMolu'sageis2years"
"DifferencebetweenMolu'sandKolu'sageis5years"

Now,togetthemaximumpossiblevalue,thedifferencebetweenBholu's
andKolu'sageshouldbemaximumi.e.Molu'sageshouldbeinbetween
Bholu'sandKolu'sage.Then,thedifferencebetweenBholu'sandKolu'sage
is7years.

Hence,themaximumpossiblevalueofthesumofthedifferenceintheir
agestakentwoatatimeis(2+5+7)14years.
Ifitisgiventhat:
252=3
100x2=20
36/3=2

Whatis1443=?
Submitted
Answer

Thereare3possibleanswerstoit.

Answer1:9
Simplyreplacethefirstnumberbyitssquareroot.
(25)52=3
(100)10x2=20
(36)6/3=2
(144)123=9

Answer2:11

Dropthedigitinthetenspositionfromthefirstnumber.
(2)52=3
1(0)0x2=20
(3)6/3=2
1(4)43=11

Youwillgetthesameansweronremovingleftandrightdigitalternatively
fromthefirstnumberi.eremoveleftdigitfromfirst(2),rightdigitfrom
second(0),leftdigitfromthird(3)andrightdigitfromforth(4).
(2)52=3
10(0)x2=20
(3)6/3=2
14(4)3=11

Answer3:14
Dropleftandrightdigitalternativelyfromtheactualanswer.
252=(2)3(dropleftdigiti.e.2)
100*2=20(0)(droprightdigiti.e.0)
36/3=(1)2(dropleftdigiti.e.1)
1443=14(1)(droprightdigiti.e.1)

A3digitnumberissuchthatit'sunitdigitisequaltotheproductoftheother
twodigitswhichareprime.Also,thedifferencebetweenit'sreverseand
itselfis396.

Whatisthesumofthethreedigits?

Answer

Therequirednumberis236andthesumis11.

Itisgiventhatthefirsttwodigitsoftherequirednumberareprimenumbers
i.e.2,3,5or7.Notethat1isneitherprimenorcomposite.Also,thethird
digitisthemultiplicationofthefirsttwodigits.Thus,firsttwodigitsmust
beeither2or3i.e.22,23,32or33whichmeansthattherearefourpossible
numbers224,236,326and339.

Now,itisalsogiventhatthedifferencebetweenit'sreverseanditselfis

396.Only236satisfiesthiscondition.Hence,thesumofthethreedigitsis
11.

Thereare4mugsplacedupturnedonthetable.Eachmughavethesame
numberofmarblesandastatementaboutthenumberofmarblesinit.The
statementsare:TwoorThree,OneorFour,ThreeorOne,OneorTwo.

Onlyoneofthestatementiscorrect.Howmanymarblesarethereunder
eachmug?

Answer

Asimpleone.

Asitisgiventhatonlyoneofthefourstatementiscorrect,thecorrect
numbercannotappearinmorethanonestatement.Ifitappearsinmorethan
onestatement,thenmorethanonestatementwillbecorrect.

Hence,thereare4marblesundereachmug.
AtUniversityofProbability,thereare375freshmen,293sophomores,187
juniors,&126seniors.Onestudentwillrandomlybechosentoreceivean
award.

Whatpercentchanceistherethatitwillbeajunior?Roundtothenearest
wholepercent
Answer

19%

Thispuzzleiseasy.Dividethenumberofjuniors(187)bythetotalnumber
ofstudents(981),&thenmultiplythenumberby100toconverttoa
percentage.

Hencetheansweris(187/981)*100=19%

Ifyouweretodialany7digitsonatelephoneinrandomorder,whatisthe
probabilitythatyouwilldialyourownphonenumber?

Assumethatyourtelephonenumberis7digits.
Answer

1in10,000,000

Thereare10digitsi.e.09.Firstdigitcanbedialedin10ways.Seconddigit
canbedialedin10ways.Thirddigitcanbedialedin10ways.Andsoon.....

Thus,7digitcanbedialedin10*10*10*10*10*10*10(=10,000,000)ways.
And,youhavejustonetelephonenumber.Hence,thepossibilitythatyou
willdialyourownnumberis1in10,000,000.

Notethat0123456maynotbeavalid7digittelephonenumber.Butwhile
dialinginrandomorder,thatisoneofthepossible7digitnumberwhichyou
maydial.
Ananthropologistdiscoversanisolatedtribewhosewrittenalphabet
containsonlysixletters(callthelettersA,B,C,D,EandF).Thetribehasa
tabooagainstusingthesamelettertwiceinthesameword.It'sneverdone.

Ifeachdifferentsequenceoflettersconstituesadifferentwordinthe
language,whatisthemaximumnumberofsixletterwordsthatthelanguage
canemploy?
Submitted

Answer

Thelanguagecanemploymaximumof720sixletterwords.

Itisasimplepermutationproblemofarranging6letterstogetdifferent
sixletterwords.Anditcanbedoneinin6!waysi.e.720ways.

Inotherwords,thefirstlettercanbeanyofthegiven6letters(AthroughF).
Then,whateverthefirstletteris,thesecondletterwillalwaysbefromthe
remaining5letters(assamelettercannotbeusedtwice),andthethirdletter
alwaysbefromtheremaining4letters,andsoon.Thus,thedifferent
possiblesixletterwordsare6*5*4*3*2*1=720

Kate,Demi,Madona,Sharon,BritneyandNicoledecidedtolunchtogether
inarestaurant.Thewaiterledthemtoaroundtablewithsixchairs.


Howmanydifferentwayscantheyseat?

Answer

Thereare120differentpossibleseatingarrangments.

NotethatonaroundtableABCDEFandBCDEFAisthesame.

Thefirstpersoncansitonanyoneoftheseats.Now,forthesecondperson
thereare5options,forthethirdpersonthereare4options,fortheforth
personthereare3options,forthefifthpersonthereare2optionsandforthe
lastpersonthereisjustoneoption.

Thus,totaldifferentpossibleseatingarrangementsare
=5*4*3*2*1
=120
3blocksarechosenrandomlyonachessboard.Whatistheprobabilitythat
theyareinthesamediagonal?

Answer

Therearetotalof64blocksonachessboard.So3blockscanbechosen
64
outof64in
C
ways.
3
Sothesamplespaceis=41664

Thereare2diagonalonchessboardeachonehaving8blocks.Consider
oneofthem.
8
3blocksoutof8blocksindiagonalcanbechosenin
C
ways.
3
8
Butthereare2suchdiagonals,hencefavourables=2*
C
=2*56=112
3

Therequireprobabilityis
=112/41664
=1/372
=0.002688

WhatistheareaofthetriangleABCwithA(e,p)B(2e,3p)andC(3e,5p)?

wherep=PI(3.141592654)

Answer

AtrickyONE.

Given3pointsarecolinear.Hence,itisastraightline.

Henceareaoftriangleis0.

SiluandMeenuwerewalkingontheroad.

Silusaid,"Iweigh51Kgs.Howmuchdoyouweigh?"

Meenurepliedthatshewouldn'trevealherweightdirectlyassheis
overweight.Butshesaid,"Iweigh29Kgsplushalfofmyweight."

HowmuchdoesMeenuweigh?

Answer

Meenuweighs58Kgs.

ItisgiventhatMeenuweighs29Kgsplushalfofherownweight.Itmeans
that29Kgsistheotherhalf.Sosheweighs58Kgs.

Solvingmathematically,let'sassumethatherweightisXKgs.
X=29+X/2
2*X=58+X
X=58Kgs
Considerthesum:ABC+DEF+GHI=JJJ

Ifdifferentlettersrepresentdifferentdigits,andtherearenoleadingzeros,

whatdoesJrepresent?

Answer

ThevalueofJmustbe9.

Sincetherearenoleadingzeros,Jmustbe7,8,or9.(JJJ=ABC+DEF+
GHI=14?+25?+36?=7??)

Now,theremainderleftafterdividinganynumberby9isthesameasthe
remainderleftafterdividingthesumofthedigitsofthatnumberby9.Also,
notethat0+1+...+9hasaremainderof0afterdividingby9andJJJhasa
remainderof0,3,or6.

Thenumber9istheonlynumberfrom7,8and9thatleavesaremainderof
0,3,or6ifyouremoveitfromthesum0+1+...+9.Hence,itfollowsthat
Jmustbe9.
AmanhasTenHorsesandninestablesasshownhere.
[][][][][][][][][]
ThemanwantstofitTenHorsesintoninestables.HowcanhefitTen
horsesintoninestables?
Submitted
Answer

Theanswerissimple.Itsaysthemanwantstofit"TenHorses"intonine
stables.Thereareninelettersinthephrase"TenHorses".Soyoucanputone
lettereachinallninestables.
[T][E][N][H][O][R][S][E][S]

Amanisatariverwitha9gallonbucketanda4gallonbucket.Heneeds
exactly6gallonsofwater.

Howcanheusebothbucketstogetexactly6gallonsofwater?

Notethathecannotestimatebydumpingsomeofthewateroutofthe9
gallonbucketorthe4gallonbucket
Answer

Forthesackofexplanation,let'sidentify4gallonbucketasBucketPand9
gallonbucketasBucketQ.

4gallonbucket 9gallonbucket
Operation
(BucketP)
(BucketQ)
Initially

FillthebucketQwith9gallonwater

Pour4gallonwaterfrombucketQto
bucketP

EmptybucketP

Pour4gallonwaterfrombucketQto
bucketP

EmptybucketP

Pour1gallonwaterfrombucketQto
bucketP

FillthebucketQwith9gallonwater

Pour3gallonwaterfrombucketQto
bucketP

9gallonbucketcontains6gallonofwater,asrequired.

EachofthefivecharactersinthewordBRAINhasadifferentvaluebetween
0and9.Usingthegivengrid,canyoufindoutthevalueofeachcharacter?
BRAIN31

BBRBA31

NIABB32

NIBAI30

IRAAA23

3729252729
Thenumbersontheextremerightrepresentthesumofthevalues

representedbythecharactersinthatrow.Also,thenumbersonthelastraw
representthesumofthevaluesrepresentedbythecharactersinthatcolumn.
e.g.B+R+A+I+N=31(fromfirstrow)

Answer

B=7,R=6,A=4,I=5andN=9

Maketotal10equations5forrowsand5forcolumnsandsovlethem.

FromRow3andRow4,
N+I+A+B+B=N+I+B+A+I+2
B=I+2

FromRow1andRow3,
B+R+A+I+N=N+I+A+B+B1
R=B1

FromColumn2,
R+B+I+I+R=29
B+2R+2I=29
B+2(B1)+2I=29
3B+2I=31
3(I+2)+2I=31
5I=25
I=5

Hence,B=7andR=6

FromRow2,
B+B+R+B+A=31
3B+R+A=31
3(7)+6+A=31
A=4

FromRow1,
B+R+A+I+N=31
7+6+4+5+N=31

N=9

Thus,B=7,R=6,A=4,I=5andN=9

Submit

Answer

Users
Answer(24)

BrainV

Thereare9coins.Outofwhichoneisoddonei.eweightislessormore.
Howmanyiterationsofweighingarerequiredtofindoddcoin?
Answer

Itisalwayspossibletofindoddcoinin3weighingsandtotellwhetherthe
oddcoinisheavierorlighter.
1. Take8coinsandweigh4against4.
Ifbotharenotequal,gotostep2
Ifbothareequal,gotostep3

2.

Oneofthese8coinsistheoddone.Namethecoinsonheaviersideof
thescaleasH1,H2,H3andH4.Similarly,namethecoinsonthe
lightersideofthescaleasL1,L2,L3andL4.EitheroneofH'sis
heavieroroneofL'sislighter.Weigh(H1,H2,L1)against(H3,H4,
X)whereXisonecoinremaininginintialweighing.
Ifbothareequal,oneofL2,L3,L4islighter.WeighL2against
L3.
Ifbothareequal,L4istheoddcoinandislighter.
IfL2islight,L2istheoddcoinandislighter.
IfL3islight,L3istheoddcoinandislighter.

If(H1,H2,L1)isheaviersideonthescale,eitherH1orH2is
heavier.WeightH1againstH2
Ifbothareequal,thereissomeerror.
IfH1isheavy,H1istheoddcoinandisheavier.
IfH2isheavy,H2istheoddcoinandisheavier.

If(H3,H4,X)isheaviersideonthescale,eitherH3orH4is
heavierorL1islighter.WeightH3againstH4
Ifbothareequal,L1istheoddcoinandislighter.
IfH3isheavy,H3istheoddcoinandisheavier.
IfH4isheavy,H4istheoddcoinandisheavier.

3.

TheremainingcoinXistheoddone.WeighXagainsttheanyone
coinusedininitialweighing.
Ifbothareequal,thereissomeerror.
IfXisheavy,Xistheoddcoinandisheavier.
IfXislight,Xistheoddcoinandislighter.

Inasportscontestthereweremmedalsawardedonnsuccessivedays(n>
1).
1. Onthefirstday1medaland1/7oftheremainingm1medalswere
awarded.
2. Onthesecondday2medalsand1/7ofthenowremainingmedalswas
awardedandsoon.
th
3. Onthen
andlastday,theremainingnmedalswereawarded.
Howmanydaysdidthecontestlast,andhowmanymedalswereawarded
altogether?
Answer

Total36medalswereawardedandthecontestwasfor6days.

Onday1:Medalsawarded=(1+35/7)=6:Remaining30medals
Onday2:Medalsawarded=(2+28/7)=6:Remaining24medals
Onday3:Medalsawarded=(3+21/7)=6:Remaining18medals
Onday4:Medalsawarded=(4+14/7)=6:Remaining12medals
Onday5:Medalsawarded=(5+7/7)=6:Remaining6medals
Onday6:Medalsawarded6

Igotthisanswerbywritingsmallprogram.Ifanyoneknowanyother
simplermethod,dosubmitit.

Anumberof9digitshasthefollowingproperties:

Thenumbercomprisingtheleftmosttwodigitsisdivisibleby2,that
comprisingtheleftmostthreedigitsisdivisibleby3,theleftmostfour
by4,theleftmostfiveby5,andsoonfortheninedigitsofthe
numberi.e.thenumberformedfromthefirstndigitsisdivisiblebyn,
2<=n<=9.
Eachdigitinthenumberisdifferenti.e.nodigitsarerepeated.
Thedigit0doesnotoccurinthenumberi.e.itiscomprisedonlyof
thedigits19insomeorder.

Findthenumber.

Answer

Theansweris
381654729

OnewaytosolveitisTrial&Error.Youcanmakeitbiteasierasodd
positionswillalwaysoccupyODDnumbersandevenpositionswillalways
occupyEVENnumbers.Further5thpositionwillcontain5as0doesnot
occur.

Theotherwaytosolvethisproblemisbywritingacomputerprogramthat
systematicallytriesallpossibilities.

1/3rdofthecontentsofacontainerevaporatedonthe1stday.3/4thofthe
remainingcontentsofthecontainerevaporatedonthesecondday.

Whatpartofthecontentsofthecontainerisleftattheendofthesecond
day?
Answer

AssumethatcontentsofthecontainerisX

Onthefirstday1/3rdisevaporated.
(11/3)ofXisremainingi.e.(2/3)X

OntheSecondday3/4thisevaporated.Hence,
(13/4)of(2/3)Xisremaining
i.e.(1/4)(2/3)X=(1/6)X


Hence1/6thofthecontentsofthecontainerisremaining

Therearefourpeopleinaroom(notincludingyou).Exactlytwoofthese
fouralwaystellthetruth.Theothertwoalwayslie.

YouhavetofigureoutwhoiswhoINONLY2QUESTIONS.Your
questionshavetobeYESorNOquestionsandcanonlybeansweredbyone
person.(Ifyouaskthesamequestiontotwodifferentpeoplethenthat
countsastwoquestions).Keepinmindthatallfourknoweachother's
characteristicswhethertheylieornot.

Whatquestionswouldyouasktofigureoutwhoiswho?Rememberthat
youcanaskonly2questions.
Submitted
Youhave3baskets,&eachonecontainsexactly4balls,eachofwhichisof
thesamesize.Eachballiseitherred,black,white,orpurple,&thereisone
ofeachcolorineachbasket.

Ifyouwereblindfolded,&lightlyshookeachbasketsothattheballswould
berandomlydistributed,&thentook1ballfromeachbasket,whatchanceis
therethatyouwouldhaveexactly2redballs?

Answer

Thereare64differentpossibleoutcomes,&in9ofthese,exactly2ofthe
ballswillbered.Thereisthusaslightlybetterthan14%chance
[(9/64)*100]thatexactly2ballswillbered.

Amuchfasterwaytosolvetheproblemistolookatitthisway.Thereare3
scenarioswhereexactly3ballsarered:

123

RRX

RXR


XRR

Xisanyballthatisnotred.
Thereisa4.6875%chancethateachofthesesituationswilloccur.

Takethefirstone,forexample:25%chancethefirstballisred,multiplied
bya25%chancethesecondballisred,multipliedbya75%chancethethird
ballisnotred.

Becausethereare3scenarioswherethisoutcomeoccurs,youmultiplythe
4.6875%chanceofanyoneoccurringby3,&youget14.0625%

Considerastatelotterywhereyougettochoose8numbersfrom1to80,no
repetitonallowed.TheLotteryCommissionchooses11fromthose80
numbers,againnorepetition.Youwinthelotteryifatleast7ofyour
numbersarethereinthe11chosenbytheLotteryCommission.

Whatistheprobablityofwinningthelottery?

Answer

Theprobabilityofwinningthelotteryistwoinonebillioni.e.onlytwo
personcanwinfromonebillion!!!

Let'sfindoutsamplespacefirst.TheLotteryCommissionchooses11
numbersfromthe80.Hence,the11numbersfromthe80canbeselected
80
13
in
C
wayswhichisveryveryhighandisequalto1.04776*10

11

Now,youhavetoselect8numbersfrom80whichcanbeselectedin
80C8ways.Butweareinterestedinonlythosenumberswhicharein11
numbersselectedbytheLotteryCommision.Thereare2cases.
Youmightselect8numberswhichallaretherein11numbers
11
choosenbytheLotteryCommission.Sothereare
C
ways.
8
Anothercaseisyoumightselect7luckynumbersand1nonlucky
11
69
numberfromtheremaining69numbers.Thereare(
C
)*(
C
)
7
1
waystodothat.

Sototalluckywaysare
11
11
69
=(
C
)+(
C
)*(
C
)
8
7
1
=(165)+(330)*(69)
=165+22770
=22935

Hence,theprobabilityofthewinninglotteryis
=(Totalluckyways)/(TotalSamplespace)
13
=(22935)/(1.04776*10
)
9
=2.1889*10

i.e.2inabillion.

Submit
Users
BrainVista

Puzzle
Answer
Answer(4)
Answer
TomoveaSafe,twocylindricalsteelbars7inchesindiameterareusedas
rollers.

Howfarwillthesafehavemovedforwardwhentherollershavemadeone
revolution?
Answer

Thesafemusthavemoved22inchesforward.

Iftherollersmakeonerevolution,thesafewillmovethedistanceequalto
thecircumferenceoftheroller.Hence,thedistancecoveredbythesafeis
=PI*Diameter(or2*PI*Radius)
=PI*7
=3.14159265*7
=21.99115
=22inchesapprox.
Submitt
Ifarookandabishopofastandardchesssetarerandomlyplacedon
achessboard,whatistheprobabilitythatoneisattackingtheother?

Notethatbotharedifferentcoloredpieces.
Subm
Answer

TheprobabilityofeithertheRookortheBishopattackingtheotheris
0.3611

64
ARookandaBishoponastandardchessboardcanbearrangedin
P
=
2
64*63=4032ways

Now,thereare2casesRookattackingBishopandBishopattackingRook.
NotethattheRookandtheBishopneverattackeachothersimultaneously.
Let'sconsiderboththecasesonebyone.

CaseIRookattackingBishop
TheRookcanbeplacedinanyofthegiven64positionsanditalways
attacks14positions.Hence,totalpossiblewaysoftheRookattackingthe
Bishop=64*14=896ways

CaseIIBishopattackingRook
Viewthechessboardasa4cocentrichollowsquareswiththeoutermost
squarewithside8unitsandtheinnermostsquarewithside2units.

Ifthebishopisinoneoftheouter28squares,thenitcanattack7positions.
Ifthebishopisinoneofthe20squaresatnextinnerlevel,thenitcanattack
9positions.Similarlyifthebishopisinoneofthe12squaresatnext
innerlevel,thenitcanattack11positions.Andifthebishopisinoneofthe
4squaresatnextinnerlevel(theinnermostlevel),thenitcanattack13
positions.

Hence,totalpossiblewaysoftheBishopattackingtheRook
=28*7+20*9+12*11+4*13
=560ways

Thus,therequiredprobabilityis
=(896+560)/4032
=13/36
=0.3611
itted
ed
HereinEnglandMcDonald'shasjustlaunchedanewadvertisingcampaign.
Thepostershows8McDonald'sproductsandunderneathclaimsthereare
40312combinationsoftheaboveitems.

Giventhatthemaximumnumberofitemsallowedis8,andyouareallowed
tohavelessthan8items,andthattheorderofpurchasedoesnotmatter(i.e.
buyingaburgerandfriesisthesameasbuyingfriesandaburger)

Howmanypossiblecombinationsarethere?AreMcDonald'scorrectin
claimingthereare40312combinations?
Answer

Totalpossiblecombinationsare12869.

Itisgiventhatyoucanordermaximumof8itemsandyouareallowedto
havelessthan8items.Also,theorderofpurchasedoesnotmatter.Let's
createatablefororderingtotalNitemsusingXproducts.
Pro
duct
s
Use
d
(X)

Items
Ordered
(N)

10

10

15

20

15

21

35

35

21

Total(T)

28

56

70

56

28

Waystochoose 8C1 8C2


Xproductsfrom
8products(W)

8C3

8C4

8C5 8C6 8C7 8C8

Totalcombinations
(T*W)

64 784 3136 4900 3136 784

64

Thus,totalpossiblecombinationsare
=64+784+3136+4900+3136+784+64+1
=12869
Whatarethechancesthatatleasttwooutofagroupoffiftypeoplesharethe
samebirthday?
Submitted
Answer

Theprobabilityofatleasttwooutofagroupof50peoplesharethesame
birthdayis97%

Probabilityofatleasttwosharethesamebirthday=1probabilityofall50
havedifferentbirthdays

Probabilityofall50havedifferentbirthday
=365/365*364/365*363/365*...*317/365*316/365
50
=(365*364*363*362*...*317*316)/365

=0.0296264

Probabilityofatleasttwosharethesamebirthday
=10.0296264
=0.9703735
=97%approx.

Thus,theprobabilityofatleasttwooutofagroupof50peoplesharethe
samebirthdayis97%

Thisexplains
whyinaschool/collegewithclassroomsof50students,
thereareatleasttwostudentswithabirthdayonthesamedayofthe
year
.Also,ifthereare23peopleintheroom,thenthereare50%chances
thatatleasttwoofthemhaveabirthdayonthesamedayoftheyear!!!

AtankcanbefilledbypipeAin30minutesandbypipeBin24minutes.
OutletpipeCcanemptythefulltankinXminutes.

IfthetankisemptyinitiallyandifallthethreepipesA,BandCareopened

simultaneously,thetankwillNEVERbefull.Givethemaximalpossible
valueofX.
Answer

ThemaximumpossiblevalueofXis13minutes20seconds.

Inoneminute,
pipeAcanfill1/30partofthetank.
pipeBcanfill1/24partofthetank.

Thus,thenetwaterlevelincreaseinoneminuteis
=1/30+1/24
=3/40partofthetank

Inordertokeepthetankalwaysempty,outletpipeCshouldemptyatleast
3/40partofthetankinoneminute.Thus,pipeCcanemptythefulltankin
40/3i.e.13minutes20seconds.
Aworkerearnsa5%raise.Ayearlater,theworkerreceivesa2.5%cutin
pay,&nowhissalaryisRs.22702.68

Whatwashissalarytobeginwith?
Answer

Rs.22176

AssumehissalarywasRs.X

Heearns5%raise.Sohissalaryis(105*X)/100

Ayearlaterhereceives2.5%cut.Sohissalaryis((105*X)/100)*(97.5/100)
whichisRs.22702.68

Hence,solvingequation((105*X)/100)*(97.5/100)=22702.68
X=22176

500menarearrangedinanarrayof10rowsand50columnsaccordingto
theirheights.

Tallestamongeachrowofallareaskedtocomeout.Andtheshortest

amongthemisA.

Similarlyafterresumingthemtotheiroriginalpositions,theshortestamong
eachcolumnareaskedtocomeout.AndthetallestamongthemisB.

NowwhoistallerAorB?
ApersonwantedtowithdrawXrupeesandYpaisefromthebank.But
cashiermadeamistakeandgavehimYrupeesandXpaise.Neitherthe
personnorthecashiernoticedthat.

Afterspending20paise,thepersoncountsthemoney.Andtohissurprise,
hehasdoubletheamounthewantedtowithdraw.

FindXandY.(1Rupee=100Paise)

Asgiven,thepersonwantedtowithdraw100X+Ypaise.

Buthegot100Y+Xpaise.

Afterspending20paise,hehasdoubletheamounthewantedto
withdraw.Hence,theequationis

2*(100X+Y)=100Y+X20

200X+2Y=100Y+X20

199X98Y=20

98Y199X=20

Now,wegotoneequationbutthereare2variables.Wehavetoapply
littlebitoflogicoverhere.WeknowthatifweinterchangeX&Y,
amountgetsdouble.SoYshouldbetwiceofXoronemorethantwiceof
Xi.e.Y=2XorY=2X+1

CaseI:
Y=2X

Solvingtwoequationssimultaneously
98Y199X=20
Y2X=0
WegetX=20/3&Y=40/2

CaseII:
Y=2X+1
Solvingtwoequationssimultaneously
98Y199X=20
Y2X=1
WegetX=26&Y=53

Now,itsobviousthathewantedtowithdrawRs.26.53

Submit

Answer

Users
Answer(2)

BrainVista
Answer

Puzzle

AttheParty:
1. Therewere9menandchildren.
2. Therewere2morewomenthanchildren.
3. Thenumberofdifferentmanwomancouplespossiblewas24.Note
thatiftherewere7menand5women,thentherewouldhavebeen35
manwomancouplespossible.

Also,ofthethreegroupsmen,womenandchildrenattheparty:
4. Therewere4ofonegroup.
5. Therewere6ofonegroup.
6. Therewere8ofonegroup.
Exactlyoneoftheabove6statementsisfalse.

Canyoutellwhichoneisfalse?Also,howmanymen,womenandchildren
arethereattheparty?
Answer

Statement(4)isfalse.Thereare3men,8womenand6children.

AssumethatStatements(4),(5)and(6)arealltrue.Then,Statement(1)is

false.ButthenStatement(2)and(3)bothcannotbetrue.Thus,
contradictorytothefactthatexactlyonestatementisfalse.

SoStatement(4)orStatement(5)orStatement(6)isfalse.Also,Statements
(1),(2)and(3)allaretrue.

From(1)and(2),thereare11menandwomen.Thenfrom(3),thereare2
possiblecaseseitherthereare8menand3womenorthereare3menand8
women.

Ifthereare8menand3women,thenthereis1child.ThenStatements(4)
and(5)botharefalse,whichisnotpossible.

Hence,thereare3men,8womenand6children.Statement(4)isfalse.

BrainTeaserNo:00242

Thereisashortageoftubelights,bulbsandfansinavillageKharghar.It
isfoundthat
Allhousesdonothaveeithertubelightorbulborfan.
exactly19%ofhousesdonothavejustoneofthese.
atleast67%ofhousesdonothavetubelights.
atleast83%ofhousesdonothavebulbs.
atleast73%ofhousesdonothavefans.
Whatpercentageofhousesdonothavetubelight,bulbandfan?

Answer

42%housesdonothavetubelight,bulbandfan.

Let'sassumethatthereare100houses.Hence,thereshouldbetotal300
itemsi.e.100tubelights,100bulbsand100fans.

Fromthegivendata,weknowthatthereisshortageofatleast(67+83+73)
223itemsinevery100houses.

Also,exactly19housesdonothavejustoneitem.Itmeansthatremaining
81housesshouldaccountfortheshortageofremaining(22319)204items.

Ifthoseremaining81housesdonothave2itemseach,therewouldbea
shortageof162items.Buttotalof204itemsareshort.Hence,atleast
(204162)42housesdonothaveall3itemstubelight,bulbandfan.

Thus,42%housesdonothavetubelight,bulbandfan.
Whatistheremainderleftafterdividing1!+2!+3!++100!By7?

Thinkcarefully!!!
Answer

Atrickyone.

7!onwardsalltermsaredivisibleby7as7isoneofthefactor.Sothereis
noremainderleftforthosetermsi.e.remainderleftafterdividing7!+8!+
9!+...+100!is0.

Theonlyparttobeconsideris
=1!+2!+3!+4!+5!+6!
=1+2+6+24+120+720
=873

Theremainderleftafterdividing873by7is5

Hence,theremainderis5.

Imaginethatyouhave26constants,labelledAthroughZ.Eachconstantis
assignedavalueinthefollowingway:A=1therestofthevaluesequal
theirpositioninthealphabet(Bcorrespondstothesecondpositionsoit
equals2,C=3,etc.)raisedtothepowerofthepreceedingconstantvalue.
So,B=2^(A'svalue),orB=2^1=2.C=3^2=9.D=4^9,etc.

Findtheexactnumericalvaluetothefollowingequation:(XA)*(XB)*
(XC)*...*(XY)*(XZ)

Answer

(XA)*(XB)*(XC)*...*(XY)*(XZ)equals0since(XX)is
zero
Ifthreebabiesareborneverysecondoftheday,thenhowmanybabieswill

bebornintheyear2001?
Su
Answer

9,46,08,000babies

Thetotalsecondsinyear2001
=365days/year*24hours/day*60minutes/hours*60seconds/minute
=365*24*60*60seconds
=3,15,36,000seconds

Thus,thereare3,15,36,000secondsintheyear2001.Also,threebabies
bornareeverysecond.Hence,totalbabiesborn
=3*3,15,36,000seconds
=9,46,08,000
bmitted

Replacetheletterswiththecorrectnumbers.
TWO

XTWO

THREE

Submittedby:TimmyChan
Answer

T=1,W=3,O=8,H=9,R=2,E=4
138

x138

19044
Youcanreducethenumberoftrials.Tmustbe1asthereismultiplicationof
TwithTinhundred'sposition.Also,Ocannotbe0or1.Now,youhaveto
findthreedigitnumberwhosesquaresatisfiesaboveconditionsandsquare
ofthathassamelasttwodigits.Hence,itmustbebetween102and139.

BrainTeaserNo:00052

Fourwordsadduptoafifthwordnumerically:

mars

venus

uranus

saturn

neptune

Eachofthetenletters(m,a,r,s,v,e,n,u,t,andp)representaunique
numberfromtherange0..9.

Furthermore,numbers1and6arebeingusedmostfrequently.

Answer

Theeasiestwaytosolvethisproblemisbywritingacomputerprogramthat
systematicallytriesallpossiblemappingsfromthenumbersontotheletters.
Thiswillgiveyouonlyonesolutionwhichmeetstheconditionthatnumbers
1and6aremostfrequentlyused.

marsm=4

venusa=5

uranusr=9

saturns=3

+v=24593

neptunee=020163


n=1695163

u=6358691

t=8+

p=71078610

Thereare4armymen.Theyhavebeencapturedbyarebelgroupandhave
beenheldatransom.Anarmyintelligentofficerordersthemtobeburried
deepindirtuptotheirnecks.Theformatoftheirburrialareasshowninthe
figure.

Conditions
Theyeachhavehatsontheirheads.eitherblack(b)orwhite(w)look
atdiagramabove.Therearetotal2whitehatsand2blackhats.
Theyonlylookinfrontofthemnotbehind.Theyarenotallowedto
communicatebytalking.
Betweenarmyman1and2,thereisawall.
Captiveman4canseethecolourofhatson2and3
3canonlysee2'shat
2canonlyseeawalland1canseeawalltoo,butisontheotherside
Theofficerspeaksup,"Ifoneofyoucancorrectlytellmethecolourofyour
hat,youwillallgoscottfreebacktoyourcontries.Ifyouarewrong,you
willallbekilled.

Howcanoneofthembecertainaboutthehattheyarewearingandnotrisk
thelivesoftheirfellowsouldiersbytakinga50/50guess!
Submitted
Answer

Eithersoldier3orsoldier4cansavethelifeassoldier1andsoldier2can
notseecolourofanyhat,evennottheirown..Inourcasesoldier3willtell
thecolourofhishat.

Soldier4canseethehatonsoldier2andsoldier3.Ifbotharewhite,thenhe
canbesureaboutcolourofhishatwhichwillbeblackandviceversa.Butif

oneofthemiswhiteandoneisblack,thensoldier4cannotsayanythingas
hecanhaveeitherofthem.Sohewillkeepmum.

Ifsoldier4won'tsayanyhingforawhile,thensoldier3willknowthat
soldier4isnotinpositiontotellthecolourofhatonhishat.Itmeansthat
colourofsoldier3'shatisoppositeofcolourofsoldier2'shat.Sosoldier3
cantellcorrectlythecolourofhatonhisheadwhichisBlack.

Here,weareassumingthatallthesoldiersareintelligentenough.Also,this
solutionwillworkforanycombinationof2Blackhatsand2Whitehats.
Onesideofthebottomlayerofatriangularpyramidhas12balls.Howmany
arethereinthewholepyramid?

Notethatthepyramidisequilateralandsolid.
Answer

Therearetotal364balls.

Asthereare12ballsalongoneside,itmeansthatthereare12layersof
balls.Thetopmostlayerhas1ball.Thesecondlayerhas3(1+2)balls.The
thirdlayerhas6(1+2+3)balls.Thefourthlayerhas10(1+2+3+4)balls.The
fifthlayerhas15(1+2+3+4+5)balls.Similarly,thereare21,28,36,45,55,
66and78ballsintheremaininglayers.

Hence,thetotalnumberofballsare
=1+3+6+10+15+21+28+36+45+55+66+78
=364balls

Ablindfoldedmanisaskedtositinthefrontofacarromboard.Theholesof
theboardareshutwithlidsinrandomorder,i.e.anynumberofallthefour
holescanbeshutoropen.

Nowthemanissupposedtotouchanytwoholesatatimeandcandothe
following.
Opentheclosedhole.
Closetheopenhole.
Lettheholebeasitis.

Afterhehasdoneit,thecarromboardisrotatedandagainbroughttosome
position.Themanisagainnotawareofwhataretheholeswhichareopenor
closed.

Howmanyminimumnumberofturnsdoestheblindfoldedmanrequireto
eitheropenalltheholesorclosealltheholes?

Notethatwheneveralltheholesareeitheropenorclose,therewillbean
alarmsothattheblindfoldedmanwillknowthathehaswon.
Submitted
Answer

Theblindfoldedmanrequires5turns.
1.
2.
3.

4.

5.

Opentwoadjacentholes.
Opentwodiagonalholes.Nowatleast3holesareopen.If4thholeis
alsoopen,thenyouaredone.Ifnot,the4thholeisclose.
Checktwodiagonalholes.
Ifoneisclose,openitandalltheholesareopen.
Ifbothareclose,openanyonehole.Now,twoholesareopen
andtwoareclose.Thediagonalholesareintheoppositestatus
i.e.inboththediagonals,oneholeisopenandoneisclose.

Checkanytwoadjacentholes.
Ifbothareopen,closebothofthem.Now,allholesareclose.
Ifbothareclose,openbothofthem.Now,allholesareopen.
Ifoneisopenandoneisclose,invertthemi.e.closetheopen
holeandopentheclosehole.Now,thediagonalholesareinthe
samestatusi.e.twoholesinonediagonalareopenandinother
areclose.

Checkanytwodiagonalholes.
Ifbothareopen,closebothofthem.Now,allholesareclose.
Ifbothareclose,openbothofthem.Now,allholesareopen.

Inthemiddleoftheconfoundeddesert,thereisthelostcityof"Ash".To
reachit,Iwillhavetotraveloverlandbyfootfromthecoast.Onatreklike

this,eachpersoncanonlycarryenoughrationsforfivedaysandthefarthest
wecantravelinonedayis30miles.Also,thecityis120milesfromthe
startingpoint.

WhatIamtryingtofigureoutisthefewestnumberofpersons,including
myself,thatIwillneedinourGroupsothatIcanreachthecity,stay
overnight,andthenreturntothecoastwithoutrunningoutofsupplies.

Howmanypersons(includingmyself)willIneedtoaccomplishthis
mission?
Answer

Total4persons(includingyou)required.

Itisgiventhateachpersoncanonlycarryenoughrationsforfivedays.And
thereare4persons.Hence,totalof20daysrationsisavailable.
1. FirstDay
:4daysofrationsareusedup.Onepersongoesbackusing
onedayofrationsforthereturntrip.Therationsremainingforthe
furthertrekisfor15days.
2. SecondDay
:Theremainingthreepeopleuseup3daysofrations.
Onepersongoesbackusing2daysofrationsforthereturntrip.The
rationsremainingforthefurthertrekisfor10days.
3. ThirdDay
:Theremainingtwopeopleuseup2daysofrations.One
persongoesbackusing3daysofrationsforthereturntrip.The
rationsremainingforthefurthertrekisfor5days.
4. FourthDay
:Theremainingpersonusesuponedayofrations.He
staysovernight.Thenextdayhereturnstothecoastusing4daysof
rations.

Thus,total4persons,includingyouarerequired.
Atwhattimeafter4.00p.m.istheminuteshandofaclockexactlyaligned
withthehourhand?
Answer

4:21:49.5

AssumethatXminutesafter4.00PMminutehandexactlyalignswithand
hourhand.


Foreveryminute,minutehandtravels6degrees.
Hence,forXminutesitwilltravel6*Xdegrees.

Foreveryminute,hourhandtravels1/2degrees.
Hence,forXminutesitwilltravelX/2degrees.

At4.00PM,theanglebetweenminutehandandhourhandis120degrees.
Also,afterXminutes,minutehandandhourhandareexactlyaligned.So
theanglewithrespectto12i.e.VerticalPlanewillbesame.Therefore,

6*X=120+X/2
12*X=240+X
11*X=240
X=21.8182
X=21minutes49.5seconds

Hence,at4:21:49.5minutehandisexactlyalignedwiththehourhand.

SubstitutedigitsfortheletterstomakethefollowingDivisiontrue
OUT

STEM|DEMISE

|DMOC

TUIS

STEM

ZZZE

ZUMM

IST
Notethattheleftmostlettercan'tbezeroinanyword.Also,theremustbea
onetoonemappingbetweendigitsandletters.e.g.ifyousubstitute3forthe
letterM,nootherlettercanbe3andallotherMinthepuzzlemustbe3.
Submittedby:Calon
Answer

C=0,U=1,S=2,T=3,O=4,M=5,I=6,Z=7,E=8,D=9

ItisobviousthatU=1(asU*STEM=STEM)andC=0(asIC=I).

S*OisasingledigitandalsoS*Tisasingledigit.Hence,theirvalues(O,S,
T)mustbe2,3or4(astheycannotbe0or1orgreaterthan4).

Consider,STEM*O=DMOC,whereC=0.ItmeansthatMmustbe5.Now,
itssimple.O=4,S=2,T=3,E=8,Z=7,I=6andD=9.
OUT413

STEM|DEMISE2385|985628

|DMOC|9540

TUIS3162

STEM2385

ZZZE7778

ZUMM7155

IST623
Also,whenarrangedfrom0to9,itspells
CUSTOMIZED
.

BrainTeaserNo:00015

InthetowncalledAlibaug,thefollowingfactsaretrue:
Notwoinhabitantshaveexactlythesamenumberofhairs.
Noinhabitantshasexactly2025hairs.
Therearemoreinhabitantsthantherearehairsontheheadofany
oneinhabitants.
WhatisthelargestpossiblenumberoftheinhabitantsofAlibaug?

Answer

2025

Itisgiventhatnoinhabitantshaveexactly2025hairs.Hencethereare2025
inhabitantswith0to2024hairsinthehead.

Supposetherearemorethan2025inhabitants.Butthesewillviolatethe
conditionthat"Therearemoreinhabitantsthantherearehairsontheheadof
anyoneinhabitants."Asforanynumbermorethan2025,therewillbesame
numberofinhabitantsasthemaximumnumberofhairsontheheadofany
inhabitant.
TherearefourgroupsofMangoes,ApplesandBananasasfollows:
GroupI:1Mango,1Applesand1Banana
GroupII:1Mango,5Applesand7Bananas
GroupIII:1Mango,7Applesand10Bananas
GroupIV:9Mango,23Applesand30Bananas

GroupIIcostsRs300andGroupIIIcostsRs390.

CanyoutellhowmuchdoesGroupIandGroupIVcost?
Answer

GroupIcostsRs120andGroupIVcostsRs1710


Assumethatthevaluesofonemango,oneappleandonebananaareM,A
andBrespectively.

FromGroupII:M+5A+7B=300
FromGroupIII:M+7A+10B=390

Subtractingabovetoequations:2A+3B=90

ForGroupI:
=M+A+B
=(M+5A+7B)(4A+6B)
=(M+5A+7B)2(2A+3B)
=3002(90)
=300180
=120

Similarly,forGroupIV:
=9M+23A+30B
=9(M+5A+7B)(22A+33B)
=9(M+5A+7B)11(2A+3B)
=9(300)11(90)
=2700990
=1710

Thus,GroupIcostsRs120andGroupIVcostsRs1710.
TicTacToeisbeingplayed.One'X'hasbeenplacedinoneofthecorners.
No'O'hasbeenplacedyet.

Wheredoestheplayerthatisplaying'O'hastoputhisfirst'O'sothat'X'
doesn'twin?

Assumethatbothplayersareveryintelligent.Explainyouranswer
Answer

"O"shouldbeplacedinthecenter.

Let'snumberthepositionsas:
1|2|3

4|5|6

7|8|9
Itisgiventhat"X"isplacedinoneofthecornerposition.Let'sassumethat
itsatposition1.

Now,let'stakeeachpositiononebyone.
If"O"isplacedinposition2,"X"canalwayswinbychoosing
position4,5or7.
If"O"isplacedinposition3,"X"canalwayswinbychoosing
position4,7or9.
If"O"isplacedinposition4,"X"canalwayswinbychoosing
position2,3or5.
If"O"isplacedinposition6,"X"canalwayswinbychoosing
position3,5or7.
If"O"isplacedinposition7,"X"canalwayswinbychoosing
position2,3or9.
If"O"isplacedinposition8,"X"canalwayswinbychoosing
position3,5or7.
If"O"isplacedinposition9,"X"canalwayswinbychoosing
position3,or7.
If"O"isplacedinposition5i.e.centerposition,"X"can'twinunless"O"
doessomethingfoolish))

Hence,"O"shouldbeplacedinthecenter.

Amit,Bhavin,HimanshuandRakesharesittingaroundatable.
TheElectonicsEngineerissittingtotheleftoftheMechanical
Engineer.
AmitissittingoppositetoComputerEngineer.
HimanshulikestoplayComputerGames.
BhavinissittingtotherightoftheChemicalEngineer.
Canyoufigureouteveryone'sprofession?

Answer

AmitistheMechanicalEngineer.BhavinistheComputerEngineer.
HimanshuandRakeshareeitherChemicalEngineerorElecronicsEngineer.

AmitandBhavinaresittingoppositetoeachother.WhereasChemical
EngineerandElecronicsEngineeraresittingoppositetoeachother.

WecannotfindoutwhoisChemicalEngineerandElecronicsEngineeras
dataprovidedisnotsufficient

FivefriendswithsurnameBatliwala,Pocketwala,Talawala,Chunawalaand
Natakwalahavetheirfirstnameandmiddlenameasfollow.
1. FourofthemhaveafirstandmiddlenameofParesh.
2. ThreeofthemhaveafirstandmiddlenameofKamlesh.
3. TwoofthemhaveafirstandmiddlenameofNaresh.
4. OneofthemhaveafirstandmiddlenameofElesh.
5. PocketwalaandTalawala,eitherbotharenamedKamleshorneitheris
namedKamlesh.
6. EitherBatliwalaandPocketwalabotharenamedNareshorTalawala
andChunawalabotharenamedNaresh.
7. ChunawalaandNatakwalaarenotbothnamedParesh.
WhoisnamedElesh?
Answer

PocketwalaisnamedElesh.

From(1)and(7),itisclearthatBatliwala,PocketwalaandTalawalaare
namedParesh.

From(6)and(5),ifPocketwalaorTalawalabotharenamedKamlesh,then
eitherofthemwillhavethreenamesi.e.Paresh,KamleshandNaresh.
Hence,PocketwalaandTalawalabotharenotnamedKamlesh.Itmeansthat
Batliwala,ChunawalaandNatakwalaarenamedKamlesh.

NowitisclearthatTalawalaandChunawalaarenamedNaresh.Also,
PocketwalaisnamedElesh.

Mr.Waglegoestoworkbyabus.Onedayhefallsasleepwhenthebusstill
hastwiceasfartogoasithasalreadygone.

Halfwaythroughthetriphewakesupasthebusbouncesoversomebad
potholes.Whenhefinallyfallsasleepagain,thebusstillhashalfthe
distancetogothatithasalreadytravelled.Fortunately,Mr.Waglewakesup
attheendofhistrip.

WhatportionofthetotaltripdidMr.Waglesleep?

Answer

Mr.waglesleptthroughhalfhistrip.

Let'sdrawatimeline.Picturethebusrouteonalineshowenbelow:

________________________

Start1/31/22/3End

showstimeforwhichMr.Waglewasnotsleeping

_____showstimeforwhichMr.Waglewassleeping

WhenMr.Waglefellasleepthefirsttime,thebussillhadtwiceasfartogo
asithadalreadygone,thatmarksthefirstthirdofhistrip.

Hewakeuphalfwaythroughthetripi.esleptfrom1/3marktothe1/2mark.
Hefellsleepagainwhenthebusstillhadhalfthedistancetogothatithad
alreadytraveledi.e2/3mark.

Addingup,allsleepingtimes,
=(1/21/3)+(12/3)
=1/6+1/3
=1/2

Hence,Mr.waglesleptthroughhalfhistrip.

BrainTeaserNo:00068

Inyoursockdrawer,youhavearatioof5pairsofbluesocks,4pairsof
brownsocks,and6pairsofblacksocks.

Incompletedarkness,howmanysockswouldyouneedtopullouttoget
amatchingpairofthesamecolor?

4
Ifyoudon'tagree,tryityourself!
Youhaveabucketofjellybeans.Somearered,someareblue,andsome
green.Withyoureyesclosed,pickout2ofalikecolor.

Howmanydoyouhavetograbtobesureyouhave2ofthesame?

Youhaveabucketofjellybeans.Somearered,someareblue,andsome
green.Withyoureyesclosed,pickout2ofalikecolor.

Howmanydoyouhavetograbtobesureyouhave2ofthesame?
Ifyouselect4Jellybeansyouareguarenteedthatyouwillhave2thatare
thesamecolor.

Thereare70employeesworkingwithBrainVistaofwhich30arefemales.
Also,
30employeesaremarried
24employeesareabove25yearsofage
19marriedemployeesareabove25years,ofwhich7aremales
12malesareabove25yearsofage
15malesaremarried.
Howmanyunmarriedfemalesarethereandhowmanyofthemareabove
25?
Answer

15unmarriedfemales&noneareabove25yearsofage.

Simplyputallgiveninformationintothetablestructureandyouwillgetthe
answer.

Unmarrie
d

Married
Below25

Above25

Below25

Above25

Female

12

15

Male

20

Thereisasafewitha5digitnumberasthekey.The4thdigitis4greater
thantheseconddigit,whilethe3rddigitis3lessthanthe2nddigit.The1st
digitisthricethelastdigit.Thereare3pairswhosesumis11.

Findthenumber.
Answer

65292

Aspergivenconditions,therearethreepossiblecombinationsfor2nd,3rd
and4thdigits.Theyare(3,0,7)or(4,1,8)or(5,2,9)

Itisgiventhatthereare3pairswhosesumis11.Allpossiblepairsare(2,
9),(3,8),(4,7),(5,6).Nowrequirednumberis5digitnumberandit
contains3pairsof11.Soitmustnotbehaving0and1init.Hence,theonly
possiblecombinationfor2nd,3rdand4thdigitsis(5,2,9)

Also,1stdigitisthricethelastdigit.Thepossiblecombinationsare(3,1),
(6,2)and(9,3),outofwhichonly(6,2)with(5,2,9)gives3pairsof11.
Hence,theansweris65292.
Myfriendcollectsantiquestamps.Shepurchasedtwo,butfoundthatshe
neededtoraisemoneyurgently.SoshesoldthemforRs.8000each.Onone
shemade20%andontheothershelost20%.

Howmuchdidshegainorloseintheentiretransaction?
Answer

ShelostRs666.67

Considerthefirststamp.Shemades20%onitaftersellingitforRs8000.

Sotheoriginalpriceoffirststampis
=(8000*100)/80
=Rs6666.67

Similarly,considersecondstamp.Shelost20%onitaftersellingitforRs
8000

Sotheoriginalpriceofsecondstampis
=(8000*100)/80
=Rs10000

Totalbuyingpriceoftwostamps
=Rs6666.67+Rs10000
=Rs16666.67

Totalsellingpriceoftwostamps
=Rs8000+Rs8000
=Rs16000

Hence,shelostRs666.67

Assumeforamomentthattheearthisaperfectlyuniformsphereofradius
6400km.Supposeathreadequaltothelengthofthecircumferenceofthe
earthwasplacedalongtheequator,anddrawntoatightfit.

Nowsupposethatthelengthofthethreadisincreasedby12cm,andthatit
ispulledawayuniformlyinalldirections.

Byhowmanycm.willthethreadbeseparatedfromtheearth'ssurface?
Answer

Thecicumferenceoftheearthis
=2*PI*r
=2*PI*6400km
=2*PI*6400*1000m
=2*PI*6400*1000*100cm
=1280000000*PIcm

wherer=radiusoftheearth,PI=3.141592654

Hence,thelengthofthethreadis=1280000000*PIcm

Nowlengthofthethreadisincreasdby12cm.Sothenewlengthis=
(1280000000*PI)+12cm

Thisthreadwillmakeoneconcentriccirclewiththeearthwhichisslightly
awayfromtheearth.Thecircumfernceofthatcircleisnothingbut
(1280000000*PI)+12cm

AssumethatradiusoftheoutercircleisRcm
Therefore,
2*PI*R=(1280000000*PI)+12cm

Solvingaboveequation,R=640000001.908cm
Radiusoftheearthisr=640000000cm

Hence,thethreadwillbeseparatedfromtheearthby
=Rrcm
=640000001.908640000000
=1.908cm
Scientistdecidedtodoastudyonthepopulationgrowthofrabbits.Insidea
controlledenvironment,1000rabbitswereplaced.

Sixmonthslater,therewere1000Zrabbits.Atthebeginningofthe3rdyear,
therewereroughly2828Zrabbits,whichwas4timeswhatthescientists
placedinthereatthebeginningofthe1styear.

IfZisapositivevariable,howmanyrabbitswouldbethereatthebeginning
ofthe11thyear?
Submi
Answer

Atthebeginningofthe11thyear,therewouldbe1,024,000rabbits.

Atthebeginning,therewere1000rabbits.Also,therewere4000rabbitsat
thebeginningofthirdyearwhichisequalto2828Z.Thus,Z=4000/2828
i.e.1.414(thesquarerootof2)

Notethat2828Zcanberepresentedas2000*Z*Z(Z=1.414),whichcanbe
furthersimplifiedas1000*Z*Z*Z*Z

Also,itisgiventhatattheendof6months,therewere1000Zrabbits.

Itisclearthatthepopulationgrowthis1.414timeseverysixmonthsi.e.2
timeseveryyear.AfterNyears,thepopulationwouldbe1000*(Z^(2N))i.e.
1000*(2^N)

Thus,atthebeginningofthe11thyear(i.e.after10years),therewouldbe
1000*(2^10)i.e.1,024,000rabbits.
tted
Aclassof100students.24ofthemaregirlsand32arenot.WhichbaseamI
using?

Answer

LetthebasebeX.

Therefore
(X*X+X*0+0)=(2*X+4)+(3*X+2)
X*X=5*X+6
X*X5*X6=0
(X6)(X+1)=0

Thereforebaseis6

Amanisstrandedonadesertisland.Allhehastodrinkisa20ozbottleof
sprite.

Toconservehisdrinkhedecidesthatonthefirstdayhewilldrinkoneoz
andtherefillthebottlebackupwithwater.Onthe2nddayhewilldrink2oz
andrefillthebottle.Onthe3rddayhewilldrink3ozandsoon...

Bythetimeallthespriteisgone,howmuchwaterhashedrunk?
Submitt
Answer

Themandrunk190ozofwater.

Itisgiventhatthemanhas20ozbottleofsprite.Also,hewilldrink1ozon
thefirstdayandrefillthebottlewithwater,willdrink2ozonthesecondday
andrefillthebottle,willdrink3ozonthethirddayandrefillthebottle,and
soontill20thday.Thusattheendof20days,hemusthavedrunk(1+2+3
+4+.....+18+19+20)=210ozofliquid.

Outofthat210oz,20ozisthespritewhichhehadinitially.Hence,hemust
havedrunk190ozofwater.
ed
Youhavefour9'sandyoumayuseanyofthe(+,,/,*)asmanytimesas
youlike.Iwanttoseeamathematicalexpressionwhichusesthefour9'sto
=100

Howmanysuchexpressionscanyoumake?
Submitted
Answer

Thereare5suchexpressions.

99+(9/9)=100

(99/.99)=100

(9/.9)*(9/.9)=100

((9*9)+9)/.9=100

(999)/.9=100
Twoplanestakeoffatthesameexactmoment.Theyareflyingacrossthe
Atlantic.OneleavesNewYorkandisflyingtoParisat500milesperhour.
TheotherleavesParisandisflyingtoNewYorkatonly450milesperhour
(becauseofastrongheadwind).

WhichonewillbeclosertoPariswhentheymeet?
TheywillbothbethesamedistancefromPariswhentheymeet!!!

12memberswerepresentataboardmeeting.Eachmembershookhands
withalloftheothermembersbefore&afterthemeeting.


Howmanyhandshakeswerethere?
Answer

132

Thinkofitthisway:thefirstpersonshakeshandswith11people,thesecond
personalsoshakeshandswith11people,butyouonlycount10,becausethe
handshakewiththefirstpersonwasalreadycounted.Thenadd9forthe
thirdperson,8forthefourth,&soon.

66handshakestookplacebefore&66afterthemeeting,foratotalof132.
Arrangefiveplanetssuchthat4ofthemaddupto5thplanetnumerically.
Eachofthelettersoftheplanetshouldrepresentauniquenumberfromthe
range09.Youhavetousealltendigits.

Thereisanamazingmathematicalrelationshipexistsamongthenamesof
theplanet.
Answer

Thetoughtprocessisinitiallytofindplanetssuchthatthetotalnumberof
alphabetsinthemis10.

TheonlypossiblecombinationofplanetsisSaturn,Uranus,Venus,Mars
andNeptunebecauseforothercombinationstherewillbemorethan10
alphabets.Amongthesefive,Neptuneisthelenghtiest,soitmustbethesum
oftheotherfour.

SATURN

URANUS

VENUS

+MARS

NEPTUNE

NowtheonlypossiblevalueforNis1.ByfindingthevalueforS,wecan
reachtheresult:

358691

695163

20163

+4593

1078610

Youhave14apples.YourFriendMargetakesaway3andgivesyou2.You
drop7butpickup4.Brettakes4andgives5.YoutakeonefromMargeand
giveittoBretinexchangefor3more.Yougivethose3toMargeandshe
givesyouanappleandanorange.FrankcomesandtakestheappleMarge
gaveyouandgivesyouapear.YougivethepeartoBretinexchangeforan
apple.FrankthentakesanapplefromMarge,givesittoBretforanorange,
givesyoutheorangeforanapple.

Howmanypearsdoyouhave?
Submitted
Answer

None

FrankgaveyouapearinexchangeoftheapplewhichMargegaveyou.And
yougavethatpeartoBretinexchangeforanapple.Alltheothersexchanges
involvedapplesand/ororganges.
Fourcouplesaregoingtothemovie.Eachrowholdseightseats.Bettyand
Jimdon'twanttositnexttoAliceandTom.AliceandTomdon'twanttosit
nexttoGertrudeandBill.Ontheotherhand,SallyandBobdon'twanttosit
nexttoBettyandJim.

Howcanthecouplesarrangethemselvesinarowsothattheyallsitwhere
theywouldlike?
Submittedby:TaraSmith

Answer

Fromthegivendata,itcanbeinferredthat:
(Sally&Bob)NOT(Betty&Jim)NOT(Alice&Tom)NOT(Gertrude&
Bill)

(A)NOT(B)meansAandBcannotseatnexttoeachother.

Now,itisobviousthat(Betty&Jim)and(Alice&Tom)willoccupythe
cornerseatsasbothofthemcanhaveonlyoneneighbour.Therefore,
(Gertrude&Bill)willseatnextto(Betty&Jim)
(Sally&Bob)willseatnextto(Gertrude&Bill)
(Alice&Tom)willseatnextto(Sally&Bob)

Thus,therearetwopossiblearrangementsamirrorimagesofeachother.

1.(Betty&Jim)(Gertrude&Bill)(Sally&Bob)(Alice&Tom)
2.(Alice&Tom)(Sally&Bob)(Gertrude&Bill)(Betty&Jim)
Substitutedigitsfortheletterstomakethefollowingadditionproblemtrue.
WHOSE

TEETH

ARE

+AS

SWORDS
Notethattheleftmostlettercan'tbezeroinanyword.Also,theremustbea
onetoonemappingbetweendigitsandletters.e.g.ifyousubstitute3forthe
letterH,nootherlettercanbe3andallotherHinthepuzzlemustbe3.
Answer

ItisobviousthatS=1andT=9.

Also,(H+E)shouldbegreaterthan10andhence,(E+H+E)must20.
Thus,thereare3possiblevaluesfor(E,H)pair:(6,8)or(7,6)or(8,4).Use
trialnerrorandeverythingwillfitin.

WHOSE28516

TEETH96698

ARE476

+AS+41

SWORDS125731

WhenSocrateswasimprisonedforbeingadisturbinginfluence,hewasheld
inhighesteembyhisguards.Allfourofthemhopedthatsomethingwould
occurthatwouldfacilitatehisescape.Oneevening,theguardwhowason
dutyintentionallyleftthecelldooropensothatSocratescouldleavefor
distantparts.

Socratesdidnotattempttoescape,asitwashisphilosophythatifyouaccept
society'srules,youmustalsoacceptit'spunishments.However,theopen
doorwasconsideredbytheauthoritiestobeaseriousmatter.Itwasnotclear
whichguardwasonthatevening.Thefourguardsmakethefollowing
statementsintheirdefense:

Aaron:
A)Ididnotleavethedooropen.
B)Clementwastheonewhodidit.

Bob:
A)Iwasnottheonewhowasondutythatevening.
B)Aaronwasonduty.

Clement:

A)Bobwastheoneondutythatevening.
B)IhopedSocrateswouldescape.

David:
A)Ididnotleavethedooropen.
B)IwasnotsurprisedthatSocratesdidnotattempttoescape.

Consideringthat,intotal,threestatementsaretrue,andfivestatementsare
false,whichguardisguilty
Answer

Davidistheguilty.

Notethat"Allfourofthemhopedthatsomethingwouldoccurthatwould
facilitatehisescape".ItmakesClement'sstatementBTrueandDavid's
statementBFalse.

Nowconsidereachofthemasaguilty,oneatatime.
Aaro
Clem
True

Bob
David
n
ent
Stmts

IfAaron
isguilty

False False True

True

False True True

False

True True

False

False False True True

False

False False True False False

IfBobis
True
guilty

False False False True

If
Clement
isguilty

True

True

IfDavid
isguilty

True

False True

True

Sinceintotal,threestatementsaretrueandfivestatementsarefalse.Itis
clearfromtheabovetablethatDavidis?

BrainTeaserNo:00474

Givenanywholenumbertakethesumofthedigits,andtheproductofthe

digits,andmultiplythesetogethertogetanewwholenumber.

Forexample,startingwith6712,thesumofthedigitsis(6+7+1+2)=16,
andtheproductofthedigitsis(6*7*1*2)=84.Theanswerinthiscaseis
then84x16=1344.

Ifwedothisagainstartingfrom1344,weget(1+3+4+4)*(1*3*4*4)=
576

Andyetagain(5+7+6)*(5*7*6)=3780

Atthisstageweknowwhatthenextanswerwillbe(withoutworkingit
out)because,asonedigitis0,theproductofthedigitswillbe0,and
hencetheanswerwillalsobe0.

Canyoufindanynumberstowhichwhenweapplytheabovementioned
rulerepeatedly,weneverendupat0?

BrainTeaserNo:00474

Givenanywholenumbertakethesumofthedigits,andtheproductofthe
digits,andmultiplythesetogethertogetanewwholenumber.

Forexample,startingwith6712,thesumofthedigitsis(6+7+1+2)=16,
andtheproductofthedigitsis(6*7*1*2)=84.Theanswerinthiscaseis
then84x16=1344.

Ifwedothisagainstartingfrom1344,weget(1+3+4+4)*(1*3*4*4)=
576

Andyetagain(5+7+6)*(5*7*6)=3780

Atthisstageweknowwhatthenextanswerwillbe(withoutworkingit
out)because,asonedigitis0,theproductofthedigitswillbe0,and
hencetheanswerwillalsobe0.

Canyoufindanynumberstowhichwhenweapplytheabovementioned
rulerepeatedly,weneverendupat0?


TherewereNstationsonarailroad.AfteraddingXstations46additional
ticketshavetobeprinted.

FindNandX.
Answer

LetbeforeaddingXstations,totalnumberoftickets
t=N(N1)

AfteraddingXstationstotalnumberofticketsare
t+46=(N+X)(N+X1)

Subtracting1stfrom2nd
46=(N+X)(N+X1)N(N1)
2
2
2
46=N
+NXN+NX+X
XN
+N
2
46=2NX+X
X
2
46=(2N1)X+X

2
X
+(2N1)X46=0

Nowthereareonlytwopossiblefactorsof46.Theyare(46,1)and(23,2)

CaseI:(46,1)
2N1=45
2N=46
N=23
AndX=1

CaseII:(23,2)
2N1=21
2N=22
N=11
AndX=2

Hence,thereare2possibleanswers.

Anemergencyvehicletravels10milesataspeedof50milesperhour.

Howfastmustthevehicletravelonthereturntripiftheroundtriptravel

timeistobe20minutes?
Answer

75milesperhour

Whilegoingtothedestination,thevehicletravels10milsatthespeedof50
milesperhour.Sothetimetakentotravel10milesis
=(60*10)/50
=12minutes

Nowit'sgiventhatroundtriptraveltimeis20minutes.Sothevehicle
shouldcompleteitsreturntripof10milesin8minutes.Sothespeedofthe
vehiclemust
=(60*10)/8
=75milesperhour
Allofthestudentsatacollegearemajoringinpsychology,business,or
both.73%ofthestudentsarepsychologymajors,&62%arebusiness
majors.

Ifthereare200students,howmanyofthemaremajoringinboth
psychology&business?
Answer

70studentsaremajoringinboth,psychology&business

If73%ofthestudentsarepsychologymajors,weknowthat27%arenot
psychologymajors.Bythesamereasoning,38%arenotbusinessmajors,
because62%ofthestudentsdomajorinbusiness.So:27+38=65

65%ofthestudentsarenotmajoringinbothpsychology&business,so35%
aredoublemajors,atotalof70students.
Twotrainsstartingatsametime,onefromBangaloretoMysoreandotherin
oppositedirectionarriveattheirdestination1hrand4hrsrespectivelyafter
passingeachother.

Answer

ThespeedofBangaloreMysoretrainisTWICEthespeedof
MysoreBangaloretrain.

LetthedistancebetweenBangaloreandMysoreisDkms.
Also,letspeedofthetrainfromBangaloretoMysoreisXkm/hrandspeed
ofthetainfromMysoretoBangaloreisYkm/hr.

Now,assumethatboththetrainsmeteachotheratTkmsfromthe
Bangalore(pointPinfigure)
TimetakenbyBangaloreMysoretraintoreachP=Timetakenby
MysoreBangaloretraintoreachP
(T/X)=(DT)/Yequ(I)

Also,BangaloreMysoretrainandMysoreBangaloretrainarrivedestination
1hrand4hrsrespectivelyafterpassingeachother.Itmeansthat
BangaloreMysoretraintravels(DT)kmsin1hratXkm/hrand
MysoreBangaloretraintravelsTkmsin4hrsatYkm/hr.Hence,
(DT)=Xand
T=4*Y

SubstitutingthesevaluesinequationI,weget
(4*Y)/X=X/Y
4*Y*Y=X*X
2*Y=X

Hence,thespeedofBangaloreMysoretrainisTWICEthespeedof
MysoreBangaloretrain.
Howmuchfasterisonetrainfromother?

Answer

49times

Let'sassumethateveryoneclinkedtheirmugwithfriendtohisleftonly.It
meansthattherearetotal49clinks.Nowtherightclinkofeachpersonisleft

clinkofthepersononrightwhichisalreadyhappened.Hence,thereareonly
49clinks.

Mrs.Watsherfacehadagaragesale.AcustmernamedGinaboughtanold
lampandarug.Shepaidatotalof$5.25foreverything.Therugcost25
centsmorethanthelamp.

Howmuchdideachcost?
Submittedby:Kimi
Answer

Thelampcost$2.50andtherugcost$2.75

Asimpleone.

Assumethatthelampcost$L.
Hencetherugmusthavecost$(L+0.25)
Also,totalcostis$5.25,Hencetheequation:
L+L+0.25=5.25
2*L=5
L=2.50

Hence,thelampcost$2.50andtherugcost$2.75

BrainTeaserNo:00518

Write1111......(243times)i.e.a243digitnumberwithall1s.

Provethatitisdivisibleby243.
Submitt
Answer

Proveitusingthemathematicalinduction.

Firsthereareacoupleofthingstonote:

[1]Anumberwhosedigitsadduptoamultipleofthreeisdivisableby3.
e.g.369:3+6+9=18:1+8=9whichisamultipleof3hence369isdivisable
by3.

[2]Wheneveranumber(X)ismultipliedwithanothernumber(Y)thenthe
product(X*Y)willhaveallthefactorsofXaswellasallthefactorsofYin
itssetoffactors.
e.g.ifXhasfactorsof(1,P,Q,X)andYhasfactorsof(1,Q,R,Y)thenX*Y
hasfactorsof(1,P,Q,Q,R,X,Y).

Let
N=anyseriesofdigits(e.g.N=369)
D=thenumberofdigitsinN(e.g.ifN=369thenD=3)
P=isanumberconstructedinthefollowingway:a1,followedby(D1)0s,
followedbyanother1,followedby(D1)0s,followedbyanother1.(e.g.if
N=369thenD=3andPwouldbe1001001)NotethatPwillalwaysbe
divisibleby3.

Also,ifwemultiplyNwithPweareessentiallyrepeatingNfor(D1)times.
e.g.ifN=369thenD=3,P=1001001andN*P=369369369

Let'sstartwithN=111.ItisclearthatNisdivisibleby3.(From[1])
Also,D=3andP=1001001
N*P=111111111(9times)
Theresultingnumber111111111mustbedivisibleby9asNandPbothare
divisibleby3.

Now,let'sstartwithN=111111111.ItisclearthatNisdivisibleby9.
Also,D=9andP=1000000001000000001
N*P=111111111...(27times)
Theresultingnumber1111111...(27times)mustbedivisibleby27asNis
divisibleby9andPisdivisibleby3.

RepeatthesameprocedureforN=1111111...(27times)Theresulting
number1111111...(81times)mustbedivisibleby81asNisdivisibleby27
andPisdivisibleby3.

Similarly,forN=1111111...(81times)Theresultingnumber1111111...
(243times)mustbedivisibleby243asNisdivisibleby81andPis
divisibleby3.

Thus,1111111...(243times)isdivisibleby243.

ThankstoRyanHutchersonforsolution!!!
ed
Karanboughtalittleboxofmidgetmatches,eachoneinchinlength.He
foundthathecouldarrangethemallintheformofatrianglewhoseareawas
justasmanysquareinchesastherewerematches.

Hethenusedupsixofthematches,andfoundthatwiththeremainderhe
couldagainconstructanothertrianglewhoseareawasjustasmanysquare
inchesastherewerematches.

Andusinganothersixmatcheshecouldagaindopreciselythesame.

Howmanymatcheswerethereintheboxoriginally?

Notethatthematchboxcanholdmaximumof50matches.
Answer

Initially,therewere42or36matchesinthematchbox.

Thereare42matchesintheboxwithwhichhecouldformatriangle20,15,
7,withanareaof42squareinches.After6matcheshadbeenused,the
remaining36matcheswouldformatriangle17,10,9,withanareaof36
squareinches.Afterusinganother6matches,theremaining30matches
wouldformatriangle13,12,5,withanareaof30squareinches.Afterusing
another6,the24remainingwouldformatriangle10,8,6,withanareaof
24squareinches.

Thus,therearetwopossibleanswers.Therewereeither42or36matchesin
thematchbox.

Alsoitisinterestingtoknowthattherearejust5suchtrianglesforwhichthe
perimeterandtheareaisthesame(assumingallsidesareintegers)andthey
are:
1. 24
(10,8,6)
2. 30
(13,12,5)
3. 36
(17,10,9)

4.
5.

42
(20,15,7)
60
(29,25,6)

Findthevaluesofeachofthealphabets.

NOON

SOON

+MOON

JUNE
Answer

Usingtrialanderror.Thereare2solutionstoitandmaybemore.

2442

1442

+5442

9326

4114

5114

+0114

9342


Wehavetofillnumberfrom1to12attheintersectionpointoftwoormore
lines.Wehavetoconstructastarusingtwotriangle.Thesumofallnumber
lyinginstraightlinesshouldbesame.Thiscanbeeasiltyunderstoodbythe
fig.andhencesolved.
Submittedby:VaibhavGupta
Submit
Answer

Users
Answer(3)

BrainVista
Answer

PuzzleA
Friend

Addto
Favourite

Wehaveoneanswerwheresumofallthenumberslyinginstraightlinesis
26.

Ifyouhaveothers,dosubmitthem.

BrainTeaserNo:00355

Montu,Bantu,ChantuandPintuhavepets.

Montusays,"IfPintuandIeachhaveadog,thenexactlyoneofBantu
andChantuhasadog."

Bantusays,"IfChantuandIeachhaveacat,thenexactlyoneofMontu
andPintuhasadog."

Chantusays,"IfMontuandIeachhaveadog,thenexactlyoneofBantu

andPintuhasacat."

Pintusays,"IfBantuandIeachhaveacat,thenexactlyoneofBantuand
Ihasadog."

Onlyoneofthefouristellingthetruth.Whoistellingthetruth?

Answer

Bantuistellingthetruth.

ForaIFTHENstatementtobefalse,IFparthastobetrueandTHEN
parthastobefalse.

Sinceonlyonestatementistrueandremainingthreearefalse,IFpartof
threestatementsaretrue&THENpartofonestatementistrue.Let'sputthe
giveninformationintable.ThepetnameinthenormaltextrepresentstheIF
partandthepetnameinroundbracketsrepresentstheTHENpart.

Montu Bantu Chantu Pintu

Montusays

Dog

(Dog)

(Dog)

Dog

Bantusays

(Dog)

Cat

Cat

(Dog)

Chantusays

Dog

(Cat)

Dog

(Cat)

Cat
(Dog)

Cat
(Dog)

Pintusays

ItisclearthattheIFpartofthestatementsmadebyMontu,Chantuand
Pintuaretrueastheydonotcontradicteachother.AndtheIFpartofthe
statementmadebyBantuisfalse.

Thus,Bantuistellingthetruth.

MontuhaveaDogandmayormaynothaveaCat.
BantuhaveaCat.
ChantuhaveaDog.

PintuhaveaDogandaCat.

BrainTeaserNo:00520

Somebodymarkedthesixfacesofadiewiththenumbers1,2and3
eachnumbertwice.Thediewasputonatable.FourpeopleAbu,Babu,
CaluandDabusataroundthetablesothateachonewasabletoseeonly
threesidesofthedieataglance.
Abuseesthenumber1andtwoevennumbers.
BabuandCalucanseethreedifferentnumberseach.
Dabuseesnumber2twiceandhecan'trememberthethirdnumber.
Whatnumberisfacedownonthetable?

Answer

Number3isfacedownonthetable.

IfAbucanseetwoevennumbersi.e.number2twice,andifDabucansee
number2twice,thennumber2mustbefacingup.

Noweverythingelseissimple.(seethefollowingdiagram)

DabuAbu

322

CaluBabu

Thus,thenumberhiddenfromtheviewisnumber3andhencetheanswer.
TwoidenticalpackofcardsAandBareshuffledthroughly.Onecardis
pickedfromAandshuffledwithB.ThetopcardfrompackAisturnedup.
IfthisistheQueenofHearts,whatarethechancesthatthetopcardinBwill
betheKingofHearts?
Answer

52/2703

Therearetwocasestobeconsidered.

CASE1:
KingofHeartsisdrawnfromPackAandshuffledwithPackB

ProbabilityofdrawingKingofHeartsfromPackA=1/51(asQueenof
Heartsisnottobedrawn)
ProbabilityofhavingKingofHeartsonthetopofthePackB=2/53

Sototalprobabilityofcase1=(1/51)*(2/53)=2/(51*53)

CASE2:
KingofHeartsisnotdrawnfromPackA

ProbabilityofnotdrawingKingofHeartsfromPackA=50/51(asQueenof
Heartsisnottobedrawn)
ProbabilityofhavingKingofHeartsonthetopofthePackB=1/53

Sototalprobabilityofcase2=(50/51)*(1/53)=50/(51*53)

Nowaddingboththeprobability,therequiredprobabilityis
=2/(51*53)+50/(51*53)
=52/(51*53)
=52/2703
=0.0192378
Howmanypossiblecombinationsarethereina3x3x3rubicscube?

Inotherwords,ifyouwantedtosolvetherubicscubebytryingdifferent
combinations,howmanymightittakeyou(worstcasesenerio)?

Howmanyfora4x4x4cube?
Submitted

Answer

Thereare4.3252*10^19possiblecombinationsfor3x3x3Rubicsand
7.4012*10^45possiblecombinationsfor4x4x4Rubics.

Let'sconsider3x3x3Rubicsfirst.

Thereare8cornercubes,whichcanbearrangedin8!ways.
Eachofthese8cubescanbeturnedin3differentdirections,sothereare3^8
orientationsaltogether.Butifyougetallbutoneofthecornercubeinto
chosenpositionsandorientations,onlyoneof3orientationsofthefinal
cornercubeispossible.Thus,totalwayscornercubescanbeplaced=(8!)*
(3^8)/8=(8!)*(3^7)

Similarly,12edgecubescanbearrangedin12!ways.
Eachofthese12cubescanbeturnedin2differentdirections,sothereare
2^12orientationsaltogether.Butifyougetallbutoneoftheedgecubeinto
chosenpositionsandorientations,onlyoneof2orientationsofthefinaledge
cubeispossible.Thus,totalwaysedgecubescanbeplaced=(12!)*
(2^12)/2=(12!)*(2^11)

Here,wehaveessentiallypulledthecubesapartandstuckcubesbackin
placewhereverweplease.Inreality,wecanonlymovecubesaroundby
turningthefacesofthecubes.Itturnsoutthatyoucan'tturnthefacesin
suchawayastoswitchthepositionsoftwocubeswhilereturningallthe
otherstotheiroriginalpositions.Thusifyougetallbuttwocubesinplace,
thereisonlyoneattainablechoiceforthem(not2!).Hence,wemustdivide
by2.

Totaldifferentpossiblecombinationsare
=[(8!)*(3^7)]*[(12!)*(2^11)]/2
=(8!)*(3^7)*(12!)*(2^10)
=4.3252*10^19

Similarly,for4x4x4Rubicstotaldifferentpossiblecombinationsare
=[(8!)*(3^7)]*[(24!)]*[(24!)/(4!^6)]/24
=7.4011968*10^45


Notethatthereare24edgecubes,whichyoucannotturnin2orientations
(henceno2^24/2).Also,thereare4centercubesperfacei.e.(24!)/(4!^6).
Youcanswitch2cubeswithoutaffectingtherestofthecombinationas
4*4*4hasevendimensions(hencenodivisionby2).Butpatternononeside
isrotatedin4directionsover6faces,hencedivideby24.

BrainTeaserNo:00528

Substitutedigitsfortheletterstomakethefollowingrelationtrue.
NEVER

LEAVE

+ME

ALONE
Notethattheleftmostlettercan'tbezeroinanyword.Also,theremustbe
aonetoonemappingbetweendigitsandletters.e.g.ifyousubstitute3
fortheletterM,nootherlettercanbe3andallotherMinthepuzzlemust
be3.

Answer

Atoughone!!!

SinceR+E+E=10+E,itisclearthatR+E=10andneitherRnorEis
equalto0or5.Thisistheonlyentrypointto

solveit.Nowusetrialnerrormethod.

NEVER21419

LEAVE31541

+ME+61

ALONE53021
Thereare20peopleinyourapplicantpool,including5pairsofidentical
twins.

Ifyouhire5peoplerandomly,whatarethechancesyouwillhireatleast1
pairofidenticaltwins?(Needlesstosay,thiscouldcausetrouble))
Subm
Answer

Theprobabilitytohire5peoplewithatleast1pairofidenticaltwinsis
25.28%

5peoplefromthe20peoplecanbehiredin20C5=15504ways.

Now,divide20peopleintotwogroupsof10peopleeach:
G1withalltwins
G2withallpeopleotherthantwins

Let'sfindoutallpossiblewaystohire5peoplewithoutasinglepairof
indenticaltwins.
People
People
NoofwaystohireG1
Noof
Total
fromG1 fromG2
withoutasinglepairof
waysto
ways
indenticaltwins
hireG2
0

10C0

10C5

252

10C1

10C4

2100

10C2*8/9

10C3

4800

10C3*8/9*6/8

10C2

3600

10C4*8/9*6/8*4/7

10C1

800

10C5*8/9*6/8*4/7*2/6 10C0

32

Total

11584

Thus,totalpossiblewaystohire5peoplewithoutasinglepairofindentical
twins=11584ways

So,totalpossiblewaystohire5peoplewithatleastasinglepairof
indenticaltwins=1550411584=3920ways

Hence,theprobabilitytohire5peoplewithatleastasinglepairof
indenticaltwins
=3920/15504
=245/969
=0.2528
=25.28%
itted
VeerusaystoJay,"CanyoufigureouthowmanyEggsIhaveinmy
bucket?"Hegives3cluestoJay:IfthenumberofEggsIhave
1. isamultipleof5,itisanumberbetween1and19
2. isnotamultipleof8,itisanumberbetween20and29
3. isnotamultipleof10,itisanumberbetween30and39
HowmanyEggsdoesVeeruhaveinhisbucket?

Answer

32eggs

Let'sapplyall3conditionseparatelyandputallpossiblenumbers
together.

Firstconditionsaysthatifmultipleof5,thenthenumberisbetween1and
19.Hence,thepossiblenumbersare(5,10,15,21,22,23,24,25,26,27,
28,29,30,31,32,33,34,35,36,37,38,39)

Secondconditionsaysthatifnotamultipleof8,thenthenumberis
between20and29.Hence,thepossiblenumbersare(8,16,20,21,22,
23,25,26,27,28,29,32)

Thirdconditionsaysthatifnotamultipleof10,thenthenumberis
between30and39.Hence,thepossiblenumbersare(10,20,31,32,33,
34,35,36,37,38,39)

Onlynumber32isthereinall3resultsets.Thatmeansthatonlynumber
32satisfiesallthreeconditions.Hence,Veeruhave32eggsinhisbucket.

Submit

Answer

Users
Answer(14)

BrainV

Mr.Black,Mr.WhiteandMr.GreywerechattingintheYahooconference.
Theywerewearingablacksuit,awhitesuitandagreysuit,notnecessarily
inthesameorder.

Mr.Greysentmessage,"Weallarewearingsuitthatareofthesamecoloras
ournamesbutnoneofusiswearingasuitthatisthesamecolorashis
name."

Onthatapersonwearingthewhitesuitreplied,"Whatdifferencedoesthat
make?"

Canyoutellwhatcolorsuiteachofthethreepersonshadon?
Answer

Mr.GreyiswearingBlacksuit.
Mr.WhiteiswearingGreysuit.
Mr.BlackiswearingWhitesuit.

Mr.Greymustnotbewearinggreysuitasthatisthesamecolourashis
name.Also,hewasnotwearingwhitesuitasthepersonwearingwhitesuit
respondedtohiscomment.SoMrGreymustbewearingablacksuit.

Similarly,Mr.Whitemustbewearingeitherblacksuitorgreysuit.ButMr.
Greyiswearingablacksuit.Hence,Mr.Whitemustbewearingagreysuit.

And,Mr.Blackmustbewearingwhitesuit.
Substitutenumbersfortheletterssothatthefollowingmathematical
expressionsarecorrect.
ABCDEFGHI

=IE=IE=IE

369

Notethatthesamenumbermustbeusedforthesameletterwheneverit
appears.
Answer

A=2,B=1,C=9,D=4,E=3,F=8,G=6,H=5,I=7

Let'sstartwithGHI=9*IE.NotethatIappearsonboththeside.Also,after
multiplyingIEby9theanswershouldhaveIattheunit'splace.Thepossible
valuesofIEare19,28,37,46,55,64,73,82and91outofwhichonly64,
73and82satisfiesthecondition.(asallalphabetshouldrepresentdifferent
digits)

Now,considerDEF=6*IE.Outofthreeshortlistedvalues,only73
satisfiestheequation.Also,ABC=3*IEissatisfiedby73.

Hence,A=2,B=1,C=9,D=4,E=3,F=8,G=6,H=5,I=7
219438657

=73=73=73

369

BrainTeaserNo:00374

A,B,CandDarerelatedtoeachother.
Oneofthefouristheoppositesexfromeachoftheotherthree.
DisA'sbrotheroronlydaughter.
AorBisC'sonlyson.
BorCisD'ssister.
Answer

A,B&DaremalesCisfemale.BisC'sonlyson.A&DareC's
brothers.
A(male)C(female)D(male)


B(male)
Workoutwhichrelationcanholdanddiscardthecontradictoryoptions.

From(2)and(4),Dcannotbeaonlydaughterandhaveasister(BorC).
Hence,DisA'sbrotheri.e.DisaMale.

From(4),let'ssaythatBisD'ssisteri.e.BisFemale.
From(3),AisC'sonlysoni.e.AisMale.
ButDisA'sbrotherwhichmeansthatAisnotC'sonlyson.Hence,our
assumptionwaswrong.

Thus,CisD'ssisteri.e.CisFemale.AndBmustbeC'sonlyson.

NowitisclearthatD&BareMalesandCisFemale.AmustbeaMaleas
onlyoneofthemisofoppositesexfromeachoftheotherthree.AndheisC
&D'sbrother.Howaretheyrelatedtoeachother?

Dr.DoLittlealwaysgoeswalkingtotheclinicandtakesthesametimewhile
goingandwhilecomingback.Onedayhenoticedsomething.

Whenheleftthehome,thehourhandandtheminutehandwereexactly
oppositetoeachotherandwhenhereachedtheclinic,theyweretogether.

Similarly,whenhelefttheclinic,thehourhandandtheminutehandwere
togetherandwhenhereachedthehome,theywereexactlyoppositetoeach
other.

HowmuchtimedoesDr.DoLittletaketoreachhomefromtheclinic?Give
theminimalpossibleanswer.
Answer

32minutes43.6seconds

Intwelvehours,theminutehandandthehourhandaretogetherfor11
times.Itmeansthatafterevery12/11hours,boththehandsaretogether.

Similarlyintwelvehours,theminutehandandthehourhandareexactly

oppositetoeachotherfor11times.Itmeansthatafterevery12/11hours,
boththehandsareopposite.

Now,let'stakeanexample.Weknowthatat12boththehandsaretogether
andat6boththehandsareexactlyoppositetoeachother.

After6,boththehandsareinoppositionat[6+(12/11)]hours,[6+2*(12/11)]
hours,[6+3*(12/11)]hoursandsoon.Thesixthsuchtimeis[6+6*(12/11)]
hourswhichisthefirsttimeafter12.Thusafter12,boththehandsare
oppositetoeachotherat12:32:43.6

Hence,Dr.DoLittletakes32minutesand43.6secondstoreachhomefrom
theclinic.

SlowRunExpressrunsbetweenBangaloreandMumbai,Fortheupaswell
asthedownjourney,thetrainleavesthestartingstationat10:00PM
everydayandreachesthedestinationat11:30PMafterthreedays.

Mr.HaanioncetravelledbySlowRunExpressfromMumbaitoBangalore.
HowmanySlowRunExpressdidhecrossduringhisjourney?
Answer

Mr.Haanicrossed7SlowRunExpressesduringhisjourney.

Let'ssaythatMr.HaanitravelledbySlowRunExpressonWednesday
10:00PMfromMumbai.Thefirsttrainhewouldhavecrossedistheone
scheduledtoarriveatMumbaiat11:30PMthesamedayi.e.theonethatleft
Bangaloreat10:00PMonlastSunday.

Also,hewouldhavecrossedthelasttrainjustbeforereachingBangaloreon
Saturday.

Thus,Mr.Haanimusthavecrossed7SlowRunExpressesduringhis
journey.
Sixcabinsnumbered16consecutively,arearrangedinarowandare
separatedbythindividers.Thesecabinsmustbeassignedtosixstaff
membersbasedonfollowingfacts.

1.
2.
3.
4.
5.

MissShalaka'sworkrequireshertospeakonthephonefrequently
throughouttheday.
MissShudhapreferscabinnumber5as5isherluckynumber.
Mr.ShaanandMr.Sharmaoftentalktoeachotherduringtheirwork
andpreferstohaveadjacentcabins.
Mr.Sinha,Mr.ShaanandMr.Solankiallsmoke.MissShudhais
allergictosmokeandmusthavenonsmokersadjacenttoher.
Mr.Solankineedssilenceduringwork.

Canyoutellthecabinnumbersofeachofthem?
Answer

Thecabinsfromlefttoright(16)areofMr.Solanki,Mr.Sinha,Mr.
Shaan,Mr.Sharma,MissShudhaandMissShalaka.

From(2),cabinnumber5isassignedtoMissShudha.

AsMissShudhaisallergictosmokeandMr.Sinha,Mr.Shaan&Mr.
Solankiallsmoke,theymustbeincabinnumbers1,2and3notnecessarily
inthesameorder.Also,MissShalakaandMr.Sharmamustbeincabin4
and6.

From(3),Mr.Shaanmustbeincabin3andMr.Sharmamustbeincabin4.
Thus,MissShalakaisincabin6.

AsMr.SolankineedssilenceduringworkandMr.Shaanisincabin3who
oftentalkstoMr.Sharmaduringwork,Mr.Solankimustbeincabin1.
Hence,Mr.Sinhaisincabin2.

Thus,thecabinsnumbersare
1#Mr.Solanki,
2#Mr.Sinha,
3#Mr.Shaan,
4#Mr.Sharma,
5#MissShudha,
6#MissShalaka
SkyFicityisservedby6subwaylinesA,E,I,O,UandZ.
Whenitsnows,morningserviceonlineEisdelayed.
Whenitrainsorsnows,serviceonthelinesA,UandZisdelayed

bothmorningandafternoon.
Whenthetemperaturedropsbelow20C,afternoonserviceis
cancelledoneitherlineAorlineO,butnotboth.
Whenthetemperaturerisesabove40C,afternoonserviceiscancelled
oneitherlineIorlineZ,butnotboth.
WhenserviceonlineAisdelayedorcancelled,serviceonlineIis
alsodelayed.
WhenserviceonlineZisdelayedorcancelled,serviceonlineEis
alsodelayed.

OnFebruary10,itsnowsalldaywiththetemperatureat18C.Onhowmany
linesservicewillbedelayedorcancelled,includingbothmorningand
afternoon?
SkyFicityisservedby6subwaylinesA,E,I,O,UandZ.
Whenitsnows,morningserviceonlineEisdelayed.
Whenitrainsorsnows,serviceonthelinesA,UandZisdelayed
bothmorningandafternoon.
Whenthetemperaturedropsbelow20C,afternoonserviceis
cancelledoneitherlineAorlineO,butnotboth.
Whenthetemperaturerisesabove40C,afternoonserviceiscancelled
oneitherlineIorlineZ,butnotboth.
WhenserviceonlineAisdelayedorcancelled,serviceonlineIis
alsodelayed.
WhenserviceonlineZisdelayedorcancelled,serviceonlineEis
alsodelayed.
OnFebruary10,itsnowsalldaywiththetemperatureat18C.Onhowmany
linesservicewillbedelayedorcancelled,includingbothmorningand
afternoon?
Inacertaingame,if2wixsomesareworth3changs,and4changsareworth
1plut,then6plutesareworthhowmanywixsomes?

Answer

Itisgiventhat
2wixsomes=3changs
8wixsomes=12changs(I)

Also,giventhat
4changs=1plut

12changs=3plutes
8wixsomes=3plutesFrom(I)

Therefore,
6plutes=16wixsomes

Inacertainyear,thenumberofgirlswhograduatedfromCityHighSchool
wastwicethenumberofboys.If3/4ofthegirlsand5/6oftheboyswentto
collegeimmediatelyaftergraduation,whatfractionofthegraduatesthat
yearwenttocollegeimmediatelyaftergraduation?

Answer

AssumethatnumberofboysgraduatedfromCityHighSchool=B
Therefore,numberofgirlsgraduatedfromCityHighSchool=2*B

Itisgiventhat3/4ofthegirlsand5/6oftheboyswenttocollege
immediatelyaftergraduation.
Hence,totalstudentswenttocollege
=(3/4)(2*B)+(5/6)(B)
=B*(3/2+5/6)
=(7/3)B

Fractionofthegraduatesthatyearwenttocollegeimmediatelyafter
graduation
=[(7/3)B]/[3*B]
=7/9

Therefore,theansweris7/9

Amuleandadonkeywerecarryingfullsacksontheirbacks.

Themulestartedcomplainingthathisloadwastooheavy.Thedonkeysaid
tohim"Whyareyoucomplaining?IfyougavemeoneofyoursacksI'd
havedoublewhatyouhaveandifIgiveyouoneofmysackswe'dhavean
evenamount."

Howmanysackswereeachofthemcarrying?Givetheminimalpossible
answer.

Submitt
Answer

Themulewascarrying5sacksandthedonkeywascarrying7sacks.

Let'sassumethatthemulewascarryingMsacksandthedonkeywas
carryingDsacks.

Asthedonkeytoldthemule,"IfyougavemeoneofyoursacksI'dhave
doublewhatyouhave."
D+1=2*(M1)
D+1=2M2
D=2M3

Thedonkeyalsosaid,"IfIgiveyouoneofmysackswe'dhaveaneven
amount."
D1=M+1
D=M+2

Comparingboththeequations,
2M3=M+2
M=5

SubstitutingM=5inanyofaboveequation,wegetD=7

Hence,themulewascarrying5sacksandthedonkeywascarrying7sacks.
ed
Twopeopleenteraraceinwhickyouruntoapointandback.PersonA
runs20mphtoandfromthepoint.PersonBrunstothepointgoing10mph
and30mphgoingback.

Whocameinfirst?
Submitted
Answer

PersonAcameinfirst.

Let'sassumethatthedistancebetweenstartandthepointisDmiles.

TotaltimetakenbyPersonAtofinish
=(D/20)+(D/20)

=D/10
=0.1D

TotaltimetakenbyPersonBtofinish
=(D/10)+(D/30)
=2D/15
=0.1333D

Thus,PersonAistheWinner.

Alternatively(ifyoudon'tlikemathematics)),analysethesituationas
follow:

NotethatinitiallyspeedofPersonA(20mph)wastwicethespeedofPerson
B(10mph).Hence,whenPersonA(20mphforward)reachedthepoint,
PersonB(10mphforward)washalfway.WhenPersonA(20mphback)
finished,PersonB(still10mphforward)reachedthepoint.

Thus,PersonAwinstheraceandbythattimePersonBcoversonlyhalfthe
distance,nomatterhowfarthepointis!!!
MarkatehalfofapizzaonMonday.HeatehalfofwhatwasleftonTuesday
andsoon.Hefollowedthispatternforoneweek.

Howmuchofthepizzawouldhehaveeatenduringtheweek?
Submitted
Answer

Markwouldhaveate127/128(99.22%)ofthepizzaduringtheweek.

MarkatehalfthepizzaonMonday.OnTuesday,hewouldhaveatehalfof
theremainingpizzai.e.1/4oftheoriginalpizza.Similarly,hewouldhave
ate1/8oftheoriginalpizzaonWednesdayandsoonforthesevendays.

TotalpizzaMarkateduringtheweekis
=1/2+1/4+1/8+1/16+1/32+1/64+1/128
=127/128
=99.22%oftheoriginalpizza

IntheGeneralmeetingof"FriendsClub",Sameersaid,"Therepairstothe
ClubwillcometoatotalofRs3120andIproposethatthisamountshould
bemetbythemembers,eachpayinganequalamount."

Theproposalwasimmediatelyagreed.However,fourmembersoftheClub
chosetoresign,leavingtheremainingmemberstopayanextraRs26each.

HowmanymembersdidtheCluboriginallyhave?
Answer

TheCluboriginallyhad24members.

AssumethattherewereinitiallyNmembers.

As4membersresignedandremainingmemberspaidRs26each,itmeans
thattotalamountof4membersisequaltoRs26eachfromremaining(N4)
members.Thus,

4*(3120/N)=26*(N4)
2
12480=26N
104N
2
26N
104N12480=0

SolvingthequadraticequationwegetN=24.

Hence,theCluboriginallyhad24members.

BrainTeaserNo:00206

AtankcanbefilledbypipeAin30minutesandbypipeBin24minutes.
OutletpipeCcanemptythefulltankinonehourandtwentyminutes.

IfthetankisemptyinitiallyandifallthethreepipesA,BandCare
openedsimultaneously,inhowmuchtimewillthetankbefull?

Answer

Thetankwillbefullin16minutes.

Inoneminute,

pipeAcanfill1/30partofthetank.
pipeBcanfill1/24partofthetank.
pipeCcanempty1/80partofthetank.

Thus,thenetwaterlevelinoneminuteis
=1/30+1/241/80
=15/240partofthetank

Hence,thetankwillbefullin240/15i.e.16minutes.

AricholdArabhasthreesons.Whenhedied,hewilledhis17camelstothe
sons,tobedividedasfollows:

FirstSontoget1/2ofthecamelsSecondSontoget1/3rdofthecamels
ThirdSontoget1/9thofthecamels.

Thesonsaresittingtheretryingtofigureouthowthiscanpossiblybedone,
whenaveryoldwisemangoesridingby.Theystophimandaskhimtohelp
themsolvetheirproblem.Withouthesitationhedividesthecamelsproperly
andcontinuesridingonhisway.

Howdidhedoit?
Answer

Theoldmantemporarilyaddedhiscameltothe17,makingatotalof18
camels.

Firstsongot1/2ofit=9

Secondsongot1/3ofit=6

Thirdsongot1/9ofit=2

Foratotalof17.Hethentakeshiscamelbackandridesaway......

Thereweretwomenstandingonastreet.Theonesaystotheother,"Ihave
3daughters,theproductoftheiragesis36.WhatistheageoftheOLDEST
daughter?"


Thesecondguysays,"Ineedmoreinformation."So,thefirstguysays,"The
sumoftheiragesisequaltotheaddressofthehouseacrossthestreet."

Thesecondguylooksattheaddressandsays,"Istillneedmore
information."So,thefirstguysays,"Myoldestdaughterwearsareddress."

Answer

Theansweris9years.

Firstyouneedtofindallthepossiblesetsofthreenumbersthatwhen
multipliedequals36:

1136
1218
1312
149
166
229
236
334

Thenyouaddthenumberstogethertofindthesum
1136=38
1218=21
1312=16
149=14
166=13
229=13
236=11
334=10

Eventhoughwedon'tknowtheaddresstheguyknowsit.Forhimtoneed
moreinformationthatmeansthatatleasttwoofthesetsofnumbershas
thesamesum.Twoofthemdo,166and229.

WhenthefirstguysaidthathisOLDESTdaugherwearsareddressthat

meantthattherehadtobetheoldest.So166can'tpossiblybethe
answer.Sothepossiblepossiblityis229andtheOLDESTdaughteris9
yearsold.

Therefore,theansweris9.

Submit
User
Answer
Thereare3coloredboxesRed,GreenandBlue.Eachboxcontains2
envelopes.EachenvelopecontainsmoneytwoofthemcontainRs.25000
each,twoofthemcontainRs.15000eachandremainingtwocontainRs.
10000each.

Thereisonestatementwrittenonthecoverofeachbox.
*RedBox:Both,aredboxandablueboxcontainRs.10000each.
*GreenBox:Both,agreenboxandaredboxcontainRs.25000each.
*BlueBox:Both,ablueboxandagreenboxcontainRs.15000each.

Onlyoneoftheabove3statementsistrueandthecorrespondingbox
containsthemaximumamount.

Canyoutellwhichboxcontainsthemaximumamountandhowmuch?
Answer

BlueboxcontainsthemaximumamountRs.40000

Asitisgiventhatonlyoneofthegiven3statementsistrueassumeinturn,
eachstatementtobetrue&theother2falseandcheckwhetherthe
correspondingboxcontainsthemaximumamount.

Let'sassumethatthestatementontheBlueboxistrue.Thus,thegiven3
statementscanbeinterpretedas
*Atmostone,aredboxorablueboxcontainsRs.10000.
*Atmostone,agreenboxoraredboxcontainsRs.25000.
*Both,ablueboxandagreenboxcontainRs.15000each.

Goingthroughallpossiblecombinations,wecanconcludethat
RedBox:Rs.10000+Rs.25000=Rs.35000
GreenBox:Rs.10000+Rs.15000=Rs.25000
BlueBox:Rs.15000+Rs.25000=Rs.40000

Youcantestoutforothertwostatementsi.e.assumingRedboxstatement
trueandthenGreenboxstatementtrue.Inboththecases,otherstatements
willcontradictthetruestatement.

Sachin,DravidandGangulyplayedinaCricketmatchbetweenIndiaand
England.
Noneofthemscoredmorethan99runs.
IfyouaddthedigitsoftherunsscoredbySachintohisownscore,
youwillgettherunsscoredbyDravid.
IfyoureversethedigitsoftherunsscoredbyDravid,youwillgetthe
runsscoredbyGanguly.
Thetotalrunsscoredbythemis240.
Canyoufigureouttheirindividualscores?
Answer

Sachin,DravidandGangulyscored75,87and78respectively.

Sachin'sscoremustbelessthan86,otherwiseDravid'sscorewouldbemore
than99.Also,hemusthavescoredatleast42incaseDravidandGanguly
scored99each.

Also,asnoneofthemscoredmorethan99andthetotalrunsscoredbythem
is240theirindividualscoresmustbearound80.

Now,usetrialnerrormethodtosolvetheteaser.
Threemen,includingGianniandthreewoman,includingSachiareinlineat
theBrentWoodpostoffice.Eachhastwodifferentpiecesofbusinessto
conduct.
1. Thefirstpersonisawoman.
2. Carloswantstosendanovernightpackage.
3. LauisjustaheadofPimentelliwhoisthesamesexasLau.
4. Gianniistwoplacesaheadofthepersonwhowantstobuystamps.
5. KnutsonwhoistheoppositesexthanRendlerisn'tthepersonwho

6.

7.

8.
9.
10.

11.
12.
13.

wantedtocomplainaboutamailcarrier.
Thesixpeople,notnecessarilyinthesameorderareAnthony,
Donna,thepersonwhowantstofilloutachangeofaddressform,the
onewhowantstobuyamoneyorder,theonewhowantstosend
AirmailtoTibetandthesecondpersonintheline.
Thefourtasksofthelasttwopeopleinline,notnecessarilyinthe
sameorderaresendingbooksfourthclass,buyingamoneyorder,
pickingupapackageandcomplainingaboutamailcarrier.
Thepersonwhowantstosendbooksfourthclassisjustbehinda
personofthesamesex.
Maryisjustbehindapersonwhowantstosendaninsuredpackage.
ThepersonwhowantstosendAirmailtoTibetiseithertwoplaces
aheadofortwoplacesbehindtheonewhowantstoaddpostagetohis
orhermeter.
Anthonyisn'ttwoplacesbehindthewhowantstopickuparegistered
letter.
Torisezaistwoplacesaheadofthepersonwhowantstopickupa
package.
Knutsonisn'tjustaheadofthepersonwhowantstosendanitem
parcelpost.

Canyoufigureoutwhereeachcustomerisintheline,hisorherfullname
(onesurnameisLoti)andthetwothingsheorshewantstoaccomplish?
ProvideyouranswerisPOSITIONFIRSTNAMELASTNAME
BUSINESSformat.
Submit
Answer

Users
Answer(8)

Answer

AveryTOUGHpuzzle!!!
POS

FIRST
NAME

LAST
NAME

BUSINESS

Sachi

Loti

FillOutaChangeofAddressForm
AddPostagetoMeter

Gianni

Lau

PickUpaRegisteredLetter

SendanItemParcelPost
3

Carlos

Pimentelli

OvernightPackage
SendAirmailtoTibet

Donna

Toriseza

BuyStamps
SendanInsuredPackage

Mary

Knutson

BuyaMoneyOrder
SendBooksfourthClass

Anthony

Rendler

ComplainAboutaMailCarrier
PickUpaPackage

BrainTeaserNo:00164

Substitutedigitsfortheletterstomakethefollowingrelationtrue.
WORLD

+TRADE

CENTER
Notethattheleftmostlettercan'tbezeroinanyword.Also,theremustbe
aonetoonemappingbetweendigitsandletters.e.g.ifyousubstitute3
fortheletterW,nootherlettercanbe3andallotherWinthepuzzle
mustbe3.

Answer

Atoughone.

ItisobviousthatC=1.Also,themaximumpossiblevalueofEis7.Now,
startputtingpossiblevaluesofD,EandRastheyoccurefrequentlyanduse
trialnerror.

WORLD53684

+TRADE+76042

CENTER129726

BrainTeaserNo:00107

Ifyoulookataclockandthetimeis3:15.

Whatistheanglebetweenthehourandtheminutehands?(Theanswer
tothisisnotzero!)

Answer

7.5degrees

At3:15minutehandwillbeperfactlyhorizontalpointingtowards3.
Whereashourhandwillbetowards4.Also,hourhandmusthavecovered
1/4ofanglebetween3and4.

Theanglebetweentwoadjacentdigitsis360/12=30degrees.

Hence1/4ofitis7.5degrees.
Anapplevendorhas1000applesand10emptyboxes.Heaskshissonto
placeallthe1000applesinallthe10boxesinsuchamannerthatifheasks
foranynumberofapplesfrom1to1000,hissonshouldbeabletopickthem
intermsofboxes.

Howdidthesonplacealltheapplesamongthe10boxes,giventhatany
numberofapplescanbeputinonebox.
Answer

1,2,4,8,16,32,64,128,256,489

Let'sstartfromscratch.
Theapplevandorcanaskforonly1apple,sooneboxmustcontain1
apple.
Hecanaskfor2apples,sooneboxmustcontain2apples.

Hecanaskfor3apples,inthatcaseboxoneandboxtwowilladdup
to3.
Hecanaskfor4apples,sooneboxi.e.thirdboxmustcontain4
apples.
Nowusingboxnumberone,twoandthreecontaining1,2and4
applesrespectively,hissoncangiveupto7apples.Hence,forthbox
mustcontain8apples.
Similarly,usingfirstfourboxescontaining1,2,4and8apples,his
soncangiveupto15apples.Hencefifthboxmustcontain16apples.
Youmusthavenoticedonethingtillnowthateachboxtillnow
containspowerof2apples.Hencetheansweris1,2,4,8,16,32,64,
128,256,489.Thisistrueforanynumberofapples,hereinourcase
onlyupto1000.

BrainTeaserNo:00261

ThelettersP,Q,R,S,T,UandV,notnecessarilyinthatorderrepresents
sevenconsecutiveintegersfrom22to33.
UisasmuchlessthanQasRisgreaterthanS.
VisgreaterthanU.
Qisthemiddleterm.
Pis3greaterthanS.
Canyoufindthesequenceoflettersfromthelowestvaluetothehighest
value?

Answer

Thesequenceoflettersfromthelowestvaluetothehighestvalueis
TUSQRPV.

From(3),Qisthemiddleterm.
__________Q__________

From(4),theremustbeexactly2numbersbetweenPandSwhichgivestwo
possiblepositions.

[1]____S_____Q__P_______

[2]_______S__Q_____P____

From(1),thenumberoflettersbetweenUandQmustbesameasthe
numberoflettersbetweenSandR.Also,thenumberoflettersbetweenthem
canbe1,2or3.

Usingtrialanderror,itcanbefoundthattheremustbe2lettersbetween
them.Also,itispossibleonlyinoption[2]above.

[2]____U__S__Q__R__P____

From(2)VmustbethehighestandtheremainingTmustbethelowest
number.

_T__U__S__Q__R__P__V_

Thus,thesequenceoflettersfromthelowestvaluetothehighestvalueis
TUSQRPV.
Acontractorhademployed100labourersforaflyoverconstructiontask.He
didnotallowanywomantoworkwithoutherhusband.Also,atleasthalfthe
menworkingcamewiththeirwives.

Hepaidfiverupeesperdaytoeachman,fourruppestoeachwomanand
onerupeetoeachchild.Hegaveout200rupeeseveryevening.

Howmanymen,womenandchildrenwereworkingwiththeconstructor?
Answer

16men,12womenand72childrenwereworkingwiththeconstructor.

Let'sassumethattherewereXmen,YwomenandZchildrenworkingwith
theconstructor.Hence,

X+Y+Z=100
5X+4Y+Z=200

EliminatingXandYinturnfromtheseequations,weget
X=3Z200
Y=3004Z

Asifwomanworks,herhusbandalsoworksandatleasthalfthemen
workingcamewiththeirwivesthevalueofYliesbetweenXandX/2.
Substitutingtheselimitingvaluesinequations,weget

ifY=X,
3004Z=3Z200
7Z=500
Z=500/7i.e.71.428

ifY=X/2,
3004Z=(3Z200)/2
6008Z=3Z200
11Z=800
Z=800/11i.e.72.727

ButZmustbeaninteger,henceZ=72.Also,X=16andY=12

Therewere16men,12womenand72childrenworkingwiththe
constructor.

Becausecigarscannotbeentirelysmoked,aBobowhocollectscigarbutts
canmakeacigartosmokeoutofevery3buttsthathefinds.

Today,hehascollected27cigarbutts.Howmanycigarswillhebeableto
smoke?
Answer

13
not12

Hemakes9originalsfromthe27buttshefound,andafterhesmokesthem
hehas9buttsleftforanother3cigars.Andthenhehas3buttsforanother
cigar.

So9+3+1=13
Inasmalltown,therearethreetemplesinarowandawellinfrontofeach

temple.Apilgrimcametothetownwithcertainnumberofflowers.

Beforeenteringthefirsttemple,hewashedalltheflowershehadwiththe
waterofwell.Tohissurprise,flowersdoubled.Heofferedfewflowersto
theGodinthefirsttempleandmovedtothesecondtemple.Herealso,
beforeenteringthetemplehewashedtheremainingflowerswiththewater
ofwell.Andagainhisflowersdoubled.HeofferedfewflowerstotheGod
insecondtempleandmovedtothethirdtemple.Herealso,hisflowers
doubledafterwashingthemwithwater.HeofferedfewflowerstotheGod
inthirdtemple.

Therewerenoflowersleftwhenpilgrimcameoutofthirdtempleandhe
offeredsamenumberofflowerstotheGodinallthreetemples.

Whatistheminimumnumberofflowersthepilgrimhadinitially?How
manyflowerdidheoffertoeachGod?
Answer

Thepilgrimhad7flowers,initiallyandheoffered8flowerstoeach
God.

AssumethatthepilgrimhadXflowersinitiallyandheofferedYflowersto
eachGod.

Fromtheabovefigure,thereare(8X7Y)flowerswhenthepilgrimcame
outofthethirdtemple.Butitisgiventhattherewerenoflowersleftwhen
hecameoutofthirdtemple.Itmeansthat

(8X7Y)=0
8X=7Y

TheminimumvaluesofXandYare7and8respectivelytosatisfyabove
equation.Hence,thepilgrimhad7flowersandheoffered8flowerstoeach
God.

Ingeneral,thepilgrimhad7Nflowersinitiallyandheoffered8Nflowersto
eachGod,whereN=1,2,3,4,.....

BrainTeaserNo:00432

Tanyawantstogoonadateandprefersherdatetobetall,darkand
handsome.
1. Ofthepreferredtraitstall,darkandhandsomenotwoofAdam,
Bond,CruzandDumbohavethesamenumber.
2. OnlyAdamorDumboistallandfair.
3. OnlyBondorCruzisshortandhandsome.
4. AdamandCruzareeitherbothtallorbothshort.
5. BondandDumboareeitherbothdarkorbothfair.
WhoisTanya'sdate?

Answer

CruzisTanya'sdate.

Asnotwoofthemhavethesamenumberofpreferredtraitsfrom(1),
exactlyoneofthemhasnoneofthepreferredtraitsandexactlyoneofthem
hasallthepreferredtraits.

From(4)and(5),thereareonlytwopossibilities:
*Adam&CruzbotharetallandBond&Dumbobotharefair.
*Adam&CruzbothareshortandBond&Dumbobotharedark.

Butfrom(2),secondpossibilityisimpossible.Sothefirstoneisthecorrect
possibilityi.e.Adam&CruzbotharetallandBond&Dumbobotharefair.

Thenfrom(3),Bondisshortandhandsome.

Also,from(1)and(2),Adamistallandfair.Also,Dumboistheperson
withoutanypreferredtraits.CruzisDark.AdamandCruzarehandsome.
Thus,followingaretheindividualpreferredtraits:

CruzTall,DarkandHandsome
AdamTallandHandsome
BondHandsome
DumboNone:(

Hence,CruzisTanya'sdate.
ConsideragameofTowerofHanoi(liketheonethatyoucanplayon
BrainVista).

Ifthetowerhas2discs,theleastpossiblemoveswithwhichyoucanmove
theentiretowertoanotherpegis3.

Ifthetowerhas3discs,theleastpossiblemoveswithwhichyoucanmove
theentiretowertoanotherpegis7.

Whatistheleastpossiblemoveswithwhichyoucanmovetheentiretower
toanotherpegifthetowerhasNdiscs?
Submitted
Answer

Therearenumberofwaystofindtheanswer.

Tomovethelargestdisc(atlevelN)fromonetowertotheother,itrequires
(N1)
2
moves.Thus,tomoveNdiscsfromonetowertotheother,thenumber
ofmovesrequiredis
(N1)
(N2)
(N3)
2
1
0
=2
+2
+2
+.....+2
+2
+2

N
=2
1

ForNdiscs,thenumberofmovesisonemorethantwotimesthenumberof
movesforN1discs.Thus,therecursivefunctionis
F(1)=1
F(N)=2*[F(N1)]+1

whereNisthetotalnumberofdiscs

Also,onecanarriveattheanswerbyfindingthenumberofmovesfor
smallernumberofdiscsandthenderivethepattern.
For1disc,numberofmoves=1
For2discs,numberofmoves=3
For3discs,numberofmoves=7
For4discs,numberofmoves=15
For5discs,numberofmoves=31

N
Thus,thepatternis2
1
Aboyfoundthathehada48inchstripofpaper.Hecouldcutaninchoff
everysecond.

Howlongwouldittakeforhimtocut48pieces?Hecannotfoldthestrip
andalso,cannotstacktwoormorestripsandcutthemtogether.
Submi
Answer

47seconds.

Toget48pieces,theboyhavetoputonly47cuts.i.e.hecancut46pieces
in46seconds.Aftergetting46pieces,hewillhavea2incheslongpiece.He
cancutitintotwowithjustaonecutin1second.Hence,totalof47
seconds.
ttedby:Kimi

ThecricketmatchbetweenIndiaandPakistanwasover.
HarbhajanscoredmorerunsthanGanguly.
SachinscoredmorerunsthanLaxmanbutlessthanDravid
BadaniscoredasmuchrunsasAgarkarbutlessthanDravidandmore
thanSachin.
GangulyscoredmorerunsthaneitherAgarkarorDravid.
Eachbatsmanscored10runsmorethanhisimmediatebatsman.Thelowest
scorewas10runs.Howmuchdideachoneofthemscore
Answer

Asimpleone.Usethegivenfactsandputdownalltheplayersinorder.The
orderisasfollowwithHarbhajan,thehighestscorerandLaxman,thelowest

scorer.
1. Harbhajan
2. Ganguly
3. Dravid
4. Badani,Agarkar
5. Sachin
6. Laxman
Also,asthelowestscorewas10runs.Laxmanmusthavescored10,Sachin
20,Badani&Agarkar30andsoon.
1. Harbhajan60runs
2. Ganguly50runs
3. Dravid40runs
4. Badani,Agarkar30runseach
5. Sachin20runs
6. Laxman10runs
Thereare10statementswrittenonapieceofpaper:
1. Atleastoneofstatements9and10istrue.
2. Thiseitheristhefirsttrueorthefirstfalsestatement.
3. Therearethreeconsecutivestatements,whicharefalse.
4. Thedifferencebetweenthenumbersofthelasttrueandthefirsttrue
statementdividesthenumber,thatistobefound.
5. Thesumofthenumbersofthetruestatementsisthenumber,thatisto
befound.
6. Thisisnotthelasttruestatement.
7. Thenumberofeachtruestatementdividesthenumber,thatistobe
found.
8. Thenumberthatistobefoundisthepercentageoftruestatements.
9. Thenumberofdivisorsofthenumber,thatistobefound,(apartfrom
1anditself)isgreaterthanthesumofthenumbersofthetrue
statements.
10. Therearenothreeconsecutivetruestatements.
Findtheminimalpossiblenumber?
Submitted
Answer

Thenumebris420.

Ifstatement6isfalse,itcreatesaparadox.Hence,
Statement6mustbe
true
.

ConsiderStatement2:
Ifitistrue,itmustbethefirsttruestatement.Otherwise,itcreatesa
paradox.
Ifitisfalse,itmustbethesecondfalsestatement.Otherwise,it
createsaparadox.
Inboththecases,
Statement1isfalse
.

AsStatement1isfalse,
Statement9andStatement10botharefalse
i.e.
therearethreeconsecutivetruestatements.
1

2 3 4 5

7 8

10

False True False False

Let\'sassumethatStatement3isfalsei.e.therearenothreeconsecutive
falsestatements.ItmeansthatStatement2andStatement8mustbetrue,
elsetherewillbethreeconsecutivefalsestatements.
1

4 5

10

False True False True True False False

Also,atleasttwoofStatements4,5and7mustbetrueastherearethree
consecutivetruestatements.

AccordingtoStatement8,thenumberthatistobefoundisthepercentageof
truestatements.Hence,numberiseither50or60.NowifStatement7is
true,thenthenumberofeachtruestatementdividesthenumber,thatistobe
found.But7and8donotdivideeither50or60.Hence,Statement7isfalse
whichmeansthatStatement4and5aretrue.ButStatement5contradictsthe
Statement8.Hence,ourassumptionthatStatement3isfalseiswrongand
Statement3istrue
i.e.thereare3consecutivefalsestatementswhich
meansthat
Statement8isfalse
asthereisnootherpossibilitiesof3
consecutivefalsestatements.

Also,
Statement7istrue
asStatement6isnotthelasttruestatement.

4 5

10

False True True True False False False

AccordingtoStatement7,thenumberofeachtruestatementdividesthe
number,thatistobefound.AndaccordingtoStatement5,thesumofthe
numbersofthetruestatementsisthenumber,thatistobefound.Forall
possiblecombinations
Statement5isfalse
.

There3consecutivetruestatements.Hence,
Statement2andStatement4
aretrue
.
1

10

False True True True False True True False False False

Now,theconditionsforthenumbertobefoundare:
1. Thenumebrisdivisibleby5(Statement4)
2. Thenumebrisdivisibleby2,3,4,6,7(Statement7)
3. Thenumberofdivisorsofthenumber,thatistobefound,(apartfrom
1anditself)isnotgreaterthanthesumofthenumbersofthetrue
statements.(Statement9)
Theminimumpossiblenumberis420.

Thedivisorsof420,apartfrom1anditselfare2,3,4,5,6,7,10,12,14,15,
20,21,28,30,35,42,60,70,84,105,140,210.Therearetotalof22
divisors.Also,thesumofthenumbersofthetruestatementsis22
(2+3+4+6+7=22),whichsatisfiesthethirdcondition.
AnkitandTejasdividedabagofApplesbetweenthem.

Tejassaid,"It'snotfair!Youhave3timesasmanyApplesIhave."Ankit
said,"OK,IwillgiveyouoneAppleforeachyearofyourage."Tejas
replied,"Stillnotfair.Now,youhavetwiceasmanyApplesasIhave."
"Dear,that'sfairenoughasIamtwiceolderthanyou.",saidAnkit.

AnkitwenttoKitchentodrinkwater.WhileAnkitwasinKitchen,Tejas
tookapplesfromAnkit'spileequaltoAnkit'sage.

Whohavemoreapplesnow?

Answer

Attheend,AnkitandTejas,bothhavethesamenumberofapples.

Let'sassumethatinitiallyTejasgotNapplesandhisageisTyears.Hence,
initiallyAnkitgot3Napplesandhisageis2Tyears.
Operation

Ankit'sApples

Tejas'sApples

Initially

3N

AnkitgaveTapplestoTejas
(equalsageofTejas)

3NT

N+T

Tejastook2TapplesfromAnkit'spile
(equalsageofAnkit)

3N3T

N+3T

ItisgiventhatafterAnkitgaveTapplestoTejas,Ankithadtwiceasmany
applesasTejashad.
3NT=2*(N+T)
3NT=2N+2T
N=3T

Fromthetable,attheendAnkithave(3N3T)applesandTejashave(N+
3T)apples.SubstitutingN=3T,weget
Ankit'sapples=3N3T=9T3T=6T
Tejas'sapples=N+3T=3T+3T=6T

Thus,attheendAnkitandTejas,bothhavethesamenumberofapples.

OneveySundayAmar,AkbarandAnthonylunchtogetherat
PreetamDaDhabawheretheyorderlassibasedonfollowingfacts.
1. UnlessneitherAmarnorAkbarhavelassi,Anthonymusthaveit.
2. IfAmardoesnothavelassi,eitherAkbarorAnthonyorbothhaveit.
3. AnthonyhaslassionlyifeitherAmarorAkbarorbothhaveit.
4. AkbarandAnthonyneverhavelassitogether.
Whoorder(s)lassi?
Answer


AmarandAnthonybothhavelassiwhereasAkbarneverdoes.

Fact(2)canbealternativelystatedthat"eitherAmarorAkbarorAnthony
musthavelassi".

FromFact(3),itcanbeinferedthateitherAmarorAkbarmusthavelassi.

Now,fromFact(1),itisapparentthatAnthonytoomusthavelassi.But
accordingtoFact(4),AkbarcannothavelassiwhenAnthonydoes.

BrainTeaserNo:00191

Decipherthissentence.

BRWQHLFKWHJKQIBWK

QICEDWZBGWKKMIKE

ZBGQHSKZBGJKZKW

BUUZBGJDBHFW.

Answer

StartwithZBGandZBGJ.Itshouldbeeither"the/then"or"you/your"
combinationastheyappearmore.

BRWQHLFKWHJKQIBWK

obstaclesarethose

QICEDWZBGWKKMIKE

thingsyouseewhen

ZBGQHSKZBGJKZKW

youtakeyoureyes

BUUZBGJDBHFW.

offyourgoals.

BrainTeaserNo:00001

AtwhattimeimmediatelypriortoSixO'clockthehandsoftheclockare
exactlyoppositetoeachother.Givetheexacttimeinhours,minutesand
seconds.

Answer

Itisobviousthatbetween5O'clockand6O'clockthehandswillnotbe
exactlyoppositetoeachother.Itisalsoobviousthatthehandswillbe
oppositetoeachotherjustbefore5O'clock.Nowtofindexacttime:

Thehourhandmoves1degreeforevery12degreesthattheminutehand
moves.LetthehourhandbeXdegreeawayfrom5O'clock.Thereforethe
minutehandis12Xdegreeawayfrom12O'clock.

ThereforesolvingforX

Anglebetweenminutehandand12O'clock+Anglebetween12O'clockand
4O'clock+Anglebetween4O'clockandhourhand=180
12X+120+(30X)=180

11X=30
HenceX=30/11degrees
(hourhandisXdegreeawayfrom5O'clock)

Noweachdegreethehourhandmovesis2minutes.

Thereforeminutesare
=2*30/11
=60/11
=5.45(means5minutes27.16seconds)

Thereforetheexacttimeatwhichthehandsareoppositetoeachotheris
=4hrs.54min.32.74seconds

AliBabahadfoursons,towhomhebequeathedhis39camels,withthe
provisothatthelegacybedividedinthefollowingway:

Theoldestsonwastoreceiveonehalftheproperty,thenextaquarter,the
thirdaneighthandtheyoungestonetenth.Thefourbrotherswereataloss
ashowtodividetheinheritanceamongthemselveswithoutcuttingupa
camel,untilastrangerappeareduponthescene.
Dismountingfromhiscamel,heaskedifhemighthelp,forheknewjust
whattodo.Thebrothersgratefullyacceptedhisoffer.

AddinghisowncameltoAliBaba's39,hedividedthe40asperthewill.
Theoldestsonreceived20,thenext10,thethird5andtheyoungest4.One
camelremained:thiswashis,whichhemountedandrodeaway.

Scratchingtheirheadsinamazement,theystartedcalculating.Theoldest
thought:isnot20greaterthanthehalfof39?Someonemusthavereceived
lessthanhispropershare!Buteachbrotherdiscoveredthathehadreceived
morethanhisdue.Howisitpossible?
Answer

Theytooktheirpercentagesfrom40andnotfrom39,sotheygotmorethan
theirshare.

Theoldestsongot1/2of40=20whichis0.5more
Thesecondsongot1/4of40=10whichis0.25more

Thethirdsongot1/8of40=5whichis0.125more
Theyoungestsongot1/10of40=4whichis0.1more

Andthestrangergot1/40of40=1whichis0.025more(Asheisnot
supposedtogetanything)

Allthesefractionsaddto=0.5+0.25+0.125+0.1+0.025=1which
strangertookaway.
Thereisafamilypartyconsistingoftwofathers,twomothers,twosons,one
fatherinlaw,onemotherinlaw,onedaughterinlaw,onegrandfather,one
grandmotherandonegrandson.

Whatistheminimumnumberofpersonsrequiredsothatthisispossible?
Answer

Therearetotal2couplesandason.GrandfatherandGrandmother,theirson
andhiswifeandagaintheirson.Sototal5people.

Grandfather,Grandmother
|
|
Son,wife
|
|
Son

AmanwentintoafastfoodrestaurantandateamealcostingRs.105,giving
theaccountantaRs.500note.Hekeptthechange,camebackafewminutes
laterandhadsomefoodpackedforhisgirlfriend.Hegavetheaccountanta
Rs.100noteandreceivedRs.20inchange.Laterthebanktoldthe
accountantthatboththeRs.500andtheRs.100noteswerecounterfeit.

Howmuchmoneydidtherestaurantlose?Ignoretheprofitofthefood
restaurant.
Answer

HelostRs.600


FirsttimerestauranthasgivenfoodworthRs.105andRs.395change.
Similarlysecondtime,foodworthRs.80andRs.20change.Here,wearenot
consideringfoodrestaurantprofits.

SLIDE

DEAN

3651
Eachofsevendigitsfrom09arerepresentedbyadifferentletterabovesuch
thatthesubtractionistrue.

Whatwordrepresents3651?
Answer

3651representsLENS.

Let'sassignpossiblevaluestoeachletterandthenusetrialnerror.

Smustbe1.

ThenD(underL)mustbegreaterthan5.IfDis6,thenLis0.ButthenA
mustbe0or1whichisimpossible.Hence,thepossiblevaluesofDare7,8
or9.

NmustbeE+1.Also,DmustbeA+5asthepossiblevaluesofDare7,8
or9,Dcannotbe(10+A)+5.

Nowusingtrialnerror,wegetS=1,I=2,L=3,A=4,N=5,E=6andD=9

SLIDE13296

DEAN9645

3651LENS
Hence,3651representsLENS.
Adam,Burzin,ClarkandEdmundeachliveinanapartment.Their
apartmentsarearrangedinarownumbered1to4fromlefttoright.Also,
oneofthemisthelandlord.
1. IfClark'sapartmentisnotnexttoBurzin'sapartment,thenthe
landlordisAdamandlivesinapartment1.
2. IfAdam'sapartmentisrightofClark'sapartment,thenthelandlordis
Edmundandlivesinapartment4.
3. IfBurzin'sapartmentisnotnexttoEdmund'sapartment,thenthe
landlordisClarkandlivesinapartment3.
4. IfEdmund'sapartmentisrightofAdam'sapartment,thenthelandlord
isBurzinandlivesinapartment2.
Whoisthelandlord?

Answer

Clarkisthelandlord.

Assumeeachstatementtrue,oneatatimeandseethatnootherstatementis
contradicted.

Let'sassumethatStatement(1)istrue.Then,Adamisthelandlordandlives
inapartment1.Also,otherthree'sapartmentswillbeontherightofhis
apartmentwhichcontradictsStatement(4)i.e.IfEdmund'sapartmentis
rightofAdam'sapartment,thenthelandlordisBurzin.Thus,Adamisnot
thelandlord.

Let'sassumethatStatement(2)istrue.Then,Edmundisthelandlordand
livesinapartment4.Also,otherthree'sapartmentswillbeontheleftofhis
apartmentwhichagaincontradictsStatement(4)i.e.IfEdmund'sapartment
isrightofAdam'sapartment,thenthelandlordisBurzin.Thus,Edmundis
notthelandlordeither.

Let'sassumethatStatement(3)istrue.Then,Clarkisthelandlordandlives
inapartment3.Itsatisfiesallthestatementsfor
(1)Adam(2)Edmund(3)Clark(4)Burzin

Hence,Clarkisthelandlord.

Similarly,youcanassumeStatement(4)trueandfindoutthatitalso
contradicts.

BrainTeaserNo:00456

B,JandParerelatedtoeachother.
1. AmongthethreeareB'slegalspouse,J'ssiblingandP's
sisterinlaw.
2. B'slegalspouseandJ'ssiblingareofthesamesex.
Whoisthemarriedman?

Answer

Jisthemarriedman.

Notethataperson'ssisterinlawmaybethewifeofthatperson'sbrotheror
thesisterofthatperson'sspouse.

Thereare2cases:
1. IfB'slegalspouseisJ,thenJ'ssiblingmustbePandP'ssisterinlaw
mustbeB.
2. IfB'slegalspouseisP,thenP'ssisterinlawmustbeJandJ'ssibling
mustbeB.
ItisgiventhatB'slegalspouseandJ'ssiblingareofthesamesex.Also,itis
obviousthatP'ssisterinlawisfemale.Then,B'slegalspouseandJ'ssibling
bothmustbemales.

B'sspouseJ'ssiblingP'ssisterinlaw

(male)(male)(female)

CaseIJPB

CaseIIPBJ


CaseIIisnotpossibleasB&Paremarriedtoeachotherandbotharemale.
Hence,Jisthemarriedman.

BrainTeaserNo:00041

Apolygonhas1325diagonals.Howmanyverticesdoesithave?

Answer

Theformulatofindnumberofdiagonals(D)giventotalnumberofvertices
orsides(N)is

N*(N3)

D=

Usingtheformula,weget
1325*2=N*(N3)
2
N
3N2650=0

Solvingthequadraticequation,wegetN=53or50

Itisobviousthatansweris
53
asnumberofverticescannotbenegative.

Alternatively,youcanderivetheformulaastriangehas0diagonals,
quadrangelhas2,pentagonhas5,hexagonhas9andsoon......

Hencetheseriesis0,0,0,2,5,9,14,........(asdiagramwith1,2or3
verticeswillhave0diagonals).

Usingtheseriesonecanarrivetotheformulagivenabove.

BrainTeaserNo:00076

Acubeismadeofawhitematerial,buttheexteriorispaintedblack.

Ifthecubeiscutinto125smallercubesofexactlythesamesize,how

manyofthecubeswillhaveatleast2oftheirsidespaintedblack?
Answer

44

36ofthecubeshaveEXACTLY2oftheirsidespaintedblack,butbecausea
cubewith3ofitssidespaintedblackhas2ofitssidespaintedblack,you
mustalsoincludethecornercubes.Thiswasatrickquestion,buthopefully
thetitleofthepuzzletippedyouofftothis.

BrainTeaserNo:00238

Imagineatriangleofcoinsonatablesothatthefirstrowhasonecoinin
itandthesecondrowhastwocoinsinitandsoon.Ifyoucanonlymove
onecoinatatime,howmanymovesdoesittaketomakethetriangle
pointtheotherway?

Foratrianglewithtworowitisone,foratrianglewiththreerowsitis
two,foratrianglewithfourrowsitisthree.

Foratrainglewithfiverowsisitfour?
Submitted

Answer

Ittakes5movestomakethetrianglewith5rowspointtheotherway.

0=acointhathasnotbeenmoved.
X=theoldpositionofthemovedcoin
8=thenewpositionofthemovedcoin.

________X
_______XX
____80008
_____0000
____X000X
_______88
________8

Fortraingleofanynumberofrows,theoptimalnumberofmovescanbe
achievedbymovingtheverticallysymmetricalcoinsi.e.bymovingsame
numberofcoinsfrombottomleftandright,andremainingcoinsfromthe
top.

Foratrianglewithanoddnumberofrows,thetotalmovesrequireare:
2
(N
/4)(N4)WhereN=4,6,8,10,...

Foratrianglewithevennumberofrows,thetotalmovesrequireare:
2
((N
1)/4)(N4)WhereN=5,7,9,11,...

ThankstoAlexCrosseforsubmittingaboveformulas.

BrainTeaserNo:00053

AmanisgoingtoanAntiqueCarauction.Allpurchasesmustbepaidfor
incash.Hegoestothebankanddrawsout$25,000.

Sincethemandoesnotwanttobeseencarryingthatmuchmoney,he
placesitin15evelopesnumbered1through15.Eachenvelopecontains
theleastnumberofbillspossibleofanyavailableUScurrency(i.e.no
twotensinplaceofatwenty).

Attheauctionhemakesasuccessfulbidof$8322foracar.Hehandsthe
auctioneerenvelopesnumber(s)2,8,and14.Afteropeningtheenvelopes
theauctioneerfindsexactlytherightamount.

Howmanyonesdidtheauctioneerfindintheenvelopes?

Answer

Eachenvelopecontainsthemoneyequaltothe2raisedtotheenvelope
numberminus1.Thesentence"Eachenvelopecontainstheleastnumberof
billspossibleofanyavailableUScurrency"isonlytomisguideyou.Thisis
alwayspossibleforanyamount!!!

Onemorethingtonoticehereisthatthemanmusthaveplacedmoneyin
envelopesinsuchawaythatifhebidsforanyamountlessthan$25000,he
shouldbeabletopickthemintermsofenvelopes.

0
Firstenvelopecontains,2
=$1
1
Secondenvelopecontains,2
=$2
2
Thirdenvelopecontains,2
=$4
3
Fourthenvelopecontains,2
=$8andsoon...

Hencetheamountinenvelopesare$1,$2,$4,$8,$16,$32,$64,$128,
$256,$512,$1024,$2048,$4096,$8192,$8617

Lastenvelope(No.15)containsonly$8617astotalamountisonly$25000.

Nowashebidsfor$8322andgivesenvelopenumber2,8and14which
contains$2,$128and$8192respectively.

EnvelopeNo2conrainsone$2bill
EnvelopeNo8conrainsone$100bill,one$20bill,one$5bill,one$2bill
andone$1bill
EnvelopeNo14conrainseightyone$100bill,one$50bill,four$10bill
andone$2bill

Hencetheauctioneerwillfindone$1billintheenvelopes.

BrainTeaserNo:00090

Theminuteandthehourhandofawatchmeetevery65minutes.

Howmuchdoesthewatchloseorgaintimeandbyhowmuch?

Answer

Theminuteandthehourhandmeet11timesin12hoursinnormalwatchi.e.
theymeetafterevery
=(12*60)/11minutes
=65.45minutes
=65minutes27.16seconds


Butinourcasetheymeetafterevery65minutesmeansthewatchisgaining
27.16seconds.

BrainTeaserNo:00093

Thereisanumberthatis5timesthesumofitsdigits.Whatisthis
number?Answerisnot0.

Answer

Thenumberis45,simplybecause
45=5*(4+5)
Howdoesonefindthisnumber?

LetTbethedigitinthetensplaceandUbethedigitintheunitsplace.
Then,thenumberis10*T+U,andthesumofitsdigitsisT+U.

Thefollowingequationcanbereadilywritten:
10*T+U=5*(T+U)or
10*T+U=5*T+5*Uor
5*T=4*U

Thus,T/U=4/5

SinceTandUaredigits,Tmustbe4andUmustbe5.
Therearesixboxescontaining5,7,14,16,18,29ballsofeitherredorblue
incolour.Someboxescontainonlyredballsandotherscontainonlyblue.

Onesalesmansoldoneboxoutofthemandthenhesays,"Ihavethesame
numberofredballsleftoutasthatofblue."

Whichboxistheonehesoldsout?
Answer

Totalnoofballs=5+7+14+16+18+29=89

Totalnumberofballsareodd.Also,samenumberofredballsandblueballs
areleftoutaftersellingonebox.Soitisobviousthattheboxwithodd

numberofballsinitissoldouti.e.5,7or29.

Nowusingtrialanderrormethod,
(8929)/2=60/2=30and
14+16=5+7+18=30

Soboxwith29ballsissoldout.

BrainTeaserNo:00218

EktagotchocolatestogiveherfriendsonherBirthday.Ifshegives3
chocolatestoeachfriend,onefriendwillgetonly2chocolates.Also,if
shegives2chocolatestoeachfriends,shewillleftwith15chocolates.

HowmanychocolatesEktagotonherBirthday?andhowmanyfriends
arethere?

Answer

47Chocolatesand16Friends

Let'sassumethattherearetotalCchocolatesandFfriends.

Accordingtofirstcase,ifshegives3chocolatestoeachfriend,onefriend
willgetonly2chocolates.
3*(F1)+2=C

Similarly,ifshegives2chocolatestoeachfriends,shewillleftwith15
chocolates.
2*F+15=C

Solvingabove2equations,F=16andC=47.Hence,Ektagot47
chocolatesand16friends
PoojaandEshameteachotherafterlongtime.Inthecourseoftheir
conversation,PoojaaskedEshaherage.Eshareplied,"Ifyoureversemy
age,youwillgetmyhusbund'sage.Heisofcourseolderthanme.Also,the
differencebetweenourageis1/11thofthesumofourage."

CanyouhelpoutPoojainfindingEsha'sage?
Answer

Esha'sageis45years.

AssumethatEsha'sageis10X+Yyears.Hence,herhunsbandsageis(10Y
+X)years.

Itisgiventhatdifferencebetweentheirageis1/11thofthesumoftheirage.
Hence,
[(10Y+X)(10X+Y)]=(1/11)[(10Y+X)+(10X+Y)]
(9Y9X)=(1/11)(11X+11Y)
9Y9X=X+Y
8Y=10X
4Y=5X

Hence,thepossiblevaluesareX=4,Y=5andEsha'sageis45years.
Afishhadatailaslongasitsheadplusaquarterthelenghtofitsbody.Its
bodywasthreequartersofitstotallength.Itsheadwas4incheslong.

Whatwasthelengthofthefish?
Submitted

Thefishis128incheslong.

Itisobviousthatthelenghtofthefishisthesummationoflenghtsofthe
head,thebodyandthetail.Hence,
Fish(F)=Head(H)+Body(B)+Tail(T)

Butitisgiventhatthelenghtoftheheadis4inchesi.e.H=4.Thebodyis
threequartersofitstotallengthi.e.B=(3/4)*F.Andthetailisitsheadplus
aquarterthelenghtofitsbodyi.e.T=H+B/4.Thus,theequationis
F=H+B+T
F=4+(3/4)*F+H+B/4
F=4+(3/4)*F+4+(1/4)*(3/4)*F
F=8+(15/16)*F
(1/16)*F=8
F=128inches

Thus,thefishis128incheslong.

Assumethatyouhavejustheardofascandalandyouarethefirstoneto
know.Youpassitontofourpersoninamatterof30minutes.Eachofthese
fourinturnpassesittofourotherpersonsinthenext30minutesandsoon.

HowlongitwilltakeforeverybodyintheWorldtogettoknowthescandal?

AssumethatnobodyhearsitmorethanonceandthepopulationoftheWorld
isapproximately5.6billions.
Answer

EverybodyintheWorldwillgettoknowthescandalin8hours.

Youcametoknowofascandalandyoupasseditonto4personsin30
minutes.Sototal(1+4)5personswouldknowaboutitin30minutes.

Bytheendofonehour,16morepersonswouldknowaboutit.Sototalof
(1+4+16)21personswouldknowaboutitinonehour.

Similarly,theother(1+4+16+64)personswouldhaveknowaboutitinone
andahalfhours.(1+4+16+64+256)personswouldhaveknowaboutitin
twohoursandsoon...

Itcanbededucedthatthetermsoftheaboveseriesarethepowerof4i.e.
4^0,4^1,4^2,4^3andsoonupto(2N+1)terms.Also,thelasttermwould
be4^2NwhereNisthenumberofhours.

Sumoftheabovementionedseries=[4^(2N+1)1]/3

Thesumoftheseriesmustbe5.6billions.Hence,equatingthesumofthe
serieswith5.6billions,wegetN=8hours.

ScandalstravelFAST!!!
ABC

EFG

I
Eachofthedigitsfrom1to9isrepresentedbyadifferentletterabove.Also,
A+B+C=C+D+E=E+F+G=G+H+I=13

WhichdigitdoesErepresent?
Answer

Erepresents4.

Findoutallpossiblegroupsofthreedifferentnumbersthataddupto13and
arrangethemaccordingtogivencondition.

Ifonenumberis9,itmustgowith1and3.
Ifonenumberis8,itmustgowitheither1and4or2and3.
Ifonenumberis7,itmustgowitheither1and5or2and4.
Ifonenumberis6,itmustgowitheither2and5or3and4.

Itisclearthat9mustgowith1and3.Also,nodigitmaybeusedinmore
thantwosums.Hence,thereare2cases:
CaseI
:If8goeswith1and4,then7goeswith2and4,then6goeswith2
and5.
CaseII
:If8goeswith2and3,then7goeswith2and4,then6goeswith3
and4.

ButincaseII,3isusedinthreesums.Hence,CaseIiscorrect.Andthe
possiblearrangementsare:

931562

87

472481

63

59

Thus,Emustbe4.
A,BandCarethreepointsonastraightline,notnecessarilyequidistant
withBbeingbetweenAandC.Threesemicirclesaredrawnonthesame
sideofthelinewithAB,BCandACasthediameters.BDisperpendicular
tothelineABC,andDliesonthesemicircleAC.

Ifthefunnyshapeddiagrambetweenthethreesemicircleshasanareaof
1000squarecms,findthelengthofBD.

Answer

ThelengthofBDis35.68cms

Thereare3rightangledtrianglesABD,CBDandADC.

FromABD,AB^2+BD^2=AD^2I
FromCBD,CB^2+BD^2=CD^2II
FromADC,AD^2+CD^2=AC^2III

AddingIandII,
AB^2+BC^2+2*BD^2=AD^2+CD^2IV

FROMIIIandIV
AB^2+BC^2+2*BD^2=AC^2
AB^2+BC^2+2*BD^2=(AB+CB)^2
2*BD^2=2*AB*CB
BD^2=AB*CB
BD=SQRT(AB*CB)

Giventhatfunnyshapeddiagrambeweenthreesemicircleshasanareaof
1000squarecms.
[PI/2*(AC/2)^2][PI/2*(AB/2)^2][PI/2*(BC/2)^2]=1000
PI/8*[AC^2AB^2BC^2]=1000
PI*[(AB+BC)^2AB^2BC^2]=8000
PI*[2*AB*BC]=8000
AB*BC=4000/PI

HenceBD=SQRT(4000/PI)=35.68cms
wherePI=3.141592654

Hence,thelengthofBDis35.68cms.

Submit

Answer

Users
Answer(33)

BrainVista
Answer

BrainTeaserNo:00660

Gomzihas3timepiecesinhishouseawallclock,analarmclockanda
wristwatch.Thewristwatchisalwaysaccurate,whereasthewallclock
gains2minuteseverydayandthealarmclockloses2minuteseveryday.

Atexactlymidnightlastnight,allthreewatcheswereshowingthesame
time.

Iftodayis25July2003,thenonwhichdateallthreeclockswillshowthe
sametimeagain?

Answer

Allthreeclockswillshowthesametimeagainonmidnightbetween19
July2004and20July2004.

Aclockfinishesonroundin12*60i.e.720minutes.

Ifaclockgains2minuteseveryday,thenitwouldbe720minutesahead
after360days.Thus,after360days,itwillshowthesametimeagain.

Similary,ifaclockloses2minuteseveryday,thenitwouldbe720minutes
behindafter360days.Thus,after360days,itwillshowthesametime
again.

Thus,after360daysallthreeclockswillshowthesametimeagaini.e.
midnightbetween19July2004and20July2004.
Youhave9marbles.8marblesweigh1ounceeach,&onemarbleweighs
1.5ounces.Youareunabletodeterminewhichistheheaviermarbleby
lookingatthem.Youhaveaweighingscalethatconsistsof2pans,butthe
scaleisonlygoodfor2totalweighings.

Howcanyoudeterminewhichmarbleistheheaviestoneusingthescale&
in2weighings?
Answer

Divide9marblesinto3groupsof3marbleseach.

Takeany2groupsandplacethemoneachpan.Iftheybalance,removethe
marblesfromthepans,&placeany2ofthemarblesfromtheremaining
unweighedgrouponthepans,1oneachpan.

Ifoneisheavier,itistheheaviermarble,butiftheybalance,theremaining
unweighedmarbleistheheavierone.

Ifyourfirstweighingdoesnotbalance,removethemarblesfromthelighter
pan,&place1marbleoneachpanfromtheheavierpan.Theheavier1isthe
1.5ouncemarble,butiftheybalance,thenthemarblefromtheheavypan
fromthefirstweighingthatwasnotweighedinthesecondweighingisthe
heavy1.
Onceaweekawagondriverleaveshishutanddriveshiswagontotheriver
docktopickupsuppliesforhistown.At4:05PM,onefifthofthewayto
thedock,hepassestheTemple.At4:15PM,onethirdoftheway,hepasses
thePreetamDaDhabaa.

Atwhattimedoeshereachedthedock?

Answer

5:05PM

At4:05PM,thewagondriverpassesthetemple,onefifthofthewaytothe
dock.Also,at4:15PM,hepassesthePreetamDaDhabaa,onethirdofthe
way.Thus,hetravels2/15(1/31/5)ofthedistancein10minutes.

At4:15PM,hehasalreadytravelled1/3ofthedistance.Thus2/3oftheway
isremaining,whichcanbetravelledin
=((2/3)*10)/(2/15)
=50minutes

At4:15,hewasatPreetamDaDhabaa.andremainingwaywilltake50more
minutes.Hence,thedriverwillreachat5:05PMtothedock.

BrainTeaserNo:00115

Fourprisonersescapefromaprison.
Theprisoners,Mr.East,Mr.West,Mr.South,Mr.Northheadtowards
differentdirectionsafterescaping.
Thefollowinginformationoftheirescapewassupplied:
TheescaperouteswereNorthRoad,SouthRoad,EastRoadand
WestRoad
Noneoftheprisonerstooktheroadwhichwastheirnamesake
Mr.EastdidnottaketheSouthRoad
Mr.WestdidnottheSouthRoad
TheWestRoadwasnottakenbyMr.East
Whatroaddideachoftheprisonerstaketomaketheirescape

Answer

Putallthegiveninformationintothetablestructureasfollow:

NorthRoad

SouthRoad

EastRoad

WestRoad

Mr.North

No

Mr.South

No

Mr.East

No

No

No

Mr.West

No

No

Nowfromtable,twothingsareobviousandtheyare:
Mr.NorthtooktheSouthRoad
Mr.EasttooktheNorthRoad
Putthisinformationintothetable,Alsokeepinmindthattheprisonershead
towardsdifferentdirectionsafterescaping.

NorthRoad

SouthRoad

EastRoad

WestRoad

Mr.North

No

YES

No

No

Mr.South

No

No

Mr.East

YES

No

No

No

Mr.West

No

No

No

Nowfromthetable:
Mr.WesttooktheEastRoad
Mr.SouthtooktheWestRoad
Sotheansweris:
Mr.NorthtooktheSouthRoad
Mr.SouthtooktheWestRoad
Mr.EasttooktheNorthRoad
Mr.WesttooktheEastRoad

Shahrukhspeakstruthonlyinthemorningandliesintheafternoon,whereas
Salmanspeakstruthonlyintheafternoonandliesinthemorning.

AsaysthatBisShahrukh.

IsitmorningorafternoonandwhoisAShahrukhorSalman?
Answer

ItisAfternoonandAcanbeSalmanorShahrukh.IfAisSalman,heis
speakingtruth.IfAisShahrukh,heislying.


Wanttoconfirmit?Considerfollowing4possibleanswersandcheckforits
truthnessindividually.
1. ItisMorningandAisShahrukh
2. ItisMorningandAisSalman
3. ItisAfternoonandAisShahrukh
4. ItisAfternoonandAisSalman

Arichmandied.Inhiswill,hehasdividedhisgoldcoinsamonghis5sons,
5daughtersandamanager.

Accordingtohiswill:Firstgiveonecointomanager.1/5thoftheremaining
totheelderson.Nowgiveonecointothemanagerand1/5thofthe
remainingtosecondsonandsoon.....Aftergivingcoinsto5thson,divided
theremainingcoinsamongfivedaughtersequally.

Allshouldgetfullcoins.Findtheminimumnumberofcoinshehas?
Answer

Wetriedtofindoutsomesimplemathematicalmethodandfinallywewrote
smallCprogramtofindouttheanswer.Theansweris3121coins.

Hereisthebreakup:
Firstson=624coins
Secondson=499coins
Thirdson=399coins
Forthson=319coins
Fifthson=255coins
Daughters=204each
Manager=5coins
Thereisagridof20squaresby10squares.Howmanydifferentrectangles
arepossible?

Notethatsquareisarectangle.
Answer

11550

TheGenericsolutiontothisis:
Totalnumberofrectangles=(Summationofrownumbers)*(Summationof
columnnumbers)

Herethereare20rowsand10columnsorviceversa.Hence,totalpossible
rectangles
=(20+19+18+17+16+....+3+2+1)*(10+9+8+7+....+3+2
+1)
=(210)*(55)
=11550

Hence,total11,550differentrectanglesarepossible.

Ifyoudon'tbelieveit,tryformulaonsomesmallergridslike4x2,3x2,3x3
etc...
IfA+B=C,DC=AandEB=C,thenwhatdoesD+Fstandsfor?Provide
youranswerinlettertermsaswellasinnumberterms.
Submittedby:David
Answer

Jor10

Asimpleone.

Assumethateachcharacterrepresentsthenumberequivalenttotheposition
inthealphabeti.e.A=1,B=2,C=3,D=4andsoon.Nowlet'scheckour
assumption.

A+B=Ci.e.1+2=3
DC=Ai.e.43=1
EB=Ci.e.52=3

Thus,ourassumptionwasCorrect.Hence,D+F=Ji.e.4+6=10
Awomantookacertainnumberofeggstothemarketandsoldsomeof
them.

Thenextday,throughtheindustryofherhens,thenumberleftoverhad
beendoubled,andshesoldthesamenumberasthepreviousday.

Onthethirddaythenewremainderwastripled,andshesoldthesame
numberasbefore.

Onthefourthdaytheremainderwasquadrupled,andhersalesthesameas
before.

Onthefifthdaywhathadbeenleftoverwerequintupled,yetshesold
exactlythesameasonallthepreviousoccasionsandsodisposedofher
entirestock.

Whatisthesmallestnumberofeggsshecouldhavetakentomarketthefirst
day,andhowmanydidsheselldaily?Notethattheanswerisnotzero.
Submitted
Answer

Shetook103eggstomarketonthefirstdayandsold60eggseveryday.

Let'sassumethatshehadNeggsonthefirstdayandshesoldXeggs
everyday.Puttingdownthegiveninformationinthetableasfollow.
Days

Eggsatthestartoftheday

EggsSold

EggsRemaining

Day1

NX

Day2

2N2X

2N3X

Day3

6N9X

6N10X

Day4

24N40X

24N41X

Day5

120N205X

120N206X

Itisgiventhatshedisposedofherentirestockonthefifthday.Butfromthe
tableabove,thenumberofeggsremainingare(120N206X).Hence,
120N206X=0
120N=206X
60N=103X

ThesmallestvalueofNandXmustbe103and60respectively.Hence,she
took103eggstomarketonthefirstdayandsold60eggseveryday.
Johnlivesin"FriendsSociety"whereallthehousesareinarowandare

numberedsequentiallystartingfrom1.Hishousenumberis109.

Jessylivesinthesamesociety.Allthehousenumbersontheleftsideof
Jessy'shouseaddupexactlythesameasallthehousenumbersontheright
sideofherhouse.

WhatisthenumberofJessy'shouse?Findtheminimalpossibleanswer.
Answer

Thereare288housesandJessy'shousenumberis204.

Let'sassumethatinthe"FriendsSociety"therearetotalNhousesnumbered
from1toNandJessy'shousenumberisX.

NowitisgiventhatallthehousenumbersontheleftsideofJessy'shouse
addupexactlythesameasallthehousenumbersontherightsideofher
house.Hence,
1+2+3+.....+(X1)=(X+1)+(X+2)+(X+3)+.....+N

BoththesidesoftheaboveequationsareinA.P.Hence,usingA.P.
summationformaula,

[(X1)/2][2*(1)+(X11)]=[(NX)/2][2*(X+1)+(NX1)]
[X1][(2)+(X2)]=[NX][(2X+2)+(NX1)]
(X1)(X)=(NX)(N+X+1)
2
2
2
X
X=N
+NX+NNXX
X
2
2
2
X
=N
+NX

2
2
2X
=N
+N
2
2
X
=(N
+N)/2
2
X
=N(N+1)/2

Now,usingTrialandErrormethodtofindvaluesofNandXsuchthat
aboveequationissatisfied,weget
1. N=8,X=6
2. N=49,X=35
3. N=288,X=204
4. N=1681,X=1189
5. N=9800,X=6930

ButwerequireminimalpossibleansweranditisgiventhatJohn'shouse
numberis109.Itmeansthatthereareatleast109houses.Hence,firsttwo
arenotpossible.Andtheansweris:thereare288housesandJessy'shouse
numberis204.
Makaylahad$1.19inchange.Noneofthecoinswasadollar.

Nicoleaskherforchangeforadollar,butMakaylacouldnotmakechange.

Whatcoinsdidshehave?
Submitted
Answer

AsitisgiventhatMakaylahad$1.19,itmeansshewouldhavefourpennies.
Now,theremaining$1.15incoinsmustnotaddupforexactlyadollar.
Thereforeshewouldnothave4quartersor2quartersand5dimes.Butshe
wouldhaveeither1quarteror3quarters.Hence,thereare2solutions.

SolutionI
1Quarter,9Dimes,4Pennies(0.25+0.90+0.04=$1.19)

SolutionII
3Quarters,4Dimes,4Pennies(0.75+0.40+0.04=$1.19)
Agroupoffriendswentonaholidaytoahillstation.Itrainedfor13days.
Butwhenitrainedinthemorning,theafternoonwaslovely.Andwhenit
rainedintheafternoon,thedaywasprecededbyclearmorning.

Altogethertherewere11verynicemorningsand12veryniceafternoons.
Howmanydaysdidtheirholidaylast?
Answer

Theholidaylastfor18days.

Let'sassumethenumberofdaysasfollows:
Raininthemorningandlovelyafternoon=Xdays
Clearmorningandrainintheafternoon=Ydays
Noraininthemorningandintheafternoon=Zdays

Numberofdayswithrain=X+Y=13days
Numberofdayswithclearmornings=Y+Z=11days

Numberofdayswithclearafternoons=X+Z=12days

Solvingabove3equations,wegetX=7,Y=6andZ=5

Hence,totalnumberofdaysonholiday=18days

BrainTeaserNo:00299

SubstitutedigitsfortheletterstomakethefollowingDivisiontrue
YFY

AY|NELLY

|NLY

PPL

PNH

NLY

NLY

000
Notethattheleftmostlettercan'tbezeroinanyword.Also,theremustbe
aonetoonemappingbetweendigitsandletters.e.g.ifyousubstitute3
fortheletterN,nootherlettercanbe3andallotherNinthepuzzlemust
be3.
Submittedby:Calon

Answer


SeethepatternoftheY.AY*Y=NLYi.e.YismultipliedbyYandthe
lastdigitoftheanswerisalsoY.Thus,thevalueofYwouldbe5or6.

Also,
H=0asLH=L
P=2NasPN=N
LY=P=2N
EL=p

Let'sfindouttheminimumpossiblevalues.IfN=1,thenP=2,Y=5,L=7and
E=9.NotethatthevalueofYcannotbe6asitmakesL=8andE=10which
isnotpossible.Hence,Y=5,N=1,P=2,L=7,E=9,H=0

Now,usingtrialnerrororrathersolvingF*AY=PNH,wegetF=6andA=3.

565YFY

35|19775AY|NELLY

|175|NLY

227PPL

210PNH

175NLY

175NLY

000000

BrainTeaserNo:00566

HereisthefamilytreeofMr.RAHUL
RAHUL

||||

RATISHYASHOMTRILOK

||?

|||||

AMARAMITRAMHARSHASHOK

||

|||||

HowmanychildrendoesMr.TRILOKhave?

Answer

TRILOKhave5children.

Nameofthepersonandnumberofhischildrenarerelatedbysomepattern.

Assigneachvowelfollowingvalues.

A=0E=1I=2O=3U=4

Thenumberofchildrentoanypersonisthesumofthevaluesrepresented
byvowelsinhisname.
RATISH=0(A)+2(I)=2
OM=3(O)=3
AMIT=0(A)+2(I)=2
ASHOK=0(A)+3(O)=3
TRILOK=2(I)+3(O)=5

Hence,TRILOKhave5children.

MajorJasbirisformingfivepersonSpecialTaskGroup.Thegroupmust
containoneleader,twobombexpertsandtwosoldiers.

P,QandRarepossiblebombexperts.R,SandTarepossibleleaders.U,V
andWarepossiblesoldiers.Also,PandRpreferstoworkwitheachotherin
thesameteam.TpreferstoworkonlyifVworks.

HowmanydifferentpossibleGroups,MajorJasbircanmake?
Answer

MajorJasbircanmake8differentpossiblegroups.

As2bombexpertstobeselectedfromthegiven3andalsoP&Rprefersto
worktogether,PRmustbethereinallthepossibleGroups.Also,Tprefers
toworkonlyifVworks.Itdoesn'tmeanthatVwon'tworkwithoutT.

Hence,possiblegroupsare:
PRSUV
PRSVW
PRSWU

PRTUV
PRTVW

PQRUV
PQRVW
PQRWU


Hence,there8differentgroupsarepossible.
ThesecretagentXemailedsomecodetohisheadoffice.Theyare
"RADAR,LEVEL,ROTOR,REDIVIDER,MOTOR".Butfourofthese
fivewordshavesomethingincommonandoneisfake.

Canyoutellwhichoneisfake?Ignorethefactthatfourofthecodewords
areofthesamelength.
Answer

ThefakecodewordisMOTOR.

AllthecodewordsexceptMOTORarePalindromes.

BrainTeaserNo:00287

InthevillagecalledTALAJA,onlythreeTVchannelsareavailable
MoonPlus,MonyandMeeTV.

Outof4000TVviewersinthevillage,1500watchMoonTV,2000watch
Monyand2500watchMeeTV.

Amongstthese,500viewerswatchMoonPlusandMony,800watch
MoonPlusandMeeTV,and1000watchMonyandMeeTV.

Howmanyviewerswatchallthreechannels?

Answer

300viewerswatchallthreechannels.

Let'sassumethattotalXviewerswatchallthreechannels.

totalviewerswhowatchonlyMoonPlusandMony=500X
totalviewerswhowatchonlyMoonPlusandMeeTV=800X
totalviewerswhowatchonlyMonyandMeeTV=1000X

totalviewerswhowatchonlyMoonPlus

=1500(500X)(800X)X
=200+X

totalviewerswhowatchonlyMony
=2000(500X)(1000X)X
=500+X

totalviewerswhowatchonlyMeeTV
=2500(1000X)(800X)X
=700+X

Weknowthattotalviewersare4000.Summingupall7values,
X+(500X)+(800X)+(1000X)+(200+X)+(500+X)+(700+X)
=4000
X+3700=4000
X=300

Hence,total300viewerswatchallthreechannels.
Amanwaslookingataportrait.Someoneaskedhim,"Whosepictureare
youlookingat?"

Hereplied,pointingattheportrait:"BrothersandsistershaveInone,butthis
man'ssonismyfather'sson."

Nowwhosepictureisthemanlookingat?
Answer

ThemanislookingathisFATHER'sportrait.

"myfather'sson"isthemanhimselfashedonothaveanybrothersand
sisters.Sothestatementreducesto"thisman'ssonismyself."Nowitis
clearthattheportraitisofhisfather.
Giventhefollowingfacts:
1. DineshisyoungerthanFarukhandolderthanGurmit.
2. JatinisyoungerthanChanduandolderthanEshrat.
3. AmitisyoungerthanIrfanandolderthanChandu.
4. FarukhisyoungerthanBhavinandolderthanHemant.
5. IrfanisyoungerthanGurmitandolderthanJatin.

6.

HemantisolderthanGurmit.

WhoistheYoungest?
Answer

Eshratistheyoungest.

Discardwhoeverareolderthansomeone.

From(1)GurmitisyoungerthanDineshandFarukh.
From(5)JatinisyoungerthanIrfanandGurmit.
From(2)EshratisyoungerthanJatinandChandu.

Fromabove3deductions,EshratisyoungerthanDinesh,Farukh,Irfan,
Gurmit,JatinandChandu.

Also,
From(3)ChanduisyoungerthanAmitandIrfan.
From(4)HemantisyoungerthanFarukhandBhavin.
From(6)GurmitisyoungerthanHemant.

Fromabove3deductions,GurmitisyoungerthanFarukh,Bhavinand
Hemant.Also,ChanduisyoungerthanAmitandIrfan.Butasseenearlier,
EshratisyoungerthanGurmitandChandu.

Hence,Eshratistheyoungest.
LastSaturdayMilanwentforthelatenightshowandcamelate.Inthe
morningfamilymembersaskedhimwhichmoviedidhesee.Hegave
differentanswerstoeveryone.
HetoldtohisfatherthathehadgonetoseeMONEY.
Accordingtohismom,hesaweitherJOHNYorBABLU.
HiselderbrothercametoknowthathesawBHABI.
Tohissister,hetoldROBOT.
AndhisgrandpaheardthathesawBUNNY.
Thus,Milangavesixmovienames,allfiveletterwords.Buthesawsome
othermoviewithfiveletterword.Moreover,eachofthesixmovienames
mentionedabovehasexactlytwoletterscommonwiththemoviehesaw.
(withthesamepositions)

CanyoutellwhichmoviedidMilansee?
Answer

MilansawBOBBY.

ThesixmovienamesareMONEY,JOHNY,BABLU,BHABI,ROBOT
andBUNNY.

CompareMONEYandJOHNY.TheyhaveOcommonatthesecondplace
andYcommonatthefifthplace.Also,theycan'thavetwodifferentletters
each,commonwiththerequiredmovieasthelettersinremainingthree
placesarealldifferent.Thus,therequiredmoviemusthaveeitherOatthe
secondplaceorYatthefifthplaceorboth.

Similarly,comparingJOHNYandBUNNYtherequiredmoviemusthave
eitherNatthefourthplaceorYatthefifthplaceorboth.Also,comparing
MONEYandBUNNYtherequiredmoviemusthaveeitherNatthethird
placeorYatthefifthplaceorboth.

Fromtheabove3deduction,eitherYisatfifthplaceorOisatthesecond
placeandNisatthethird&fourthplace.Thelatercombinationisnot
possibleasBABLU,BHABI&ROBOTwillneedatleast3otherletters
whichmakestherequiredmovie6letterlong.Hence,therequiredmovie
musthaveYatthefifthplace.

NowYisnotthereinBABLUandBHABIatthefifthplaceandtheyhave
onlyBcommonatthefirstplace.Hence,Bmustbethefirstletter.

AsBisatthefirstplaceandYisatthefifthplaceandeverymoviehas
exactly2letterscommonwiththerequiredmovie.FromBUNNY,the
requiredmoviedonothaveUatthesecondplaceandNatthethirdand
fourthplace.NowlookingatJOHNYandMONEY,theymusthaveO
commonatthesecondplace.

UsingthesamekindofargumentsforBABLU,BHABIandROBOT,we
canconcludethatMilansawBOBBY.

Jimliesalot.Hetellsthetruthononlyonedayinaweek.

Onedayhesaid:"IlieonMondaysandTuesdays."
Thenextdayhesaid:"TodayiseitherSunday,SaturdayorThursday."
Thenextdayhesaid:"IlieonFridaysandWednesdays."

OnwhichdayoftheweekdoesJimtellthetruth?
Answer

JimtellsthetruthonTuesday.

AsJimtellstruthonlyononedayinaweek,hisstatementonday1andday
3bothcannotbefalse.Otherwisehetellstruthonmorethanonedaysina
week.Also,allthreestatementsaremadonthreeconsecutivedays,
statementmadeonday1andday3bothcannotbetrue.Thus,eitherthe
statementmadeonday1orday3istrueandotherisfalse.Also,the
statementmadeonday2mustbefalsei.e.day1isnotSaturday,Fridayor
Wednesday.

Let'sassumethatthestatement1istrue.Thenfromthestatement3,day1
mustbeeitherFridayorWednesday.Butitisalreadydeducedthatday1is
notSaturday,FridayorWednesday.

Hence,thestatementmadeonday1isfalseandthelaststatementistrue.
thenfromthestatement1,day3mustbeeitherMondayorTuesday.Butitis
alreadydeducedthatday1cannotbeSaturdayi.e.day3can'tbeMonday.
Hence,JimtellsthetruthonTuesday.

4mencandig4holesin4days.

Howmanyhoursdoesittakefor1mantodighalfahole?
Submitted
Answer

Thereisnothinglike"HALFHOLE".
ConsiderachessboardwithasingleRook.ARookcanmoveanynumberof
squaresideways/forward,butnotdiagonally.

WhatistheminimumnumberofmovestheRookneedstomake,inorderto
passoverallthesquaresonthechessboardandreturntotheoriginal
position?

Answer

16moves

AsaRookcanmoveanynumberofsquaresideways/forward,butnot
diagonallyandthereare8rowsand8columnsonthechessboardtheRook
needsminimum16movestopassoverallthesquaresandreturntothe
originalposition.
Afarmerneeds8gallonsofwater.Hehasonlythreeunmaredbuckets,two
6gallonandone11gallonbucket.

Howcanhecollect8gallonsofwaterusingthreeunmarkedbuckets?
Providesolutionwithminimalwaterwastage.
Answer

Hereisthesolutionwith10gallonwaterwastage.
OPERATIONS
6 6 11
Fill6gallonbucketwithwater

6 0 0

Empty6gallonbucketinto11gallonbucket

0 0 6

Fill6gallonbucketwithwater

6 0 6

Fill11gallonbuckettofullusingfilled6gallonbucket.Thiswill
1 0 11
leave1gallonwaterin6gallonbucket
Empty11gallonbucketintosecond6gallonbucket.

1 6 5

Empty11gallonbucketwastageof5gallonwater

1 6 0

Emptysecond6gallonbucketinto11gallonbucket

1 0 6

Fillseccond6gallonbucketwithwater

1 6 6

Fill11gallonbuckettofullusingfilledsecond6gallonbucket.
Thiswillleave1gallonwaterinsecond6gallonbucket

1 1 11

Fillfirst6gallonbucketwith1gallonwaterwhichisinsecond6
gallonbucket

2 0 11

Empty11gallonbucketintosecond6gallonbucket.

2 6 5

Empty11gallonbucketwastageof5gallonwater

2 6 0

Fill11gallonbucketwithwaterinboththe6gallonbuckets
0 0 11
Iboughtacarwithapeculiar5digitnumberedlicenceplatewhichon

reversingcouldstillberead.Onreversingvalueisincreasedby78633.

Whatstheoriginalnumberifalldigitsaredifferent?
Answer

Only0168and9canbereadupsidedown.Soonrearrangingthesedigits
wegettheansweras10968.
JackandJillareplayingcardsforastakeof$1agame.Attheendofthe
evening,Jackhaswon3gamesandJillhaswon$3.Howmanygamesdid
theyplay?
Submittedby:NathalieDrouin
Answer

Theyplayedtotalof9games.Jackwon3gamesandJillwon6games.

IfJackhaswonthreegamesandJillhaswon$3,shelostadollarforeach
loss,thereforeshehaswon6andlost3tomake$3andhewontheother3
thatshelost!

SamandMalahaveaconversation.
SamsaysIamcertainlynotover40
MalasaysIam38andyouareatleast5yearsolderthanme
NowSamsaysyouareatleast39
Allthestatementsbythetwoarefalse.Howoldaretheyreally?
Answer

Samis41andMalais37.

Let'sinverttheteaserandreaditlikethis:
SamsaysIamcertainlyover40
MalasaysIamnot38andyouareatmost4yearsolderthanme
NowSamsaysyouareatmost38
FromfirststatementitisclearthatSamisover40.Also,fromnext2
statementsitisclearthatMalaislessthen38.Hencethepossibilitiesare:
Sam=41,42,43,44,45,......
Mala=37,36,35,34,33,......

Italsosaysthatthedifferencebetweentheirageismaximum4years.

Hence,thereisonlyonepossiblepairi.e.41and37,allothercombination
havedifferencesmorethen4.

HencetheanswerSamis41andMalais37.
Apersontravelsonacyclefromhometochurchonastraightroadwith
windagainsthim.Hetook4hourstoreachthere.

Onthewaybacktothehome,hetook3hourstoreachaswindwasinthe
samedirection.

Ifthereisnowind,howmuchtimedoeshetaketotravelfromhometo
church?
Answer

LetdistancebetweenhomeandchurchisD.

Apersontook4hourstoreachchurch.Sospeedwhiletravellingtowards
churchisD/4.

Similarly,hetook3hourstoreachhome.Sospeedwhilecomingbackis
D/3.

Thereisaspeeddifferenceof7*D/12,whichisthewindhelpingpersonin1
direction,&slowinghimintheotherdirection.Averagethe2speeds,&you
havethespeedthatpersoncantravelinnowind,whichis7*D/24.

Hence,personwilltakeD/(7*D/24)hourstotraveldistanceDwhichis
24/7hours.

Answeris3hours25minutes42seconds
ThereareNsecretagentseachknowadifferentpieceofsecretinformation.
Theycantelephoneeachotherandexchangealltheinformationtheyknow.
Afterthetelephonecall,theybothknowanythingthateitherofthemknew
beforethecall.

Whataretheminimumnumberoftelephonecallsneededsothatallofthe
themknoweverything?
Answer

(2N3)telephonecalls,forN=2,3
(2N4)telephonecalls,forN>3

DividetheNsecretagentsintotwogroups.IfNisodd,onegroupwill
containoneextraagent.

Considerfirstgroup:agent1willcallupagent2,agent2willcallupagent3
andsoon.Similarlyinsecondgroup,agent1willcallupagent2,agent2
willcallupagent3andsoon.After(N2)calls,twoagentsineachthe
groupwillknowanythingthatanyoneknewinhisgroup,saytheyareY1&
Y2fromgroup1andZ1&Z2fromgroup2.

Now,Y1willcallupZ1andY2willcallupZ2.Hence,innexttwocalls
totalof4agentswillknoweverything.

Now(N4)telephonecallsarereqiuredforremaining(N4)secretagents.

Totaltelephonecallsrequireare
=(N2)+2+(N4)
=2N4

Let\'stakeanexample.Saythereare4secretagentsW,X,Y&Z.Divide
themintotwogroupsof2eachi.e.(W,X)and(Y,Z).Here,4telephone
callsarerequired.
1. WwillcallupX.
2. YwillcallupZ.
3. W,whoknowsWXwillcallupY,whoknowsYZ.
4. X,whoknowsWXwillcallupZ,whoknowsYZ.
Takeananotherexample.Saythereare5secretagentsJ,K,L,M&N.
Dividethemintotwogroupsi.e.(J,K)and(L,M,N).Here,6telephone
callsarerequired.
1. JwillcallupK.
2. LwillcallupM.
3. MwillcallupN.NowMandNknowLMN.
4. J,whoknowsJKwillcallupM,whoknowsLMN.
5. K,whoknowsJKwillcallupN,whoknowsLMN.
6. Lwillcalluptoanyoneoffour.
Mrs.FhasinvitedseveralwivesofdelegatestotheUnitedNationsforan

informalluncheon.Sheplanstoseather9guestsinarowsuchthateachlady
willbeabletoconversewiththepersondirectlytoherleftandright.Shehas
preparedthefollowinglist.

Mrs.FspeaksEnglishonly.
Mrs.GspeaksEnglishandFrench.
Mrs.HspeaksEnglishandRussian.
Mrs.JspeaksRussianonly.
Mrs.KspeaksEnglishonly.
Mrs.LspeaksFrenchonly.
Mrs.MspeaksFrenchandGerman.
Mrs.NspeaksEnglishandGerman.
Mrs.OspeaksEnglishonly.

Howmanydistinctseatingarrangementsarepossible?Giveallpossible
seatingarrangements.

NotethatABCDandDCBAarethesame.
Answer

126distinctseatingarrangementsarepossible.

Mrs.JandMrs.HmustbetogetherandMrs.JmustbeattheendasMrs.J
speaksonlyRussianandMrs.HistheonlyotherRussianspeaker.

Mrs.LspeaksonlyFrenchandtherearetwoothersMrs.GandMrs.M
whospeakFrench.Herethereare2cases.

CASEA:
Mrs.Lisattheotherend
IfMrs.Lisattheotherend,eitherMrs.GorMrs.Mmustseatnextto
her.
CASEAA:
Mrs.GseatsnexttoMrs.L
Then,Mrs.MmustseatnexttoMrs.GandMrs.Nmustseat
nexttoMrs.M.ThisisbecauseMrs.MspeaksFrenchand
German,andMrs.NistheonlyotherGermanspeaker.Thus,
thepossibleseatingarrangementis
JHxxxNMGL
,wherexis
theEnglishspeakers.Mrs.F,Mrs.KandMrs.Ocanbe
arrangedinremaining3positionsin3!differentwaysi.e.6
ways.

CASEAB:
Mrs.MseatsnexttoMrs.L
Ifso,theneitherMrs.NorMrs.GmustseatnexttoMrs.M
CASEABA:
Mrs.NseatsnexttoMrs.M
Thus,thepossibleseatingarrangementis
JHxxxxNML
,
wherexistheEnglishspeakers.Mrs.F,Mrs.G,Mrs.K
andMrs.Ocanbearrangedinremaining4positionsin
4!differentwaysi.e.24ways.
CASEABB:
Mrs.GseatsnexttoMrs.M
Thus,thepossibleseatingarrangementis
JHxxxxGML
,
wherexistheEnglishspeakers.Mrs.F,Mrs.K,Mrs.N
andMrs.Ocanbearrangedinremaining4positionsin
4!differentwaysi.e.24ways.

CASEB:
Mrs.Ldoesnotseatattheend
ItmeansthatMrs.G,Mrs.LandMrs.Mmustseattogether.Also,
Mrs.LmustseatbetweenMrs.GandMrs.M.

CASEBA:
Mrs.GseatsleftandMrs.MseatsrighttoMrs.
Li.e.GLM

CASEBAA:
GLMisattheotherend
Thus,thepossibleseatingarrangementis
JHxxxxGLM
,
wherexistheEnglishspeakers.Mrs.F,Mrs.K,Mrs.N
andMrs.Ocanbearrangedinremaining4positionsin
4!differentwaysi.e.24ways.
CASEBAB:
GLMisnotattheotherend
ThenMrs.NmustseatnexttoMrs.M.Now,wehavea
groupoffourGLMNwhereMrs.GandMrs.Nspeak
English.Thus,thepossibleseatingarrangementis
JHxxxX
,wherexistheindividualEnglishspeakersand
XisthegroupoffourfemaleswithEnglishspeakersat
thebothends.Thus,thereare4!differentwaysi.e.24
ways.

CASEBB:
Mrs.MseatsleftandMrs.GseatsrighttoMrs.
Li.e.MLG

Then,Mrs.NmustseatnexttoMrs.M.Now,wehaveagroup
offourNMLGwhereMrs.GandMrs.NspeakEnglish.Thus,
thepossibleseatingarrangementis
JHxxxX
,wherexisthe
individualEnglishspeakersandXisthegroupoffourfemales
withEnglishspeakersatthebothends.Thus,thereare4!
differentwaysi.e.24ways.
Thus,totaldifferentpossibleseatingarrangementsare:
=6(caseAA)+24(caseABA)+24(caseABB)+24(caseBAA)+24
(caseBAB)+24(caseBB)
=126seatingarrangements

Thus,126distinctseatingarrangementsarepoosible.
Whatisthesmallestnumberwhichwhendividedby10leavesaremainder
of9,whendividedby9leavesaremainderof8,whendividedby8leavesa
remainderof7,whendividedby7leavesaremainderof6andsoonuntil
whendividedby2leavesaremainderof1?
Answer

Thesmallestsuchnumberis2519.

TheeasiestwayistofindtheLeastCommonMultiple(LCM)of2,3,4,5,
6,7,8and9.Andsubtract1fromit.

TheLCMof2,3,4,5,6,7,8and9isgivenby2520.Hence,therequired
numberis2519

Threefriendsdividedsomebulletsequally.Afterallofthemshot4bullets
thetotalnumberofbulletsremainingisequaltothebulletseachhadafter
division.Findtheoriginalnumberdivided.
Answer

18

Assumethatinitialtherewere3*Xbullets.

SotheygotXbulletseachafterdivision.

Allofthemshot4bullets.Sonowtheyhave(X4)bulletseach.


Butitisgiventhat,aftertheyshot4bulletseach,totalnumberofbullets
remainingisequaltothebulletseachhadafterdivisioni.e.X

Therefore,theequationis
3*(X4)=X
3*X12=X
2*X=12
X=6

Thereforethetotalbulletsbeforedivisionis=3*X=18

BrainTeaserNo:00114

Everydayinhisbusinessamerchanthadtoweighamountsfrom1kgto
121kgs,tothenearestkg.Whataretheminimumnumberofdifferent
weightsrequiredandhowheavyshouldtheybe?

Theminimumnumberis5andtheyshouldweigh1,3,9,27and81kgs
Replaceeachletterbyadigit.Eachlettermustberepresentedbythesame
digitandnobeginningletterofawordcanbe0.

ONE

ONE

ONE

+ONE

TEN
Answer

Usetrialanderror.0=1,N=8,E=2,T=7

182


182

182

+182

728
AmanisonasearchforAtlantisandcomesuponanislandwhereallthe
inhabitantsknowwhetherAtlantisisstillaroundornot.

However,alloftheinhabitantsareeitherFairiesorTrollsandtheyallusea
spelltoappearhumanoidsoyoucannottellwhichiswhich.AndtheFaries
alwaystellthetruthandtheTrollsalwayslie,butthereisaslight
complication,someoftheFairieshavegoneinsaneandalwayslieandsome
oftheTrollshavealsogoneinsaneandalwaystellthetruth.

Sohereisyourtask:youmustaskthefirstinhabitantthatyoucometoONE
questionandfromthatONEquestionyoumustdeterminewetherAtlantisis
stillaroundornot.

Whatisthequestionthatyoumustask?
Answer

Thereare2answerstoit:

AnswerI
"Isthestatementthatyouarereliableequivalenttothestatement
thatAtlantisisstillaround?"

AnswerII
"DoyoubelievethattheStatementthatyouareaFairyis
equivalenttothestatementthatAtlantisisstillaround?"

BrainTeaserNo:00276

Afrogstartsclimbing15feetwall.Eachhourheclimbs3feetandrests
for30minutes.Duringrest,heslipsback2feet.

Howmanyhoursdoesthefrogtaketoreachthetop?
Answer

19hours

Afrogclimbs1footper11/2hoursasduring30minutesrestheslipsback
2feet.Thiswayhewillclimb12feetin18hours.Innexthourhewillclimb
3morefeeti.e.hewillcomplete15feetin19hoursandwillreachthetopof
thewall.
Ifabeareats65poundsinfisheverydayEXCEPTevery6thdaywhichit
onlyeats45poundsoffish.

Ifthebearcontinuesthis,howmanypoundsoffishwilliteatin200days?
Submittedby:David
Answer

Thebearwilleat12,340poundsoffishin200days.

Itisgiventhatonevery6thdaybeareats45poundsoffishi.e.onday
number6,12,18,24,....192,198thebeareats45poundsoffish.

Totalnumberof6thdays=200/6=33(thebeareats45pounds)
Hence,thenormaldaysare=20033=167(thebeareats65pounds)

Thus,in200days,thebearwilleat
=(167)*(65)+(33)*(45)
=10855+1485
=12,340pounds
Youhave3pointslabelledA,BandC.Youthenhaveanother3points
labelled1,2and3.TheaimofthepuzzleistoconnectpointAwithpoint1,
2and3.PointBwithpoint1,2and3andpointCwithpoint1,2and3.

Nowwhileconnectingthepointsyouhavetofollowonerulethelines
cannotcrossovereachother.
ABC

123

PS:YoucanarrangethepointsinorderaslongasthelinesDONOTcross
overeachother.
Answer

Thereisnosolutiontoit,ifyouconsider2dimensions.Itisimpossibleto
joineachofpointsA,BandCwithpoints1,2and3withoutlinescrossing
eachother.

Thereissolution,ifyouconsider3dimensions.Consideracircularbaseand
alineperpendiculartoitpassingfromthecenter.Nowtakeany3points
alongtheperimeterofthecircularbaseaspoints1,2and3.Similarlytake
any3pointsalongtheperpendicularlineaspointsA,BandC.Nowitis
quitesimpletojoineachofpointsA,BandCwithpoints1,2and3without
anyofthelinescrossingeachother.

Theotherpossible3DstructureisPyramid.Takepoints1,2and3asa
verticesofthetriangularbaseandpointsA,BandCalongtheheightofthe
Pyramidwhichisperpendiculartothetriangularbaseandpassingthrough
theapex.

BrainTeaserNo:00477

Supposefivebalesofhayareweighedtwoatatimeinallpossibleways.
Theweightsinpoundsare110,112,113,114,115,116,117,118,120,
and121.

Howmuchdoeseachbaleweigh?
Submittedby:TravisLara

Answer

Theyweigh54,56,58,59,62pounds.

Let'sassumethattheweightoffivebalesareB1,B2,B3,B4andB5pounds
respectively.Also,B1<=B2<=B3<=B4<=B5

Itisgiventhatfivebalesofhayareweighedtwoatatimeinallpossible
ways.Itmeansthateachofthebaleisweightedfourtimes.

Thus,
4*(B1+B2+B3+B4+B5)=(110+112+113+114+115+116+117+
118+120+121)
4*(B1+B2+B3+B4+B5)=1156
(B1+B2+B3+B4+B5)=289pounds

Now,B1andB2mustaddto110astheyarethelightestone.
B1+B2=110

Similarly,B4andB5mustaddto121astheyaretheheaviestone.
B4+B5=121

Fromabovethreeequation,wegetB3=58pounds

Also,itisobviousthatB1andB3willaddto112thenextpossiblehigher
value.Similarly,B3andB5willaddto120thenextpossiblelowervalue.
B1+B3=112
B3+B5=120

SubstitutingB3=58,wegetB1=54andB5=62
From2&3equations,wegetB2=56andB4=59

Hence,theweightoffivebalesare54,56,58,59and62pounds.

Pintosays,"ThehorseisnotBlack."
Sandysays,"ThehorseiseitherBrownorGrey."
Andysays,"ThehorseisBrown."

Atleastoneistellingtruthandatleastoneislying.

Canyoutellthecolorofthehorse?
Answer

ThecolorofthehorsecanbeanycolorotherthanBlackandBrown.

IfthecolorofthehorseisBlackallarelying.

IfthecolorofthehorseisBrownallaretellingtruth.

Thus,
thehorseisneitherBlacknorBrown.

IfthecolorofthehorseisGreyPintoandSandyaretellingtruthwhereas
Andyislying.

IfthecolorofthehorseisotherthanBlack,BrownandGreyPintois
tellingtruthwhereasSandyandAndyarelying.

Youmusthavenoticedthatforthegivenconditions,Pintoisalwaystelling
truthwhereasAndyisalwayslying

BrainTeaserNo:00258

Threeconvictsarebroughtintothewarden'soffice.Hesayshecan
paroleoneofthemandtodecidewhichonehewillparolehetakesout5
hats(3redand2white).Hestandsbehindthemandplacesahatoneach
oneoftheirheadsandputstheothertworemaininghatsinadrawer.

Hetellstheprisionerstheycanlookattheothershatsandiftheycantell
whichhattheyhaveontheywillbetheonewhoisparoled.

Thefirstmanlooksattheothertwoandsays,"Idon'tknow."

Thesecondmanlooksattheothershatsandsays,"Idon'tknow."

Thethirdmanwhoisblindsays,"EventhoughIhavenotthegiftof
sightIcantellbywhattheothershavesaidthatthecolorofmyhatis..."

Whatcoloristheblindmanshatandhowdoesheknow?
Submitted

Answer

Thecolorofblindman'shatisRed.

ItissurethatthefirstmansaweitherbothRedhatsoroneWhitehatand

Hello
sajeesh

myAnswe

myFavou

ModifyPe
Info

oneRedhat.Thereare6suchpossibilities:
1)RRR

2)RRW

3)RWR

4)WRR

5)WRW

6)WWR
Inallabovepossibilities,thefirstmanwon'tbesureofthecolorofhis
hat.

Now,thesecondmanknowsthatthefirstmansaweitherbothRedhats
oroneWhitehatandoneRedhat.And,healsoknowsthatitsoneofthe
above6possibilities.(likeweknow))Buthesays,"Idon'tknow".That
meansthat(2)and(5)arenotthepossibilitiesasineithercasehewould
besureofthecolorofhishat(Red)byjustlookingatthethirdman's
colorofhat(White).

Now,theblindmanknowsthattherearejust4possibilities(1),(3),(4),
(6)andinall,thecolorofhishatisRed.

Submit
Answer

Subscribe

Logout

TopofF

Bottomof

JigsawPu

JointheD

MarblesG

BallsGam

Towersof

ThinkNu

FindAD

To
p
of
Fo
rm

BrainTe

Bottom

ThreeGold(G)coins,threeSilver(S)coinsandthreeCopper(C)coinsare
arrangedinasinglerowasfollow:
GSCGSCGSC
Only2
adjacentunlikecoins
canbemovedatanyonetime.
Themovedcoinsmustbeincontactwithatleastoneothercoinin
line.i.e.nopairofcoinsistobemovedandplacedawayfromthe
remainingones.
Nocoinpairscanbereversedi.e.aSCcombinationmustremainin
thatorderinitsnewpositionwhenitismoved.
Whatistheminimumnumberofmovesrequiredtogetallthecoinsin
followingorder?
CCCSSSGGG
Showallmoves.
Answer

Minimumnumberofmovesare8.
O
r
d
e
r
Move
of
C
oi
n
s

G S G S C G S C

G S C G S C G S C
C

S C G S C

G S C

G S C C S G S C

G S C C

G S C C C S S

C C C S S

G G S G

C C C S S S G G G

G S C

C S G
C S G

S C S G

G
G

Aflyisflyingbetweentwotrains,eachtravellingtowardseachotheronthe
sametrackat60km/h.Theflyreachesoneengine,reversesitself
immediately,andfliesbacktotheotherengine,repeatingtheprocesseach
time.

Theflyisflyingat90km/h.Iftheflyflies180kmbeforethetrainsmeet,
howfarapartwerethetrainsinitially?
Answer

Initially,thetrainswere240kmapart.

Theflyisflyingatthespeedof90km/handcovers180km.Hence,thefly
fliesfor2hoursaftertrainsstarted.

It'sobviousthattrainsmet2hoursaftertheystartedtravellingtowardseach
other.Also,trainsweretravellingatthespeedof60km/h.So,eachtrain
traveled120kmbeforetheymet.

Hence,thetrainswere240kmapartinitially.

Whatistheminimumnumberofnumbersneededtoformeverynumber
from1to7,000?

Example:Toform4884,youwouldneed24s&28s.4822requiresa4,a8,
&22s,butyouwouldnotcountthenumbersagainthatyouhadalready
countedfrommaking4884.
Answer


36

Youwillneed3ofnumbers0,7,8&9,&4ofnumbers16.
AdrinksmachineoffersthreeselectionsTea,CoffeeorRandom(Eithertea
orCoffee)butthemachinehasbeenwiredupwronglysothateachbutton
doesnotgivewhatitclaims.

Ifeachdrinkcosts50p,howmuchminimummoneydoyouhavetoputinto
themachinetoworkoutwhichbuttongiveswhichselection?
Submitted
Answer

Youhavetoputjust50p.

Put50pandpushthebuttonforRandom.Thereareonly2possibilities.It
willgiveeitherTeaorCoffee.
IfitgivesTea,thenthebuttonnamedRandomisforTea.Thebutton
namedCoffeeisforRandomselection.AndthebuttonnamedTeais
forCoffee.
IfitgivesCoffee,thenthebuttonnamedRandomisforCoffee.The
buttonnamedTeaisforRandomselection.Andthebuttonnamed
CoffeeisforTea.
Thus,youcanmakeoutwhichbuttonisforwhatbyputtingjust50pand
pressingRandomselectionfirst.
Youhave13ballswhichalllookidentical.Alltheballsarethesameweight
exceptforone.Usingonlyabalancescale,canfindtheoddoneoutwith
only3weighings?

Isitpossibletoalwaystelliftheoddoneoutisheavierorlighterthanthe
otherballs?
Submittedby:BrettHurrell
Answer

Itisalwayspossibletofindoddballin3weighingsandinmostofthecases
itispossibletotellwhethertheoddballisheavierorlighter.Onlyinone
case,itisnotpossibletotelltheoddballiswhetherheavierorlighter.
1. Take8ballsandweigh4against4.

2.

Oneofthese8ballsistheoddone.Nametheballsonheaviersideof
thescaleasH1,H2,H3andH4.Similarly,nametheballsonthe
lightersideofthescaleasL1,L2,L3andL4.EitheroneofH'sis
heavieroroneofL'sislighter.Weigh(H1,H2,L1)against(H3,H4,
X)whereXisoneballfromtheremaining5ballsinintialweighing.
Ifbothareequal,oneofL2,L3,L4islighter.WeighL2against
L3.
Ifbothareequal,L4istheoddballandislighter.
IfL2islight,L2istheoddballandislighter.
IfL3islight,L3istheoddballandislighter.

3.

Ifbotharenotequal,gotostep2
Ifbothareequal,gotostep3

If(H1,H2,L1)isheaviersideonthescale,eitherH1orH2is
heavier.WeightH1againstH2
Ifbothareequal,thereissomeerror.
IfH1isheavy,H1istheoddballandisheavier.
IfH2isheavy,H2istheoddballandisheavier.

If(H3,H4,X)isheaviersideonthescale,eitherH3orH4is
heavierorL1islighter.WeightH3againstH4
Ifbothareequal,L1istheoddballandislighter.
IfH3isheavy,H3istheoddballandisheavier.
IfH4isheavy,H4istheoddballandisheavier.

Oneoftheremaining5ballsistheoddone.NametheballsasC1,C2,
C3,C4,C5.Weight(C1,C2,C3)against(X1,X2,X3)whereX1,X2,
X3areanythreeballsfromthefirstweighingof8balls.
Ifbothareequal,oneofremaining2ballsistheoddi.e.either
C4orC5.WeighC4withX1
Ifbothareequal,C5istheoddball.Butyoucannottell
whetheritisheavierorlighter.
IfC4isheavy,C4istheoddballandisheavier.
IfC4islight,C4istheoddballandislighter.

If(C1,C2,C3)isheavierside,oneofC1,C2,C3istheoddball
andisheavier.WeighC1andC2.
Ifbothareequal,C3istheoddballandisheavier.
IfC1isheavy,C1istheoddballandisheavier.
IfC2isheavy,C2istheoddballandisheavier.

If(C1,C2,C3)islighterside,oneofC1,C2,C3istheoddball
andislighter.WeighC1andC2.
Ifbothareequal,C3istheoddballandisheavier.
IfC1islight,C1istheoddballandislighter.
IfC2islight,C2istheoddballandislighter.

Howmanysquaresarethereina5inchby5inchsquaregrid?Notethatthe
gridismadeupofoneinchbyoneinchsquares.
Submittedby:KristinMonroe
Answer

Thereare55squaresina5by5grid.

Thereare25squaresofonegrid.
Thereare16squaresoffourgridsi.e.2by2
Thereare9squaresofninegridsi.e.3by3
Thereare4squaresofsixteengridsi.e.4by4
Thereis1squareoftwentyfivegirdsi.e.5by5

Hence,therearetotal25+16+9+4+1=55squares.

Youmusthavenoticedonethingthattotalnumbersquarespossibleofeach
sizeisalwaysaperfactsquarei.e.25,16,9,4,1

ForagridofNbyN,thepossiblenumberofsquaresare
2
2
2
2
2
2
2
=N
+(N1)
+(N2)
+(N3)
+.........+3
+2
+1

2
For1by1grid,totalsquares=1
=1
2
2
For2by2grid,totalsquares=2
+1
=5
2
2
2
For3by3grid,totalsquares=3
+2
+1
=14
2
2
2
2
For4by4grid,totalsquares=4
+3
+2
+1
=30
2
2
2
2
2
For5by5grid,totalsquares=5
+4
+3
+2
+1
=55
Fivehorsesranintherace.

Therewerenoties.
Sikandardidnotcomefirst.
Starwasneitherfirstnorlast.
MughalGlorycameinoneplaceafterSikandar.
Zozowasnotsecond.
RangilawastwoplacebelowZozo.

Inwhatorderdidthehorsesfinish?
Answer

It'ssimple.

Let'sfindthepossibleplaceshorsescanfinish.Possibilitiesare:
Sikandar2,3,4(not5thasMughalGlorycameoneplaceafterhim)
Star2,3,4
MughalGlory3,4,5
Zozo1,3(not4th&5thasRangilaistwoplaceafterhim)
Rangila3,5

Sotheresultis:
1Zozo
2Star
3Rangila
4Sikandar
5MughalGlory

Ifyouaddedtogetherthenumberof2'sineachofthefollowingsetsof
numbers,whichsetwouldcontainthemost2's:1333,334666,or667999?
Answer

1333

Thereasonwhyisbecause200299eachbeginswitha2!

Ifonepersonsendstheemailtotwofriends,askingeachofthemtocopy
themailandsendittotwooftheirfriends,thoseinturnsendittotwoof
theirfriendsandsoon.

Howmanyemailswouldhavebeensentbythetimeitdid30sets?

Answer

2147483646

Firstpersonsentthemailto2persons.Those2sentthemailto2persons
each,total4persons.Now,those4personsentmailtototal8persons,then8
to16persons,16to32personsandsoon....Hence,itaseriesof2,4,8,16,
32upto30numbers

ItisaGeometricserieswithcommonratio2andfirstnumberisalso2.
n
SummationofsuchseriesisgivenbyA*(R
1)/(R1)where
A=Firstterm
R=CommonRatio
n=totalnumbers

Sototalnumberoftimesmailsentbythetimeitdid30sets
30
=2*(2
1)/(21)
=2*(10737418241)
=2*1073741823
=2147483646

BrainTeaserNo:00347

Attheentrancetoamembersclubstandsastrangerseekingadmission.A
friendtoldhimthatit'seasytogetin.Youjusthavetoansweraquestion
corrcetly!Answeringwrong,however,willresultinbeingshot!

Tolivealittlelonger,themanwaitsinabackalleyneartheentrancefor
peopletogoin.Afterawhileamancomestotheentrance.Thedoor
wardenaskshim:"Twelve?"towhichhereplies"Six!"andgoesin.

"That'seasy."ourfriendthinks,buthewaitsalittlelonger.

Anothermancomestothedoor."Six?"thedoorwardenasks,towhichhe
replies"Three!"andgoesin.

"That'stoogoodtobetrue"ourfriendthinks,andhewasright.Because,
whenasked"Four?",heanswered"Two!"andwasfounddeadinthe
alley.


Whatwasthecorrectanswer?
Submittedby:MilindGadagkar
Answer

Thecorrectanswerwas"Four".

Theansweristhenumberoflettersinthewordspokenbythedoorwarden.

"Twelve"contains"Six"lettersi.e.T,W,E,L,V,E
"Six"contains"Three"lettersi.e.S,I,X
Similarly,"Four"contains"Four"lettersi.e.F,O,U,R
Thereisaperfectsphereofdiameter40cms.restingupagainstaperfectly
straightwallandaperfectlystraightfloori.e.thewallandthefloormakea
perfectrightangle.

Canaperfectsphereofdiameter7cms.passthroughthespacebetweenthe
bigsphere,thewallandthefloor?Supportyouranswerwithvalid
arguments.Don'tsubmitjust"Yes"or"No".
Submit
Answer

Users
Answer(23)

BrainVista
Answer

PuzzleA
Friend

Addto
Favourite

Forthesakeofsimplicity,considertwodimensioni.eviewsphereasa
twodimensionalcirclewithdiameter40cms.


FromFigureI
,(40cmsdiametersphere)
2
2
2
OC
=OD
+CD

2
2
2
OC
=20
+20

OC=28.28427cms

Also,XistheclosestpointtooriginOonthesphere.
CX=20cms(radius)
OX=OCCX
OX=28.2842720
OX=8.28427cms

FromFigureII
,(7cmsdiametersphere)
2
2
2
OP
=OQ
+PQ

2
2
2
OP
=(3.5)
+(3.5)

OP=4.94974cms

Also,YisthefarthestpointtooriginOonthesphere.
PY=3.5cms(radius)
OY=OP+PY
OY=4.94974+3.5
OY=8.44974cms

Now,asOY>OXi.e.smallersphererequiresmorespacethanthespace
available.Hence,smallersphereof7cmsdiametercannotpassthrough
thespacebetweenthebigsphere,thewallandthefloor.

Thepuzzlecanbesolvedbyanothermethod.
DrawalinetangenttothebigsphereatthepointXsuchthatXisthe
closestpointtotheoriginOonsphere.ThetangetwillcutXandYaxes
atAandBrespectivelysuchthatOA=OB.[SeeFigIII]Fromabove,
OX=8.28427cms.

FromtherightangletriangleOAB,wecandeductthat
OA=OB=11.71572cms
AB=16.56854cms

Now,thediameteroftheinscribedcircleofrightangletriangleisgiven

byd=a+bcwherea<=b<c

Themaximumpossiblediameterofthecirclewhichcanpassthroughthe
spacebetweenthebigsphere,thewallandtheflooris
=OA+OBAB
=11.71572+11.7157216.56854
=6.86291cms

Hence,thespherewith7cmsdiametercannotpassthroughthespace
betweenthebigsphere,thewallandthefloor.

Submit
Users
BrainVista
PuzzleA

Add
Answer
Answer(23)
Answer
Friend
Sarikamultiplied414bycertainnumberandobtained69958astheanswer.
Butshefoundthatthereissomeerrorintheanswerboththe9sinthe
answerarewrongandalltheotherdigitsarecorrect.

Canyoufindthecorrectanswer?
Answer

Thecorrectansweris60858.

Ifyoudivide69958by414,youwillget168.98.Hence,assumesomethree
digitnumberandmultiplyitby414anduse6**58astheanswer.

Assumethreedigitnumbersuchthat
***

414

***

***0

***00

6**58
Itisobviousthatthelastdigitoftheassumednumbermustbe7.
**7

414

**8

**70

**800

6**58
Now,thesecondlastdigitoftheassumednumbermustbe4or9.Also,the
firstdigitoftheassumednumbermustbe1asthefirstdigitoftheansweris
6.Usingtrialanderrorforabovetwoconditions,theansweris
147

414

588

1470

58800

60858

Findtheleastnumberwhichwhendividedby35,leavesremainder25when
dividedby45,leavesremainder35andwhendividedby55,leaves
remainder45.

Answer

3455

TheanswerisLCMof(35,45,55)minus10.
LCMof(35,45,55)is3465.
Hence,theansweris3455.
TheratioofBoystoGirlsis6:4.60%oftheboysand40%ofthegirlstake
lunchinthecanteen.What%ofclasstakeslunchincanteen?
Answer

Assumethereare6Xboysand4XGirls

TotalStudentstakinglunchincanteen
=(6X)(60/100)+(4X)(40/100)
=36(X/10)+16(X/10)
=52(X/10)

Totalstudentsare=6X+4X=10X

%ofclasstakinglunchincanteen
=((52X/10)*100)/10X
=52%
Inthefollowingmultiplication,certaindigitshavebeenreplacedwith
asterisks(*).Replacealltheasteriskssuchthattheproblemholdstheresult.
**7

X3**

*0*3

*1*

*5*

*7**3
Answer

Asimpleone.
117

X319

1053

117

351

37323
Howlongwouldittakeyoutocount1billionorallyifyoucouldcount200
everyminuteandweregivenadayoffeveryfouryears?

Assumethatyoustartcountingon1January2001.
Submitte
Answer

9Years,187Days,5Hours,20minutes

Asyoucancount200perminute,tocount1billionyourequire
=1,000,000,000/200minutes
=5,000,000minutes
=83,333.3333hours
=3,472.2222days
=9.512937years
=9Years,187Days,5Hours,20minutes

NotethatadayoffeveryfouryearwillbeaLeapday.Hence,noneedto
considerleapyear.
d
FivestudentsAkash,Chintan,Jignesh,MukundandVenkyappearedfor
anexam.Thereweretotalfivequestionstwomultiplechoice(a,borc)and
threetrue/falsequestions.Theyansweredfivequestionseachandanswered
asfollow.
IIIIIIIVV

ChintancbTrueTrueFalse

AkashccTrueTrueTrue

JigneshacFalseTrueTrue

MukundbaTrueTrueFalse

VenkybbTrueFalseTrue

Also,notwostudentsgotthesamenumberofcorrectanswers.

Canyoutellwhicharethecorrectanswers?Whataretheirindividualscore?
Answer

Thecorrectanswersareb,a,True,FalseandFalse.Also,thescoresare
Jignesh(0),Akash(1),Chintan(2),Venky(3)andMukund(4).

Asnotwostudentsgotthesamenumberofcorrectanswers,thetotal
numberofcorrectanswersmustbeeither15(1+2+3+4+5)or10
(0+1+2+3+4).

Let'sfindoutthemaximumnumberofcorrectanswerspossiblefromthe
answersgivenbythem.
ForQuestionI=2(borc)
ForQuestionII=2(borc)
ForQuestionIII=4(True)
ForQuestionIV=4(True)

ForQuestionV=3(True)

Thus,themaximumnumberofcorrectanswerspossibleare15(2+2+4+4+3)
whichmeansthatAkashwouldhavegivenallcorrectanswersasonlyhe
answeredTrueforquestionsIII,IVandV.ButthenChintanandJignesh
wouldhaveexactly3correctanswers.Andalso,MukundandVenkywould
have2correctanswers.So
noonegotallfivecorrect
.Onecanalsoarrive
atthisconclusionbytrialanderror,butthatwouldbebitlengthy.

Now,itisclearthattotalnumberofcorrectanswersare10(0+1+2+3+4).
QuestionsIIIandIVbothcannotbeFalse.Ifso,totalnumberofcorrect
answerswouldnotbe10.Sothestudentwhogotallwrongcannotbe
Chintan,AkashandMukund.

IfVenkygotallwrong,thenChintan,JigneshandMukundeachwouldhave
atleast2correctanswers.ItmeansthatAkashwouldhavetobethestudent
withonlyonecorrectanswerandthecorrectanswersforquestionsIandII
wouldbeaandarespectively.Butthenthetotalnumberofcorrectanswers
wouldbe1(a)+1(a)+1(False)+4(True)+2(Flase)=9.

Thus,Jigneshisthestudentwithallwronganswers.Thecorrectanswersare
b,a,True,FalseandFalse.Also,thescoresareJignesh(0),Akash(1),
Chintan(2),Venky(3)andMukund(4).

Elevenboysandgirlswaittotaketheirseatsinthesamerowinamovie
theater.Thereareexactly11seatsintherow.

Theydecidedthatafterthefirstpersonsitsdown,thenextpersonhastosit
nexttothefirst.Thethirdsitsnexttooneofthefirsttwoandsoonuntilall
elevenareseated.Inotherwords,nopersoncantakeaseatthatseparates
him/herfromatleastoneotherperson.

Howmanydifferentwayscanthisbeaccomplished?Notethatthefirst
personcanchooseanyofthe11seats.
Answer

Thereare1024differentways.

ThisisthetypeofBrainTeaserthatcanbesolvedusingthemethodof

induction.

Ifthereisjustaonepersonandoneseat,thatpersonhasonlyoneoption.

Iftherearetwopersonsandtwoseats,itcanbeaccomplishedin2different
ways.

Iftherearethreepersonsandthreeseats,itcanbeaccomplishedin4
differentways.Rememberthatnopersoncantakeaseatthatseparates
him/herfromatleastoneotherperson.

Similarly,fourpersonsandfourseatsproduce8differentways.Andfive
personswithfiveseatsproduce16differentways.

Itcanbeseenthatwitheachadditionalpersonandseat,thedifferentways
increasebythepoweroftwo.Forsixpersonswithsixseats,thereare32
differentways.

(N1)
ForanynumberN,thedifferentpossiblewaysare2

10
Thus,for11personsand11seats,totaldifferentwaysare2
i.e.1024
ThesecretagentXemailedacodewordtohisheadoffice.Theyare"AIM
DUEOATTIEMOD".Butfourofthesefivewordsarefakeandonlyone
containstheinformation.

TheagentXalsomailedasentenceasaclueifItellyouanyonecharacter
ofthecodeword,youwouldbeabletotellthenumberofvowelsinthecode
word.

Canyoutellwhichisthecodeword?

Answer

ThecodewordisTIE.

IfyouweretoldanyonecharacterofMOD,thenyouwouldnotbeableto
determinewhetherthenumberofvowelsareoneortwo.e.g.ifyouwere
toldM,therearetwowordswithMAIMwith2vowelsandMODwith1
vowel.Soyouwouldnotbeabletosaythenumberofvowels.Same

argumentscanbegivenforcharactersOandD.

Hence,thewordwithanyoneofM,OorDisnotacodewordi.e.AIM,
DUE,OATandMODarenotthecodeword.Thus,TIEisthecodeword.
T:twowordsTIEandOAT,bothwith2vowels
I:twowordsTIEandAIM,bothwith2vowels
E:twowordsTIEandDUE,bothwith2vowels.

BrainTeaserNo:00361

FourmenAbraham,Bobby,ClintonandDenialarestandingina
straightline.
1. Onemanisfair,handsomeandunscarred.
2. Twomenwhoarenotfair,areeachstandingnexttoAbraham.
3. Bobbyistheonlymanstandingnexttoexactlyonehandsomeman.
4. Clintonistheonlymannotstandingnexttoexactlyonescarred
man.
Whoisfair,handsomeandunscarred?

Answer

Clintonisfair,handsomeandunscarred.

From(2),boththemenstandingnexttoAbrahamarenotfair.Also,exactly
onemanisfair,handsomandunscarred.Hence,therearetwocases:

Case1::?(N,?,?):Abraham(Y,Y,N):?(N,?,?):?(?,?,?)
Case2::?(N,?,?):Abraham(?,?,?):?(N,?,?):?(Y,Y,N)

NotetherepresentationName(Fair,Handsome,Scarred)."Y"standsfor
Yesand"N"stabdsforNo.Abraham(Y,Y,N)meansAbrahamisFair,
HandsomeandUnscarred.

ItisclearthateitherAbrahamorthemanattheextremerightisfair,
handsomeandunscarred.

From(4),itisdeducedthatClintonisstandingnexttounscarredmanand
eachoftheothermenstandingnexttoexactlyonescarredman.


Case1::Clinton(N,?,N):Abraham(Y,Y,N):?(N,?,Y):?(?,?,Y)
Case2::?(N,?,Y):Abraham(?,?,Y):?(N,?,N):Clinton(Y,Y,N)

From(3),Bobbyistheonlymanstandingnexttoexactlyonehandsome
man.ButinCase1,Clintonisstandingnexttoexactlyonehandsomeman.
Hence,Case1isnotpossibleandCase2isthecorrectone.

Case2::?(N,?,Y):Abraham(?,?,Y):?(N,?,N):Clinton(Y,Y,N)

Againfrom(3)and(4),thereare2possibilitiesasshownbelow.

Case2a::Denial(N,N,Y):Abraham(?,N,Y):Bobby(N,N,N):Clinton
(Y,Y,N)
Case2b::Bobby(N,N,Y):Abraham(?,Y,Y):Denial(N,N,N):Clinton
(Y,Y,N)

Thus,Clintonisfair,handsomeandunscarred.Also,Abrahammaybeeither
fairornotfair.
AnorangecoloredglasshasOrangejuiceandwhitecoloredglasshasApple
juicebothofequalvolumes.50mloftheorangejuiceistakenandpoured
intothewhiteglass.Afterthatsimilarly,50mlfromthewhiteglassispoured
intotheorangeglass.

Ofthetwoquantities,theamountofapplejuiceintheorangeglassandthe
amountoforangejuiceinthewhiteglass,whichoneisgreaterandbyhow
much?
Answer

Thetwoquantitiesareequal.

Solveitbytakingexample.Let'sassumethatbothglassescontain450mlof
juiceeach.

Now,50mloftheorangejuiceistakenandpouredintotheWhiteglass.
Hence,orangecoloredglasscontains400mlofOrangejuiceandwhiteglass
contains450mlofApplejuiceand50mlofOrangejuicei.e.totalof500ml
fromwhiteglasscontains450mlofApplejuiceand50mlofOrangejuice.
Itmeansthat
every50mlfromwhiteglasscontains45mlofApplejuice

and5mlofOrangejuice.

Similary,50mlofjuicefromwhiteglassispouredintoorangeglass.Now
this50mlisnotapureapplejuice.Itcontains45mlofApplejuiceand5ml
ofOrangejuice.

Hence,Orangeglasscontains405mlofOrangejuiceand45mlofApple
juice.Similary,whiteglasscontains405mlofApplejuiceand45mlof
Orangejuice.

Orange
Glass
Orange
Juice

Initially

White
Glass

Apple
Juice

Orange
Juice

Apple
Juice

450ml

0ml

0ml

450ml

50mlfromOrangeGlassis
400ml
pouredintoWhiteGlass

0ml

50ml

450ml

50mlfromWhiteGlassis
pouredintoOrangeGlass

45ml

45ml

405ml

405ml

Nowitisclearthattheamountofapplejuiceintheorangeglassandthe
amountoforangejuiceinthewhiteglassarethesame.

P.S.Hereweassumed450mlasinitialquantityinboththeglassesjustfor
simplicity.Youcantrythesamebyassuminganyothernumber.Butthe
answeristhesame.

BrainTeaserNo:00433

Annie,Bunnie,CandyandDinavisitedEdyon14thFebruary.
1. Thetimeofeachvisitwasasfollows:
Annieat8:00
Bunnieat9:00
Candyat10:00
Dinaat11:00
EachtimementionedabovemaybeeitherAMorPM.

2.
3.
4.

CandydidnotvisitEdybetweenBunnieandDina.
AtleastonefemalevisitedEdybetweenAnnieandBunnie.
AnniedidnotvisitEdybeforebothCandyandDina.

CanyoutellatwhattimedidtheyindividuallyvisitEdy?

Answer

Bunnie(9:00AM)Dina(11:00AM)Annie(8:00PM)Candy
(10:00PM)

Fromthegivendata,itisclearthatatleastonefemalevisitedEdyinthe
morningandatleastonefemalevisitedEdyintheevening.Also,from(4),
AnniedidnotvisitEdyfirst.ItmeansthatAnnievisitedEdyat8:00PM

From(3),BunniemusthavevisitedEdyat9:00AM.Also,eitherCandyor
DinaorbothvisitedEdyinthemorning.

Butfrom(2),onlyDinamusthavevisitedEdyinthemorningat11:00AM
andhence,CandyvisitedEdyat10:00PM.

Theorderofvisitsmustbe:
Bunnie(9:00AM)Dina(11:00AM)Annie(8:00PM)Candy(10:00PM)

Intrainingforacompetition,youfindthatswimmingdownstream(withthe
current)inariver,youcanswim2milesin40minutes,&upstream(against
thecurrent),youcanswim2milesin60minutes.

Howlongwouldittakeyoutoswimamileinstillwater?
Answer

Youareabletoswimdownstreamat3milesanhour,&upstreamat2miles
anhour.Thereisadifferenceof1mileanhour,whichistheriverhelping
youin1direction,&slowingyouintheotherdirection.

Averagethe2rates,&youhavetheratethatyoucanswiminstillwater,
whichis2.5milesanhour.

Youcanthusswimamileinstillwaterin24minutes.
Father'sageisthreeyearsmorethanthreetimestheson'sage.Afterthree
years,father'sagewillbetenyearsmorethantwicetheson'sage.

Whatisthefather'spresentage?

Answer

Letson'spresentageisXyears.
Hence,father'spresentageis(3X+3)years.

After3years,son'sagewillbe(X+3)years.
andfather'sagewillbe(3X+6)years.

Butgiventhatafter3yearsfather'sagewillbetenyearsmorethantwice
theson'sage.
(3X+6)=2*(X+3)+10
3X+6=2X+16
X=10

Therefore,father'spresentageis33years.

Sub
mit
Ans
wer

Users
Answer(17)

Brain
Teaser
No:
00570

A
is
th
e

Brai

fat
he
r
of
tw
o
ch
ild
re
n
B
an
d
D
w
ho
ar
e
of
dif
fer
en
t
se
xe
s.
C
is
B'
s
sp
ou
se.
E
is
th
e
sa

m
e
se
x
as
D.
B
an
d
C
ha
ve
th
e
tw
o
ch
ild
re
n
F
w
ho
is
th
e
sa
m
e
se
x
as
B
an
d
G
w
ho

is
th
e
sa
m
e
se
x
as
C.
E'
s
m
ot
he
r,
H
w
ho
is
m
arr
ie
d
to
L,
is
th
e
sis
ter
of
D'
s
m
ot
he
r,

M.
E
an
d
E'
s
sp
ou
se,
I
ha
ve
tw
o
ch
ild
re
n
J
an
d
K
w
ho
ar
e
th
e
sa
m
e
se
x
as
I.

Notethat
no
persons

have
married
more
than
once.
Also,
thereare
more
number
of
females
than
males.
Canyou
tellhow
many
females
are
there?

Answer

Thereare7femalesand6males.

Assumethattherearefoursexesmale,female,XandY.Preparethe
followingtreebasedonthedatagiven:

sister

L(m)H(f)M(f)A(m)

||

||

E(x)I(y)D(x)B(y)C(x)

||

||

J(y)K(y)F(y)G(x)

Itisclearthattherearealtogether13persons2males,2females,4Xsand
5Ys.

Itisgiventhattherearemorenumberoffemalesthanmale.Hence,allY
mustrepresentfemale.Thus,thereare7femalesand6males.
Apositiveintegerthat,whenaddedto1000givesasumwhichisgreater
thanwhenmultipliedby1000.

Findthepositiveinteger.
Answer

Thepositiveintegeris1.

Sumof1and1000=1+1000=1001
Multiplicationof1and1000=1*1000=1000

Thus,sumof1and1000isgreaterthanthemultiplicationof1and1000.
Mr.D'souzahasboughtfourcarsMerc,Honda,Ford,Zenaspresentsfor
hissons'birthdays,allofwhicharenextweek.Giventhefollowing
information,whatwilleachsonget?

AlanwillnotgettheHondaunlessBarrygetstheMercandDenzilgetsthe
Ford.BarrywillnotgettheFordunlessCarlgetstheZenandAlangetsthe
Merc.DenzilwillnotgettheZenunlessAlangetstheHondaandBarrygets
theMerc.AlanwillnotgettheMercunlessCarlgetstheZenandDenzil
getstheFord.BarrywillnotgettheMercunlessAlangetstheZenand
DenzilgetstheFord.AlanwillnotgettheZenunlessBarrygetstheHonda
andCarlgetstheMerc.CarlwillnotgettheZenunlessBarrygetsthe
HondaandAlangetstheFord.AlanwillnotgettheFordunlessBarrygets
theZenandDenzilgetstheHonda.CarlwillnotgettheMercunlessDenzil
getstheHonda.
Answer

Let'sputgiven9informationinatable.Thepersonin
BoldFont
willnotget
thecorrespondingcarunlessthepersonsinNormalFontgetthe
correspondingcars.Also,thepersonwill
Italics
willgettheremainingcar.
Merc Honda

Ford

Zen

1 Barry

Alan

Denzil

Carl

Denzil Barry

Carl

Alan

3 Barry

Alan

Carl

Denzil

4 Alan

Barry

Denzil

Carl

5 Barry

Carl

Denzil

Alan

Barry

Denzil

Alan

7 Denzil

Barry

Alan

Carl

Carl

Denzil

Alan

Barry

Carl

Denzil

Carl

Now,let'sassumethatAlangetstheMerc.Thenfrom(4),Barrygetsthe
Honda,DenzilgetstheFordandCarlgetstheZen.Butfrom(7),Carlwill
notgettheZenunlessBarrygetstheHondaandAlangetstheFord.Thus,it
contradictstheoriginalassumption.Hence,AlanwillnotgettheMerc.

Let'sassumethatAlangetstheHonda.Thenfrom(1),BarrygetstheMerc,
DenzilgetstheFordandCarlgetstheZen.Butfrom(5)orfrom(7),it
contradictstheoriginalassumption.Hence,AlanwillnotgettheHonda.

Let'sassumethatAlangetstheFord.Thenfrom(8),CarlgetstheMerc,
DenzilgetstheFordandBarrygetstheZenwhichdoesnotcontradictany
ofthestatement.

Similaly,youcanassumethatAlangetstheZen.(whichiscontradictoryto
(9))

Hence,AlangetstheFord,BarrygetstheZen,CarlgetstheMercand
DenzilgetstheHonda.
Yesterdayinaparty,IaskedMr.Shahhisbirthday.Withamischievous

glintinhiseyeshereplied."ThedaybeforeyesterdayIwas83yearsoldand
nextyearIwillbe86."

CanyoufigureoutwhatistheDateofBirthofMr.Shah?Assumethatthe
currentyearis2000.
Answer

Mr.Shah'sdateofbirthis31December,1915

Todayis1January,2000.Thedaybeforeyesterdaywas30December,1999
andMr.Shahwas83onthatday.Todayi.e.1January,2000heis84.On
31December2000,hewillbe85andnextyeari.e.31December,2001he
willbe86.Hence,thedateofbirthis31December,1915.

ManypeopledothinkofLeapyearanddateofbirthas29thFebruaryas
2000istheLeapyearandthereisdifferenceof3yearsinMr.Shah'sage.
Butthatisnottheanswer.

BrainTeaserNo:00800

Thereare4mathematiciansBrahma,Sachin,PrashantandNakul
havinglunchinahotel.Suddenly,Brahmathinksof2integernumbers
greaterthan1andsays,"Thesumofthenumbersis..."andhewhispers
thesumtoSachin.Thenhesays,"Theproductofthenumbersis..."and
hewhisperstheproducttoPrashant.Afterthatfollowingconversation
takesplace:

Sachin:Prashant,Idon'tthinkthatweknowthenumbers.
Prashant:Aha!,nowIknowthenumbers.
Sachin:Oh,nowIalsoknowthenumbers.
Nakul:Now,Ialsoknowthenumbers.

Whatarethenumbers?Explainyouranswer.
Submitted

Answer

Thenumbersare4and13.

AsSachinisinitiallyconfidentthatthey(i.e.heandPrashant)don'tknow
thenumbers,wecanconcludethat
1)Thesummustnotbeexpressibleassumoftwoprimes,otherwiseSachin
couldnothavebeensureinadvancethatPrashantdidnotknowthe
numbers.
2)Theproductcannotbelessthan12,otherwisetherewouldonlybeone
choiceandPrashantwouldhavefiguredthatoutalso.

Suchpossiblesumare11,17,23,27,29,35,37,41,47,51,53,57,59,65,
67,71,77,79,83,87,89,93,95,97,101,107,113,117,119,121,123,125,
127,131,135,137,143,145,147,149,155,157,161,163,167,171,173,
177,179,185,187,189,191,197,....

Let'sexaminethemonebyone.

Ifthesumoftwonumbersis11,Sachinwillthinkthatthenumberswould
be(2,9),(3,8),(4,7)or(5,6).

Sachin:"As11isnotexpressibleassumoftwoprimes,Prashantcan'tknow
thenumbers."

Here,theproductwouldbe18(2*9),24(3*8),28(4*7)or30(5*6).Inallthe
casesexceptforproduct30,Prashantwouldknowthenumbers.

ifproductoftwonumbersis18:
Prashant:"Sincetheproductis18,thesumcouldbeeither11(2,9)or9(3,6).
Butifthesumwas9,SachinwouldhavededucedthatImightknowthe
numbersas(2,7)isthepossibleprimenumberspair.Hence,thenumbers
mustbe2and9."(ORinotherwords,9isnotinthePossibleSumList)

ifproductoftwonumbersis24:
Prashant:"Sincetheproductis24,thesumcouldbeeither14(2,12),11(3,8)
or10(4,6).But14and10arenotinthePossibleSumList.Hence,the
numbersmustbe3and8."

ifproductoftwonumbersis28:
Prashant:"Sincetheproductis28,thesumcouldbeeither16(2,14)or
11(4,7).But16isnotinthePossibleSumList.Hence,thenumbersmustbe
4and7."


ifproductoftwonumbersis30:
Prashant:"Sincetheproductis30,thesumcouldbeeither17(2,15),
13(3,10)or11(5,6).But13isnotinthePossibleSumList.Hence,the
numbersmustbeeither(2,15)or(5,6)."Here,Prashantwon'tbesureofthe
numbers.

Hence,Prashantwillbesureofthenumbersifproductiseither18,24or28.

Sachin:"SincePrashantknowsthenumbers,theymustbeeither(3,8),(4,7)
or(5,6)."Buthewon'tbesure.Hence,thesumisnot11.

Summerisingdataforsum11:
PossibleSum PRODUCT PossibleSum
2+9

18

2+9=11(possible)
3+6=9

3+8

24

2+12=14
3+8=11(possible)
4+6=10

4+7

28

2+12=14
3+8=11(possible)
4+6=10

5+6

30

2+15=17(possible)
3+10=13
5+6=11(possible)

Followingthesameprocedurefor17:
PossibleSum PRODUCT PossibleSum
2+15

30

2+15=17(possible)
3+10=13
5+6=11(possible)

3+14

42

2+21=23(possible)
3+14=17(possible)
6+7=13

4+13

52

2+26=28

4+13=17(possible)
5+12

60

2+30=32
3+20=23(possible)
4+15=19
5+12=17(possible)
6+10=16

6+11

66

2+33=35(possible)
3+22=25
6+11=17(possible)

7+10

70

2+35=37(possible)
5+14=19
7+10=17(possible)

8+9

72

2+36=38
3+24=27(possible)
4+18=22
6+12=18
8+9=17(possible)

Here,Prashantwillbesureofthenumbersiftheproductis52.

Sachin:"SincePrashantknowsthenumbers,theymustbe(4,13)."

ForallothernumbersinthePossibleSumList,Prashantmightbesureofthe
numbersbutSachinwon't.

Hereisthestepbystepexplaination:

Sachin:"Asthesumis17,twonumberscanbeeither(2,15),(3,14),(4,13),
(5,12),(6,11),(7,10)or(8,9).Also,asnoneofthemisaprimenumberspair,
Prashantwon'tbeknowingnumberseither."

Prashant:"SinceSachinissurethatbothofusdon'tknowthenumbers,the
summustbeoneofthePossibleSumList.Further,astheproductis52,two
numberscanbeeither(2,26)or(4,13).Butiftheywere(2,26),Sachinwould
nothavebeensureinadvancethatIdon'tknowthenumbersas28(2+26)is

notinthePossibleSumList.Hence,twonumbersare4and13."

Sachin:"AsPrashantnowknowsboththenumbers,outofallpossible
products30(2,15),42(3,14),52(4,13),60(5,12),66(6,11),70(7,10),72(8,9)
thereisoneproductforwhichlistofallpossiblesumcontainsONLYONE
sumfromthePossibleSumList.Andalso,nosuchtwolistsexist.[seetable
abovefor17]Hence,twonumbersare4and13."

Nakulfiguredoutboththenumbersjustaswedidbyobservingthe
conversationbetweenSachinandPrashant.

Itisinterestingtonotethattherearenoothersuchtwonumbers.We
checkedallthepossiblesumstill500!!!
Substitutedigitsfortheletterstomakethefollowingsubtractionproblem
true.
SANTA

CLAUS

XMAS
Notethattheleftmostlettercan'tbezeroinanyword.Also,theremustbea
onetoonemappingbetweendigitsandletters.e.g.ifyousubstitute3forthe
letterM,nootherlettercanbe3andallotherMinthepuzzlemustbe3.
Answer

Oneofthesimplestbrainteaserastherearetotal26possibleanswers.

ItisobviousthatS=C+1.SinceAS=S,itisclearthatA=2*Sor2*s10.
Also,LandXareinterchangeable.
SANTA CLAUS = XMAS
24034

16492

= 7542

24034

17492

= 6542

24074

15432

= 8642

24074

18432

= 5642

24534

16492

= 8042

24534

18492

= 6042

24794

16452

= 8342

24794

18452

= 6342

24804

15462

= 9342

24804

19462

= 5342

24974

16432

= 8542

24974

18432

= 6542

36806

27643

= 9163

36806

29643

= 7163

36156

27693

= 8463

36156

28693

= 7463

62132

54206

= 7926

62132

57206

= 4926

62172

53246

= 8926

62172

58246

= 3926

62402

53276

= 9126

62402

59276

= 3126

62712

53286

= 9426

62712

59286

= 3426

62932

58206

= 4726

62932

54206

= 8726

Das könnte Ihnen auch gefallen